Deck 1 Flashcards
A 25 year old man is shot in the abdomen and is transferred to the operating theatre following arrival in the emergency department, as he is unstable and a FAST scan is positive. At operation there is an extensive laceration to the right lobe of the liver and involvement of the IVC. There is massive haemorrhage. What is the most appropriate approach to blood component therapy?
- Use Factor VIII concentrates early
- Avoid use of “o” negative blood
- Transfuse packed cells, FFP and platelets in fixed ratios of 1:1:1
- Transfuse packed cells and FFP in a fixed ratio of 4:1
- Perform goal directed transfusion based on the Hb, PT and TEG studies
- Transfuse packed cells, FFP and platelets in fixed ratios of 1:1:1
There is strong evidence to support the use of haemostatic transfusion in the setting of major haemorrhage due to trauma. This advocates the use of 1:1:1 ratios.
Uncontrolled haemorrhage accounts for up to 39% of all trauma related death. In the UK approximately 2% of all trauma patients will need massive transfusion.Massive transfusion is defined as the replacement of a patient’s total blood volume in less than 24 hours, or as the acute administration of more than half the patient’s estimated blood volume per hour. In haemorrhaging patients following trauma there is evidence to support the initial administration of tranexamic acid (CRASH study). During acute bleeding the practice of haemostatic resuscitation has been shown to reduce mortality rates. The principle of haemostatic resuscitation is that blood components are transfused in fixed ratios. For example; packed red cells, FFP and platelets are administered in a ratio of 1:1:1.
The typical therapeutic end points include: Hb: 8-10 g/dl Platelets > 100 PT and APTT (INR)< 1.5 Fibrinogen > 1.0 g/l Ca2+ > 1 mmol/l pH: 7.35-7.45 BE: +/- 2 ToC > 36 °C
A 76 year old woman with a body weight of 50 kg is undergoing an excision of a lipoma from her forehead. It is the first time the senior house officer has performed the procedure. He administers 30ml of 2% lignocaine to the area. The procedure is complicated by bleeding and the patient experiences discomfort, a further 10ml of the same anaesthetic formulation is then administered. Over the following 5 minutes the patient complains of tinnitus and becomes drowsy. Which of the drugs listed below should be administered?
Temazepam Lorazepam Naloxone Intralipid 20% Sodium bicarbonate 20%
Intralipid 20%
Intralipid is indicated for the treatment of local anaesthetic toxicity. In this case the safe dose of local anaesthetic has been exceeded and is thus this lady’s symptoms are likely to represent toxicity.
Local anaesthetic toxicity
Toxicity results from either accidental intravascular injection (rapid onset of symptoms-usually correct dose), or from excessive dosage (slower onset). Local anaesthetic agents not only exert a membrane stabilising effect on peripheral nerves but will also act on excitable membranes within the CNS and Heart. The sensory neurones in the CNS are suppressed before the motor ones. As a result the early symptoms will typically be those of circumoral paraesthesia and tinnitus, followed by falling GCS and eventually coma.
Management of toxicity
Stop injecting the anaesthetic agent
High flow 100% oxygen via face mask
Cardiovascular monitoring
Administer lipid emulsion (Intralipid 20%) at 1.5ml/Kg over 1 minute as a bolus
Consider lipid emulsion infusion, at 0.25ml/ Kg/ minute
If toxicity due to prilocaine then administer methylene blue
Safe doses
10ml of lignocaine 1% contains 100mg of drug, this would constitute 70% of the maximum safe dose in a 50 kg patient. Up to 7mg / kg can be administered if adrenaline is added to the solution.
A 27 year old man is involved in a road traffic accident. He is seen in the emergency department with chest pain. Clinical examination is essentially unremarkable and he is discharged. He subsequently is found dead at home. What is the most likely underlying injury?
Tracheobronchial tree injury Traumatic aortic disruption Cardiac laceration Diaphragmatic rupture Rupture of the oesophagus
Traumatic aortic disruption
Aortic injuries that do not die at the scene may have a contained haematoma. Clinical signs are subtle and the diagnosis may not be apparent on clinical examination. Without prompt treatment the haematoma usually bursts and the patient dies.
Thoracic aorta rupture
Mechanism of injury: Decelerating force i.e. RTA, fall from a great height
Most people die at scene
Survivors may have an incomplete laceration at the ligamentum arteriosum of the aorta.
Clinical features
Contained haematoma: persistent hypotension
Detected mainly by history, CXR changes
CXR changes
Widened mediastinum
Trachea/Oesophagus to right
Depression of left main stem bronchus
Widened paratracheal stripe/paraspinal interfaces
Space between aorta and pulmonary artery obliterated
Rib fracture/left haemothorax
Diagnosis
Angiography, usually CT aortogram.
Treatment
Repair or replacement. Ideally they should undergo endovascular repair.
A 68 year old man presents to the plastics team with severe burns to his hands. He is not distressed by the burns. He has bilateral charcot joints. On examination; there is loss of pain and temperature sensation of the upper limbs.
A. Osteomyelitis B. Potts disease of the spine C. Scheuermanns disease D. Transverse myelitis E. Tabes dorsalis F. Subacute degeneration of the cord G. Brown-Sequard syndrome H. Syringomyelia I. Epidural haematoma
The correct answer is Syringomyelia
This patient has syringomyelia which selectively affects the spinotholamic tracts. Syringomyelia is a disorder in which a cystic cavity forms within the spinal cord. The commonest variant is the Arnold- Chiari malformation in which the cavity connects with a congenital malformation affecting the cerebellum. Acquired forms of the condition may occur as a result of previous meningitis, surgery or tumours. Many neurological manifestations have been reported, although the classical variety spares the dorsal columns and medial lemniscus and affecting only the spinothalamic tract with loss of pain and temperature sensation. The bilateral distribution of this patients symptoms would therefore favor syringomyelia over SCID or Brown Sequard syndrome. Osteomyelitis would tend to present with back pain and fever in addition to any neurological signs. Epidural haematoma large enough to produce neurological impairment will usually have motor symptoms in addition to any selective sensory loss, and the history is usually shorter.
A 24 year old man presents with localised spinal pain over 2 months which is worsened on movement. He is known to be an IVDU. He has no history suggestive of tuberculosis. The pain is now excruciating at rest and not improving with analgesia. He has a temperature of 39 oC.
A. Osteomyelitis B. Potts disease of the spine C. Scheuermanns disease D. Transverse myelitis E. Tabes dorsalis F. Subacute degeneration of the cord G. Brown-Sequard syndrome H. Syringomyelia I. Epidural haematoma
In an IVDU with back pain and pyrexia have a high suspicion for osteomylelitis. The most likely organism is staph aureus and the cervical spine is the most common region affected. TB tends to affect the thoracic spine and in other causes of osteomyelitis the lumbar spine is affected.
A 22 year man is shot in the back, in the lumbar region. He has increased tone and hyper-reflexia of his right leg. He cannot feel his left leg.
A. Osteomyelitis B. Potts disease of the spine C. Scheuermanns disease D. Transverse myelitis E. Tabes dorsalis F. Subacute degeneration of the cord G. Brown-Sequard syndrome H. Syringomyelia I. Epidural haematoma
Brown -Sequard syndrome is caused by hemisection of the spinal cord. It may result from stab injuries or lateral vertebral fractures. It results in ipsilateral paralysis (pyramidal tract) , and also loss of proprioception and fine discrimination (dorsal columns). Pain and temperature sensation are lost on the contra-lateral side. This is because the fibres of the spinothalamic tract have decussated below the level of the cord transection.
A patient is brought to the emergency department following a motor vehicle accident. He is unconscious and has a deep scalp laceration. His heart rate is 120/min, blood pressure is 80/40 mmHg, and respiratory rate is 35/min. Despite rapid administration of 2 litres of Hartmans solution, the patient’s vital signs do not change significantly. The injury likely to explain this patient’s hypotension is:
Epidural haematoma Sub dural haematoma Intra parenchymal brain haemorrhage Base of skull fracture None of the above
In the patient described, hypotension and tachycardia should not be uncritically attributed to the head injury, since these findings in the setting of blunt trauma are suggestive of serious thoracic, abdominal, or pelvic hemorrhage. When cardiovascular collapse occurs as a result of rising intracranial pressure, it is generally accompanied by hypertension, bradycardia, and respiratory depression.
Head injury
Patients who suffer head injuries should be managed according to ATLS principles and extra cranial injuries should be managed alongside cranial trauma. Inadequate cardiac output will compromise CNS perfusion irrespective of the nature of the cranial injury.
Types of traumatic brain injury
Extradural haematoma Bleeding into the space between the dura mater and the skull. Often results from acceleration-deceleration trauma or a blow to the side of the head. The majority of extradural haematomas occur in the temporal region where skull fractures cause a rupture of the middle meningeal artery. Raised intracranial pressure
Some patients may exhibit a lucid interval
Subdural haematoma Bleeding into the outermost meningeal layer. Most commonly occur around the frontal and parietal lobes. May be either acute or chronic.
Risk factors include old age and alcoholism.
Slower onset of symptoms than a extradural haematoma.
Subarachnoid haemorrhage Usually occurs spontaneously in the context of a ruptured cerebral aneurysm, but may be seen in association with other injuries when a patient has sustained a traumatic brain injury.
Pathophysiology
Primary brain injury may be focal (contusion/ haematoma) or diffuse (diffuse axonal injury)
Diffuse axonal injury occurs as a result of mechanical shearing following deceleration, causing disruption and tearing of axons
Intra-cranial haematomas can be extradural, subdural or intracerebral, while contusions may occur adjacent to (coup) or contralateral (contre-coup) to the side of impact
Secondary brain injury occurs when cerebral oedema, ischaemia, infection, tonsillar or tentorial herniation exacerbates the original injury. The normal cerebral auto regulatory processes are disrupted following trauma rendering the brain more susceptible to blood flow changes and hypoxia
The Cushings reflex (hypertension and bradycardia) often occurs late and is usually a pre terminal event
Management
Where there is life threatening rising ICP such as in extra dural haematoma and whilst theatre is prepared or transfer arranged use of IV mannitol/ frusemide may be required.
Diffuse cerebral oedema may require decompressive craniotomy
Exploratory Burr Holes have little management in modern practice except where scanning may be unavailable and to thus facilitate creation of formal craniotomy flap
Depressed skull fractures that are open require formal surgical reduction and debridement, closed injuries may be managed non operatively if there is minimal displacement.
ICP monitoring is appropriate in those who have GCS 3-8 and normal CT scan.
ICP monitoring is mandatory in those who have GCS 3-8 and abnormal CT scan.
Hyponatraemia is most likely to be due to syndrome of inappropriate ADH secretion.
Minimum of cerebral perfusion pressure of 70mmHg in adults.
Minimum cerebral perfusion pressure of between 40 and 70 mmHg in children.
A 19 year old sportswoman presents with knee pain which is worse on walking down the stairs and when sitting still. On examination there is wasting of the quadriceps and pseudolocking of the knee.
A. Chondromalacia patellae B. Dislocated patella C. Undisplaced fracture patella D. Displaced patella fracture E. Avulsion fracture of the tibial tubercle F. Quadriceps tendon rupture G. Osgood Schlatters disease
Chondromalacia patellae
A teenage girl with knee pain on walking down the stairs is characteristic for chondromalacia patellae (anterior knee pain). Most cases are managed with physiotherapy.
Teenage girls, following an injury to knee e.g. Dislocation patella
Typical history of pain on going downstairs or at rest
Tenderness, quadriceps wasting
A tall 18 year old male athlete is admitted to the emergency room after being hit in the knee by a hockey stick. On examination his knee is tense and swollen. X-ray shows no fractures.
A. Chondromalacia patellae B. Dislocated patella C. Undisplaced fracture patella D. Displaced patella fracture E. Avulsion fracture of the tibial tubercle F. Quadriceps tendon rupture G. Osgood Schlatters disease
Dislocated patella
A patella dislocation is a common cause of haemarthrosis and many will spontaneously reduce when the leg is straightened. In the chronic setting physiotherapy is used to strengthen the quadriceps muscles.
Most commonly occurs as a traumatic primary event, either through direct trauma or through severe contraction of quadriceps with knee stretched in valgus and external rotation
Genu valgum, tibial torsion and high riding patella are risk factors
Skyline x-ray views of patella are required, although displaced patella may be clinically obvious
An osteochondral fracture is present in 5%
The condition has a 20% recurrence rate
An athletic 15 year old boy presents with knee pain of 3 weeks duration. It is worst during activity and settles with rest. On examination there is tenderness overlying the tibial tuberosity and an associated swelling at this site.
A. Chondromalacia patellae B. Dislocated patella C. Undisplaced fracture patella D. Displaced patella fracture E. Avulsion fracture of the tibial tubercle F. Quadriceps tendon rupture G. Osgood Schlatters disease
Osgood Schlatters disease
Athletic boys and girls may develop this condition in their teenage years. It is caused by multiple micro fractures at the point of insertion of the tendon into the tibial tuberosity. Most cases settle with physiotherapy and rest.
A 42 year old man is admitted to surgery with acute appendicitis. He is known to have hypertension, psoriatic arthropathy and polymyalgia rheumatica. His medical therapy includes: Paracetamol 1g qds Codeine phosphate 30mg qds Bendrofluazide 2.5 mg od Ramipril 10mg od Methotrexate 7.5mg once a week Prednisolone 5mg od You are called by the core surgical trainee to assess this man as he has become delirious and hypotensive 2 hours after surgery. His blood results reveal:
Na+ 132 mmol/l K+ 5.2 mmol/l Urea 10 mmol/l Creatinine 111 µmol/l Glucose 3.5 CRP 158
Hb 10.2 g/dl
Platelets 156 * 109/l
WBC 14 * 109/l
What is the most likely diagnosis?
Septic shock secondary to appendicitis Neutropenic sepsis Phaeochromocytoma Perforated bowel Addisonian crisis
Features of an addisonian crisis:
Hyponatraemia
Hyperkalaemia
Hypoglycaemia
This man is on steroids for polymyalgia rheumatica. Surgery can precipitate acute adrenal deficiency. The diagnosis is further confirmed by the blood results of hyponatraemia, hyperkalaemia and hypoglycaemia. This patient urgently needs hydrocortisone.
Addisonian crisis
Causes
Sepsis or surgery causing an acute exacerbation of chronic insufficiency (Addison’s, Hypopituitarism)
Adrenal haemorrhage eg Waterhouse-Friderichsen syndrome (fulminant meningococcemia)
Steroid withdrawal
Management
Hydrocortisone 100 mg im or iv
1 litre normal saline infused over 30-60 mins or with dextrose if hypoglycaemic
Continue hydrocortisone 6 hourly until the patient is stable. No fludrocortisone is required because high cortisol exerts weak mineralocorticoid action
Oral replacement may begin after 24 hours and be reduced to maintenance over 3-4 days
A 20 year old woman trips over a step, injuring her ankle. Examination reveals tenderness over the lateral malleolus and an x-ray demonstrates an undisplaced fracture distal to the syndesmosis.
A. Surgical fixation
B. Below knee amputation
C. Application of below knee plaster
D. Application of ankle boot
E. Application of external fixation device
F. Application of compression dressing and physiotherapy
G. Immediate reduction and application of backslab
Application of ankle boot
This is a Weber A fracture. It is a stable ankle injury and can therefore be managed conservatively. Whilst this patient could also be treated in a below knee plaster, most clinicians would nowadays treat this injury in an ankle boot. Patients should be advised to mobilise in the ankle boot, as pain allows, and can wean themselves out of the boot as the symptoms improve.
Danis-Weber
Commonly used. Based on the level of the fibula fracture in relation to the syndesmosis. The more proximal, the greater the risk of syndesmotic injury and therefore fracture instability.
A - fracture below the level of the syndesmosis
B - fracture at the level of the syndesmosis / level of the tibial plafond
C - fracture above the level of the syndesmosis. This includes Maisonneuve fractures (proximal fibula fracture), which can be associated with ankle instability. Beware the high fibula fracture - it may be an ankle fracture!
The Weber classification is based purely on the the lateral side. All injuries can include a medial or posterior bony or ligamentous injury which also dictates fracture stability (bimalleolar and trimalleolar fractures are more unstable).
Defining stability of an ankle fracture underpins the treatment decision.
Weber A - Unimalleolar Weber A Weber fractures by definition are stable and therefore can be mobilised fully weight bearing in an ankle boot.
Weber C - Fractures tend to include syndesmotic disruption and are usually bimalleolar (either bony or ligamentous). They are therefore unstable and usually require operative fixation. In addition to the fracture fixation, the syndesmosis usually requires reconstruction/augmentation with screws to restore the joint integrity and function.
Weber B - B fractures vary greatly. They can be part of a trimalleolar injury and therefore extremely unstable, requiring fixation. Alternatively, a uni-malleolar Weber B fracture can be a stable injury, and therefore mobilised immediately in an ankle boot. Defining the stability can be challenging, and often involves stress radiographs, or a trial of mobilisation and repeat radiographs. Defining stability is the subject of much ongoing research. However, treating undisplaced ankle fractures in a below knee plaster, non-weight bearing for six weeks is still widely practised, and a safe approach.
When operative fixation is appropriate, it is usually via open reduction and internal fixation using plates and screws. It must be carried out when soft tissue swelling has settled in order to minimise the risk of wound problems. This can often take a week to settle.
Lauge-Hansen
Comprises two parts: first part is the foot position, and the second part is the force applied. Useful for understanding the forces involved and therefore predict the ligamentous or bony injury. Results in four injury patterns:
Supination - Adduction (SA) - 10-20%
Supination - External rotation (SER) - 40-75%
Pronation - Abduction (PA) - 5-20%
Pronation - External rotation (PER) - 5-20%
A 30 year old man injures his ankle playing football. On examination he has tenderness over both medial and lateral malleoli. X-ray demonstrates a bimalleolar fracture with a displaced distal fibula fracture, at the level of the syndesmosis and fracture of the medial malleolus with talar shift. The ankle has been provisionally reduced and splinted in the emergency department.
A. Surgical fixation
B. Below knee amputation
C. Application of below knee plaster
D. Application of ankle boot
E. Application of external fixation device
F. Application of compression dressing and physiotherapy
G. Immediate reduction and application of backslab
Surgical fixation
This is an unstable fracture pattern with a Weber B fracture of the distal fibula and a fracture of the medial malleolus. Talar shift indicates loss of ankle mortice congruity. This injury should therefore be treated with surgical fixation.
The Weber classification is based purely on the the lateral side. All injuries can include a medial or posterior bony or ligamentous injury which also dictates fracture stability (bimalleolar and trimalleolar fractures are more unstable).
Defining stability of an ankle fracture underpins the treatment decision.
Weber A - Unimalleolar Weber A Weber fractures by definition are stable and therefore can be mobilised fully weight bearing in an ankle boot.
Weber C - Fractures tend to include syndesmotic disruption and are usually bimalleolar (either bony or ligamentous). They are therefore unstable and usually require operative fixation. In addition to the fracture fixation, the syndesmosis usually requires reconstruction/augmentation with screws to restore the joint integrity and function.
Weber B - B fractures vary greatly. They can be part of a trimalleolar injury and therefore extremely unstable, requiring fixation. Alternatively, a uni-malleolar Weber B fracture can be a stable injury, and therefore mobilised immediately in an ankle boot. Defining the stability can be challenging, and often involves stress radiographs, or a trial of mobilisation and repeat radiographs. Defining stability is the subject of much ongoing research. However, treating undisplaced ankle fractures in a below knee plaster, non-weight bearing for six weeks is still widely practised, and a safe approach.
When operative fixation is appropriate, it is usually via open reduction and internal fixation using plates and screws. It must be carried out when soft tissue swelling has settled in order to minimise the risk of wound problems. This can often take a week to settle.
Lauge-Hansen
Comprises two parts: first part is the foot position, and the second part is the force applied. Useful for understanding the forces involved and therefore predict the ligamentous or bony injury. Results in four injury patterns:
Supination - Adduction (SA) - 10-20%
Supination - External rotation (SER) - 40-75%
Pronation - Abduction (PA) - 5-20%
Pronation - External rotation (PER) - 5-20%
A 50 year old female slips on wet floor injuring her ankle. On examination, she has tenderness over the lateral and medial malleolus. X-rays demonstrate an undisplaced fracture of the distal fiibula at the level of the syndesmosis and a congruent ankle mortice.
A. Surgical fixation
B. Below knee amputation
C. Application of below knee plaster
D. Application of ankle boot
E. Application of external fixation device
F. Application of compression dressing and physiotherapy
G. Immediate reduction and application of backslab
Application of below knee plaster
This is a Weber B fracture and therefore potentially unstable. Medial malleolar tenderness indicates deltoid ligament injury. As the fracture is currently undisplaced and the ankle mortice is congruent, the injury can be initially managed conservatively in a below knee plaster but the patient should be monitored in the outpatient clinic for fracture displacement in the first few weeks.
The Weber classification is based purely on the the lateral side. All injuries can include a medial or posterior bony or ligamentous injury which also dictates fracture stability (bimalleolar and trimalleolar fractures are more unstable).
Defining stability of an ankle fracture underpins the treatment decision.
Weber A - Unimalleolar Weber A Weber fractures by definition are stable and therefore can be mobilised fully weight bearing in an ankle boot.
Weber C - Fractures tend to include syndesmotic disruption and are usually bimalleolar (either bony or ligamentous). They are therefore unstable and usually require operative fixation. In addition to the fracture fixation, the syndesmosis usually requires reconstruction/augmentation with screws to restore the joint integrity and function.
Weber B - B fractures vary greatly. They can be part of a trimalleolar injury and therefore extremely unstable, requiring fixation. Alternatively, a uni-malleolar Weber B fracture can be a stable injury, and therefore mobilised immediately in an ankle boot. Defining the stability can be challenging, and often involves stress radiographs, or a trial of mobilisation and repeat radiographs. Defining stability is the subject of much ongoing research. However, treating undisplaced ankle fractures in a below knee plaster, non-weight bearing for six weeks is still widely practised, and a safe approach.
When operative fixation is appropriate, it is usually via open reduction and internal fixation using plates and screws. It must be carried out when soft tissue swelling has settled in order to minimise the risk of wound problems. This can often take a week to settle.
Lauge-Hansen
Comprises two parts: first part is the foot position, and the second part is the force applied. Useful for understanding the forces involved and therefore predict the ligamentous or bony injury. Results in four injury patterns:
Supination - Adduction (SA) - 10-20%
Supination - External rotation (SER) - 40-75%
Pronation - Abduction (PA) - 5-20%
Pronation - External rotation (PER) - 5-20%
A 30 year old woman presents with pain and swelling of the left shoulder. There is a large radiolucent lesion in the head of the humerus extending to the subchondral plate.
A. Osteosarcoma B. Osteomalacia C. Osteoporosis D. Metastatic carcinoma E. Osteoblastoma F. Giant cell tumour G. Ewing's sarcoma
Giant cell tumours on x-ray have a ‘soap bubble’ appearance. They present as pain or pathological fractures. They commonly metastasize to the lungs.
A 72 year old woman has a lumbar vertebral crush fracture. She has hypocalcaemia and a low urinary calcium.
A. Osteosarcoma B. Osteomalacia C. Osteoporosis D. Metastatic carcinoma E. Osteoblastoma F. Giant cell tumour G. Ewing's sarcoma
Osteomalacia
Hypocalcemia and low urinary calcium are biochemical features of osteomalacia. Unfortunately surgeons do need to look at some blood results!
A 16 year old boy presents with severe groin pain after kicking a football. Imaging confirms a pelvic fracture. A previous pelvic x-ray performed 2 weeks ago shows a lytic lesion with ‘onion type’ periosteal reaction.
A. Osteosarcoma B. Osteomalacia C. Osteoporosis D. Metastatic carcinoma E. Osteoblastoma F. Giant cell tumour G. Ewing's sarcoma
Ewing’s sarcoma
A Ewings sarcoma is most common in males between 10-20 years. It can occur in girls. A lytic lesion with a lamellated or onion type periosteal reaction is a classical finding on x-rays. Most patients present with metastatic disease with a 5 year prognosis between 5-10%.
A 23 year old rugby player falls directly onto his shoulder. There is pain and swelling of the shoulder joint. The clavicle is prominent and there appears to be a step deformity.
A. Glenohumeral dislocation B. Acromioclavicular dislocation C. Sternoclavicular dislocation D. Biceps tendon tear E. Supraspinatus tear F. Fracture of the surgical neck of the humerus G. Infra spinatus tear
Acromioclavicular dislocation
Acromioclavicular joint (ACJ) dislocation normally occurs secondary to direct injury to the superior aspect of the acromion. Loss of shoulder contour and prominent clavicle are key features. Note; rotator cuff tears rarely occur in the second decade.
Shoulder fractures and dislocations
Fractures
Proximal humerus
Background
Third most common fragility fracture in the elderly.
Results from low energy fall in predominantly elderly females, or from high energy trauma in young males.
Can be associated with nerve injury (commonly axillary), and fracture-dislocation of the humeral head. Detailed neurological assessment is essential for all upper limb injuries.
Anatomy
Osteology
Consists of articular head, greater tuberosity, lesser tuberosity, metaphysis and diaphysis. Between the articular head and the tuberosities is the anatomical neck (previous physis). Between the tuberosities and the metaphysis is the surgical neck.
The supraspinatus, infraspinatus and teres minor muscles attach to the greater tuberosity. The subscapularis muscle attaches to the lesser tuberosity.
Vascular Supply
Humeral head is supplied by the anterior and posterior humeral circumflex arteries. Anatomical neck fractures are at greatest risk of osteonecrosis.
Imaging
Imaging aims to both delineate the fracture pattern, and confirm/exlude the presence of an associated dislocation.
Radiographs - True anteroposterior (AP), axillary lateral and/or scapula Y view.
CT - indicated to better define intra-articular involvement and to aid pre-operative planning. MRI is not useful for fracture imaging.
Classification
Description of the fracture is often more useful than classification. Particular attention should be paid to humeral alignment, fracture displacement, and greater tuberosity position (rotator cuff will pull the GT supero-posterioly, which can cause impingement problems with malunion).
- Neer Classification: Most commonly used. Describes fracture as 2,3,or 4 part depending upon the number main fragments. Also comments on the degree of displacement. Fragments:
-greater tuberosity
-lesser tuberosity
- articular surface
- shaft
Displacement: >1cm or angulation >45 degrees.
Treatment
The vast majority of proximal humeral fractures are minimally displaced, and therefore can be managed conservatively. This involves immobilisation in a polysling, and progressive mobilisation. Pendular exercise can commence at 14 days, and active abduction from 4-6 weeks.
A 22 year old man falls over and presents to casualty. A shoulder x-ray is performed, the radiologist comments that a Hill-Sachs lesion is present.
A. Glenohumeral dislocation B. Acromioclavicular dislocation C. Sternoclavicular dislocation D. Biceps tendon tear E. Supraspinatus tear F. Fracture of the surgical neck of the humerus G. Infra spinatus tear
Glenohumeral dislocation
A Hill-Sachs lesion occurs when the cartilage surface of the humerus is in contact with the rim of the glenoid. About 50% of anterior glenohumeral dislocations are associated with this lesion.
Shoulder fractures and dislocations
Fractures
Proximal humerus
Background
Third most common fragility fracture in the elderly.
Results from low energy fall in predominantly elderly females, or from high energy trauma in young males.
Can be associated with nerve injury (commonly axillary), and fracture-dislocation of the humeral head. Detailed neurological assessment is essential for all upper limb injuries.
Anatomy
Osteology
Consists of articular head, greater tuberosity, lesser tuberosity, metaphysis and diaphysis. Between the articular head and the tuberosities is the anatomical neck (previous physis). Between the tuberosities and the metaphysis is the surgical neck.
The supraspinatus, infraspinatus and teres minor muscles attach to the greater tuberosity. The subscapularis muscle attaches to the lesser tuberosity.
Vascular Supply
Humeral head is supplied by the anterior and posterior humeral circumflex arteries. Anatomical neck fractures are at greatest risk of osteonecrosis.
Imaging
Imaging aims to both delineate the fracture pattern, and confirm/exlude the presence of an associated dislocation.
Radiographs - True anteroposterior (AP), axillary lateral and/or scapula Y view.
CT - indicated to better define intra-articular involvement and to aid pre-operative planning. MRI is not useful for fracture imaging.
Classification
Description of the fracture is often more useful than classification. Particular attention should be paid to humeral alignment, fracture displacement, and greater tuberosity position (rotator cuff will pull the GT supero-posterioly, which can cause impingement problems with malunion).
- Neer Classification: Most commonly used. Describes fracture as 2,3,or 4 part depending upon the number main fragments. Also comments on the degree of displacement. Fragments:
-greater tuberosity
-lesser tuberosity
- articular surface
- shaft
Displacement: >1cm or angulation >45 degrees.
Treatment
The vast majority of proximal humeral fractures are minimally displaced, and therefore can be managed conservatively. This involves immobilisation in a polysling, and progressive mobilisation. Pendular exercise can commence at 14 days, and active abduction from 4-6 weeks.
An 82 year old female presents to A&E after tripping on a step. She complains of shoulder pain. On examination there is pain to 90o on abduction.
A. Glenohumeral dislocation B. Acromioclavicular dislocation C. Sternoclavicular dislocation D. Biceps tendon tear E. Supraspinatus tear F. Fracture of the surgical neck of the humerus G. Infra spinatus tear
Supraspinatus tear
A supraspinatus tear is the most common of rotator cuff tears. It occurs as a result of degeneration and is rare in younger adults.
Shoulder fractures and dislocations
Fractures
Proximal humerus
Background
Third most common fragility fracture in the elderly.
Results from low energy fall in predominantly elderly females, or from high energy trauma in young males.
Can be associated with nerve injury (commonly axillary), and fracture-dislocation of the humeral head. Detailed neurological assessment is essential for all upper limb injuries.
Anatomy
Osteology
Consists of articular head, greater tuberosity, lesser tuberosity, metaphysis and diaphysis. Between the articular head and the tuberosities is the anatomical neck (previous physis). Between the tuberosities and the metaphysis is the surgical neck.
The supraspinatus, infraspinatus and teres minor muscles attach to the greater tuberosity. The subscapularis muscle attaches to the lesser tuberosity.
Vascular Supply
Humeral head is supplied by the anterior and posterior humeral circumflex arteries. Anatomical neck fractures are at greatest risk of osteonecrosis.
Imaging
Imaging aims to both delineate the fracture pattern, and confirm/exlude the presence of an associated dislocation.
Radiographs - True anteroposterior (AP), axillary lateral and/or scapula Y view.
CT - indicated to better define intra-articular involvement and to aid pre-operative planning. MRI is not useful for fracture imaging.
Classification
Description of the fracture is often more useful than classification. Particular attention should be paid to humeral alignment, fracture displacement, and greater tuberosity position (rotator cuff will pull the GT supero-posterioly, which can cause impingement problems with malunion).
- Neer Classification: Most commonly used. Describes fracture as 2,3,or 4 part depending upon the number main fragments. Also comments on the degree of displacement. Fragments:
-greater tuberosity
-lesser tuberosity
- articular surface
- shaft
Displacement: >1cm or angulation >45 degrees.
Treatment
The vast majority of proximal humeral fractures are minimally displaced, and therefore can be managed conservatively. This involves immobilisation in a polysling, and progressive mobilisation. Pendular exercise can commence at 14 days, and active abduction from 4-6 weeks.
Pathological fractures
Genetic conditions, such as osteogenesis imperfecta, may cause pathological fractures.
Osteogenesis imperfecta
Defective osteoid formation due to congenital inability to produce adequate intercellular substances like osteoid, collagen and dentine.
Failure of maturation of collagen in all the connective tissues.
Radiology may show translucent bones, multiple fractures, particularly of the long bones, wormian bones (irregular patches of ossification) and a trefoil pelvis.
Subtypes
Type I The collagen is normal quality but insufficient quantity.
Type II- Poor collagen quantity and quality.
Type III- Collagen poorly formed. Normal quantity.
Type IV- Sufficient collagen quantity but poor quality.
Osteopetrosis Bones become harder and more dense. Autosomal recessive condition. It is commonest in young adults. Radiology reveals a lack of differentiation between the cortex and the medulla described as marble bone.
A 15 year-old boy presents to the out-patient clinic with tiredness, recurrent throat and chest infections, and gradual loss of vision. Multiple x-rays show brittle bones with no differentiation between the cortex and the medulla.
A. Rickets B. Craniocleidodysostosis C. Achondroplasia D. Scurvy E. Pagets disease F. Multiple myeloma G. Osteogenesis imperfecta H. Osteomalacia I. Osteopetrosis J. None of the above
Osteopetrosis
Osteopetrosis is an autosomal recessive condition. It is commonest in young adults. They may present with symptoms of anaemia or thrombocytopaenia due to decreased marrow space. Radiology reveals a lack of differentiation between the cortex and the medulla described as marble bone. These bones are very dense and brittle.
Pathological fractures
Genetic conditions, such as osteogenesis imperfecta, may cause pathological fractures.
Osteogenesis imperfecta
Defective osteoid formation due to congenital inability to produce adequate intercellular substances like osteoid, collagen and dentine.
Failure of maturation of collagen in all the connective tissues.
Radiology may show translucent bones, multiple fractures, particularly of the long bones, wormian bones (irregular patches of ossification) and a trefoil pelvis.
Subtypes
Type I The collagen is normal quality but insufficient quantity.
Type II- Poor collagen quantity and quality.
Type III- Collagen poorly formed. Normal quantity.
Type IV- Sufficient collagen quantity but poor quality.
Osteopetrosis Bones become harder and more dense. Autosomal recessive condition. It is commonest in young adults. Radiology reveals a lack of differentiation between the cortex and the medulla described as marble bone.
A 12 year-old boy who is small for his age presents to the clinic with poor muscular development and hyper-mobile fingers. His x-rays show multiple fractures of the long bones and irregular patches of ossification.
A. Rickets B. Craniocleidodysostosis C. Achondroplasia D. Scurvy E. Pagets disease F. Multiple myeloma G. Osteogenesis imperfecta H. Osteomalacia I. Osteopetrosis J. None of the above
Osteogenesis imperfecta
Osteogenesis imperfecta is caused by defective osteoid formation due to congenital inability to produce adequate intercellular substances like osteoid, collagen and dentine. There is a failure of maturation of collagen in all the connective tissues.Radiology may show translucent bones, multiple fractures, particularly of the long bones, wormian bones (irregular patches of ossification) and a trefoil pelvis.
A 1 year-old is brought to the Emergency Department with a history of failure to thrive. On examination, the child is small for age and has a large head. X-ray shows a cupped appearance of the epiphysis of the wrist.
A. Rickets B. Craniocleidodysostosis C. Achondroplasia D. Scurvy E. Pagets disease F. Multiple myeloma G. Osteogenesis imperfecta H. Osteomalacia I. Osteopetrosis J. None of the above
Rickets
Rickets is the childhood form of osteomalacia. It is due to the failure of the osteoid to ossify due to vitamin D deficiency. Symptoms start about the age of one. The child is small for age and there is a history of failure to thrive. Bony deformities include bowing of the femur and tibia, a large head, deformity of the chest wall with thickening of the costochondral junction (rickettary rosary), and a transverse sulcus in the chest caused by the pull of the diaphragm (Harrison’s sulcus). X- Rays show widening and cupping of the epiphysis of the long bones, most readily apparent in the wrist.
Pathological fractures
Genetic conditions, such as osteogenesis imperfecta, may cause pathological fractures.
Osteogenesis imperfecta
Defective osteoid formation due to congenital inability to produce adequate intercellular substances like osteoid, collagen and dentine.
Failure of maturation of collagen in all the connective tissues.
Radiology may show translucent bones, multiple fractures, particularly of the long bones, wormian bones (irregular patches of ossification) and a trefoil pelvis.
Subtypes
Type I The collagen is normal quality but insufficient quantity.
Type II- Poor collagen quantity and quality.
Type III- Collagen poorly formed. Normal quantity.
Type IV- Sufficient collagen quantity but poor quality.
Osteopetrosis Bones become harder and more dense. Autosomal recessive condition. It is commonest in young adults. Radiology reveals a lack of differentiation between the cortex and the medulla described as marble bone.
A 24 year old motorist is involved in a road traffic accident in which he collides with the wall of a tunnel in a head on car crash, speed 85mph. He is wearing a seatbelt and the airbags have deployed. When rescuers arrive he is lucid and conscious and then dies suddenly.
A. Tension pneumothorax B. Haemopericardium C. Haemothorax D. Aortic transection E. Ruptured spleen F. Duodeno-jejunal flexure disruption G. Aorto iliac disruption H. Ileo-colic junction disruption
Aortic transection
Aortic transections typically occur distal to the ligamentum arteriosum. A temporary haematoma may prevent the immediate death that usually occurs. This is a deceleration injury. A widened mediastinum may be seen on x-ray.
A 30 year old women is involved in a road traffic accident she is a passenger in a car involved in a head on collision with another vehicle. Her car is travelling at 60mph. She has been haemodynamically stable throughout with only minimal tachycardia. On examination she has marked abdominal tenderness and a large amount of intra abdominal fluid on CT scan
A. Tension pneumothorax B. Haemopericardium C. Haemothorax D. Aortic transection E. Ruptured spleen F. Duodeno-jejunal flexure disruption G. Aorto iliac disruption H. Ileo-colic junction disruption
Duodeno-jejunal flexure disruption
This is another site of sudden deceleration injury. Given the large amount of free fluid, if it were blood, then a greater degree of haemodynamic instability would be expected.
A 17 year old boy is involved in a motorcycle accident in which he is thrown from his motorcycle. On admission he has distended neck veins and a weak pulse. The trachea is central.
A. Tension pneumothorax B. Haemopericardium C. Haemothorax D. Aortic transection E. Ruptured spleen F. Duodeno-jejunal flexure disruption G. Aorto iliac disruption H. Ileo-colic junction disruption
Haemopericardium
This is most likely a cardiac tamponade produced by haemopericardium. As little as 100ml of blood may result in tamponade as the pericardial sac is not distensible. Diagnosis is suggested by muffled heart sounds, paradoxical pulse and jugular vein distension.
A 6 year old boy pulls over a kettle and suffers superficial partial thickness burns to his legs. Which of the following will not occur?
Preservation of hair follicles Formation of vesicles or bullae Damage to sweat glands Healing by re-epithelialisation Pain at the burn site
Damage to sweat glands
Partial thickness burns are divided into superficial and deep burns, however, this is often not possible on initial assessment and it may be a week or more before the distinction is clear cut. Dermal appendages are, by definition, intact. Superficial partial thickness burns will typically heal by re-epithelialisation, deeper burns will heal with scarring.
Burns may be thermal, chemical or electrical. In the former category are burns which occur as a result of heat. Chemical burns occur when the skin is exposed to an extremely caustic or alkaline substance. Electrical burns occur following exposure to electrical current. The immediate management includes removal of the burning source which usually includes irrigation of the burned area. A detailed assessment then needs to be made of the extent of the burns and a number of charts are available for recording this information. The degree of injury relates to the temperature and duration of exposure. Most domestic burns are mainly scalds in young children.
Following the burn, there is a local response with progressive tissue loss and release of inflammatory cytokines. Systemically, there are cardiovascular effects resulting from fluid loss and sequestration of fluid into the third space. There is a marked catabolic response. Immunosupression is common with large burns and bacterial translocation from the gut lumen is a recognised event. Sepsis is a common cause of death following major burns.
Management
The initial aim is to stop the burning process and resuscitate the patient. Intravenous fluids will be required for children with burns greater than 10% of total body surface area. Adults with burns greater than 15% of total body surface area will also require IV fluids. The fluids are calculated using the Parkland formula which is; volume of fluid= total body surface area of the burn % x weight (Kg) x4. Half of the fluid is administered in the first 8 hours. A urinary catheter should be inserted. Analgesia should be given. Complex burns, burns involving the hand perineum and face and burns >10% in adults and >5% in children should be transferred to a burns unit.
Circumferential burns affecting a limb or severe torso burns impeding respiration may require escharotomy to divide the burnt tissue.
Conservative management is appropriate for superficial burns and mixed superficial burns that will heal in 2 weeks. More complex burns may require excision and skin grafting. Excision and primary closure is not generally practised as there is a high risk of infection.
There is no evidence to support the use of anti microbial prophylaxis or topical antibiotics in burn patients.
Escharotomies
Indicated in circumferential full thickness burns to the torso or limbs.
Careful division of the encasing band of burn tissue will potentially improve ventilation (if the burn involves the torso), or relieve compartment syndrome and oedema (where a limb is involved)
Based on the current guidelines, which option regarding management of head injuries is false?
Opiates should be avoided
Consider intubation if the GCS is <8 or = 8
Immediate CT head if there is > 1 episode of vomiting
Half hourly GCS assessment until GCS is 15
Contact neurosurgeons if suspected penetrating injury
Opiates should be avoided
Head injury management- NICE Guidelines
Summary of guidelines
All patients should be assessed within 15 minutes on arrival to A&E
Document all 3 components of the GCS
If GCS <8 or = to 8, consider stabilising the airway
Treat pain with low dose IV opiates (if safe)
Full spine immobilisation until assessment if:
- GCS < 15
- neck pain/tenderness
- paraesthesia extremities
- focal neurological deficit
- suspected c-spine injury
If a c-spine injury is suspected a 3 view c-spine x-ray is indicated. CT c-spine is preferred if:
- Intubated
- GCS <13
- Normal x-ray but continued concerns regarding c-spine injury
- Any focal neurology
- A CT head scan is being performed
- Initial plain films are abnormal
Immediate CT head (within 1 hour) if: GCS < 13 on admission GCS < 15 2 hours after admission Suspected open or depressed skull fracture Suspected skull base fracture (panda eyes, Battle's sign, CSF from nose/ear, bleeding ear) Focal neurology Vomiting > 1 episode Post traumatic seizure Coagulopathy
Contact neurosurgeon if: Persistent GCS < 8 or = 8 Unexplained confusion > 4h Reduced GCS after admission Progressive neurological signs Incomplete recovery post seizure Penetrating injury Cerebrospinal fluid leak
Observations
1/2 hourly GCS until 15
A 73 year old lady develops a cold, pulseless hand 3 days following a myocardial infarction.
A. Vasculitis B. Steal syndrome C. Thrombosis D. Foreign body embolus E. Clot embolus F. Vasospasm G. Direct arterial injury
Clot embolus
The development of mural or atrial appendage thrombi may occur following a myocardial infarct and co-existing atrial fibrillation may contribute to the formation. They tend to present with classical features of an embolic event.
A 6 year old child has suffered a displaced supracondylar humeral fracture. On examination, they have a cold and insensate hand with absent pulses.
A. Vasculitis B. Steal syndrome C. Thrombosis D. Foreign body embolus E. Clot embolus F. Vasospasm G. Direct arterial injury
Direct arterial injury
Both vasospasm and arterial injury may complicate supracondylar fractures and are seen in 1% of all cases. Vasospasm is usually transient and more likely when the injury is minor and reduced early. Severely displaced injuries and those with more advanced signs are usually associated with direct arterial injury.
A 26 year old man who smokes heavily develops aching, crampy pains in his legs. On examination distal limb pulses are diminished.
A. Vasculitis B. Steal syndrome C. Thrombosis D. Foreign body embolus E. Clot embolus F. Vasospasm G. Direct arterial injury
Vasculitis
This is likely to represent Buergers disease. It is commonest in young males who smoke heavily.
A 30 year old man is admitted overnight, following a road traffic accident. He has an open tibial fracture with a 20 cm wound and extensive periosteal stripping. He is neurovascularly intact and IV antibiotics and wound dressing have been administered in the emergency department.
A. Immediate skeletal stabilisation and application of negative pressure dressing
B. Combined skeletal and soft tissue reconstruction on a scheduled operating list
C. Thorough wound debridement in the emergency department
D. Immediate vascuIar shunting, followed by temporary skeletal stabilisation and vascular reconstruction
E. Intravenous antibiotics, photography and application of saline soaked gauze with impermeable dressing
F. Application of external fixator and conversion to internal fixation after two weeks
G. Fasciotomy with four compartment decompression
H. Skeletal fixation followed by vascular reconstruction
Combined skeletal and soft tissue reconstruction on a scheduled operating list
This patient has a Gustillo-Anderson Grade 3B open fracture. He will require definitive skeletal and soft tissue reconstruction, which should be performed on a combined ortho-plastic scheduled operating list, as per the BOA/BAPRAS guidelines. The surgery does not have to be performed out of scheduled hours unless there is marine/ sewage contamination, vascular compromise or it is a polytrauma.
Whilst it is reasonable to apply an external fixator prior to definitive skeletal and soft tissue reconstruction, this should be converted to internal fixation within 72 hours.
A 50 year old man is admitted after falling from scaffolding. He has an open fracture of his tibia with a 15 cm wound. He is neurovascularly intact.
A. Immediate skeletal stabilisation and application of negative pressure dressing
B. Combined skeletal and soft tissue reconstruction on a scheduled operating list
C. Thorough wound debridement in the emergency department
D. Immediate vascuIar shunting, followed by temporary skeletal stabilisation and vascular reconstruction
E. Intravenous antibiotics, photography and application of saline soaked gauze with impermeable dressing
F. Application of external fixator and conversion to internal fixation after two weeks
G. Fasciotomy with four compartment decompression
H. Skeletal fixation followed by vascular reconstruction
Intravenous antibiotics, photography and application of saline soaked gauze with impermeable dressing
The initial management of open fractures should include administration of intravenous antibiotics, photography of wound and application of a sterile soaked gauze and impermeable film. The wound should only be handled to remove gross contamination. The patient is then likely to require definitive skeletal and soft tissue reconstruction.
A 40 year old woman is admitted after being knocked off her bike. She has an open fracture of her tibia, with a 10 cm wound. No peripheral pulses are palpable. Intravenous antibiotics have been administered in the emergency department and the wound has been dressed.
A. Immediate skeletal stabilisation and application of negative pressure dressing
B. Combined skeletal and soft tissue reconstruction on a scheduled operating list
C. Thorough wound debridement in the emergency department
D. Immediate vascuIar shunting, followed by temporary skeletal stabilisation and vascular reconstruction
E. Intravenous antibiotics, photography and application of saline soaked gauze with impermeable dressing
F. Application of external fixator and conversion to internal fixation after two weeks
G. Fasciotomy with four compartment decompression
H. Skeletal fixation followed by vascular reconstruction
Immediate vascuIar shunting, followed by temporary skeletal stabilisation and vascular reconstruction
This patient has a Gustillo-Anderson Grade 3C open fracture with vascular injury. Vascular impairment requires immediate surgery and restoration of circulation, ideally within 3-4 hours. This should follow the sequence of shunting, temporary skeletal stabilisation and then vascular reconstruction as per BOA / BAPRAS guidelines. Revascularisation using vascular shunts should be performed before skeletal fixation.
A 23 year old man sustains a severe facial fracture and reconstruction is planned. Which of the following investigations will facilitate pre-operative planning?
Mandibular tomography Magnetic resonance scan of face Skull X-ray Computerised tomography of the head Orthopantomogram
Computerised tomography of the head
Significant facial fractures may have intracranial communication. CT scanning will allow delineation of injury extent and 3D reconstruction images can be created. An Orthopantomogram (OPT) will provide good images of mandible and surrounding bony structures but will not give intracranial detail. A skull x-ray lacks the detail for modern practice.
Ocular injuries
Superior orbital fissure syndrome
Severe force to the lateral wall of the orbit resulting in compression of neurovascular structures. Results in :
Complete opthalmoplegia and ptosis (Cranial nerves 3, 4, 6 and nerve to levator palpebrae superioris)
Relative afferent pupillary defect
Dilatation of the pupil and loss of accommodation and corneal reflexes
Altered sensation from forehead to vertex (frontal branch of trigeminal nerve)
Orbital blow out fracture
Typically occurs when an object of slightly larger diameter than the orbital rim strikes the incompressible eyeball. The bone fragment is displaced downwards into the antral cavity, remaining attached to the orbital periosteum. Periorbital fat may be herniated through the defect, interfering with the inferior rectus and inferior oblique muscles which are contained within the same fascial sheath. This prevents upward movement and outward rotation of the eye and the patient experiences diplopia on upward gaze. The initial bruising and swelling may make assessment difficult and patients should usually be reviewed 5 days later. Residual defects may require orbital floor reconstruction.
Nasal Fractures
Common injury
Ensure new and not old deformity
Control epistaxis
CSF rhinorrhoea implies that the cribriform plate has been breached and antibiotics will be required.
Usually best to allow bruising and swelling to settle and then review patient clinically. Major persistent deformity requires fracture manipulation, best performed within 10 days of injury.
Retrobulbar haemorrhage
Rare but important ocular emergency. Presents with:
Pain (usually sharp and within the globe)
Proptosis
Pupil reactions are lost
Paralysis (eye movements lost)
Visual acuity is lost (colour vision is lost first)
May be the result of Le Fort type facial fractures.
Management:
Mannitol 1g/Kg as 20% infusion, Osmotic diuretic, Contra-indicated in congestive heart failure and pulmonary oedema
Acetazolamide 500mg IV, (Monitor FBC/U+E) Reduces aqueous pressure by inhibition of carbonic anhydrase (used in glaucoma)
Dexamethasone 8mg orally or intravenously
In a traumatic setting an urgent cantholysis may be needed prior to definitive surgery.
Consider
Papaverine 40mg smooth muscle relaxant
Dextran 40 500mls IV improves perfusion
A 32 year old man presents with a painful swelling over the volar aspect of his hand after receiving a hard blow to his palm. On examination, he experiences pain on moving the wrist and on longitudinal compression of the thumb.
A. Pulled elbow B. Fracture of the coronoid process C. Scaphoid fracture D. Fracture of the distal humerus E. Bennets fracture F. Fracture of the shaft of the radius and ulnar G. Galeazzi fracture H. Fracture of the olecranon I. Fracture of the radial head
Scaphoid fracture
Scaphoid fractures usually occur as a result of direct hard blow to the palm or following a fall on the out-stretched hand. The main physical signs are swelling and tenderness in the anatomical snuff box, and pain on wrist movements and on longitudinal compression of the thumb
Upper limb fractures
Colles’ fracture
Fall onto extended outstretched hands
Described as a dinner fork type deformity
Classical Colles’ fractures have the following 3 features:
Features of the injury
- Transverse fracture of the radius
- 1 inch proximal to the radio-carpal joint
- Dorsal displacement and angulation
Smith’s fracture (reverse Colles’ fracture)
Volar angulation of distal radius fragment (Garden spade deformity)
Caused by falling backwards onto the palm of an outstretched hand or falling with wrists flexed
Bennett’s fracture
Intra-articular fracture of the first carpometacarpal joint
Impact on flexed metacarpal, caused by fist fights
X-ray: triangular fragment at ulnar base of metacarpal
Monteggia’s fracture
Dislocation of the proximal radioulnar joint in association with an ulna fracture
Fall on outstretched hand with forced pronation
Needs prompt diagnosis to avoid disability
Galeazzi fracture
Radial shaft fracture with associated dislocation of the distal radioulnar joint
Occur after a fall on the hand with a rotational force superimposed on it.
On examination, there is bruising, swelling and tenderness over the lower end of the forearm.
X Rays reveal the displaced fracture of the radius and a prominent ulnar head due to dislocation of the inferior radio-ulnar joint.
Barton’s fracture
Distal radius fracture (Colles’/Smith’s) with associated radiocarpal dislocation
Fall onto extended and pronated wrist
Scaphoid fractures
Scaphoid fractures are the commonest carpal fractures.
Surface of scaphoid is covered by articular cartilage with small area available for blood vessels (fracture risks blood supply)
Forms floor of anatomical snuffbox
Risk of fracture associated with fall onto outstretched hand (tubercle, waist, or proximal 1/3)
The main physical signs are swelling and tenderness in the anatomical snuff box, and pain on wrist movements and on longitudinal compression of the thumb.
Ulnar deviation AP needed for visualization of scaphoid
Immobilization of scaphoid fractures difficult
Radial head fracture
Fracture of the radial head is common in young adults.
It is usually caused by a fall on the outstretched hand.
On examination, there is marked local tenderness over the head of the radius, impaired movements at the elbow, and a sharp pain at the lateral side of the elbow at the extremes of rotation (pronation and supination).
A 26 year old man presents to the emergency department with a swelling over his left elbow after a fall on an outstretched hand. On examination, he has tenderness over the proximal part of his forearm, and has severely restricted supination and pronation movements.
A. Pulled elbow B. Fracture of the coronoid process C. Scaphoid fracture D. Fracture of the distal humerus E. Bennets fracture F. Fracture of the shaft of the radius and ulnar G. Galeazzi fracture H. Fracture of the olecranon I. Fracture of the radial head
Fracture of the radial head
Fracture of the radial head is common in young adults. It is usually caused by a fall on the outstretched hand. On examination, there is marked local tenderness over the head of the radius, impaired movements at the elbow, and a sharp pain at the lateral side of the elbow at the extremes of rotation (pronation and supination).
Upper limb fractures
Colles’ fracture
Fall onto extended outstretched hands
Described as a dinner fork type deformity
Classical Colles’ fractures have the following 3 features:
Features of the injury
- Transverse fracture of the radius
- 1 inch proximal to the radio-carpal joint
- Dorsal displacement and angulation
Smith’s fracture (reverse Colles’ fracture)
Volar angulation of distal radius fragment (Garden spade deformity)
Caused by falling backwards onto the palm of an outstretched hand or falling with wrists flexed
Bennett’s fracture
Intra-articular fracture of the first carpometacarpal joint
Impact on flexed metacarpal, caused by fist fights
X-ray: triangular fragment at ulnar base of metacarpal
Monteggia’s fracture
Dislocation of the proximal radioulnar joint in association with an ulna fracture
Fall on outstretched hand with forced pronation
Needs prompt diagnosis to avoid disability
Galeazzi fracture
Radial shaft fracture with associated dislocation of the distal radioulnar joint
Occur after a fall on the hand with a rotational force superimposed on it.
On examination, there is bruising, swelling and tenderness over the lower end of the forearm.
X Rays reveal the displaced fracture of the radius and a prominent ulnar head due to dislocation of the inferior radio-ulnar joint.
Barton’s fracture
Distal radius fracture (Colles’/Smith’s) with associated radiocarpal dislocation
Fall onto extended and pronated wrist
Scaphoid fractures
Scaphoid fractures are the commonest carpal fractures.
Surface of scaphoid is covered by articular cartilage with small area available for blood vessels (fracture risks blood supply)
Forms floor of anatomical snuffbox
Risk of fracture associated with fall onto outstretched hand (tubercle, waist, or proximal 1/3)
The main physical signs are swelling and tenderness in the anatomical snuff box, and pain on wrist movements and on longitudinal compression of the thumb.
Ulnar deviation AP needed for visualization of scaphoid
Immobilization of scaphoid fractures difficult
Radial head fracture
Fracture of the radial head is common in young adults.
It is usually caused by a fall on the outstretched hand.
On examination, there is marked local tenderness over the head of the radius, impaired movements at the elbow, and a sharp pain at the lateral side of the elbow at the extremes of rotation (pronation and supination).
A 56 year old lady presents with a painful swelling over the lower end of the forearm following a fall. Imaging reveals a distal radial fracture with disruption of the distal radio-ulnar joint.
A. Pulled elbow B. Fracture of the coronoid process C. Scaphoid fracture D. Fracture of the distal humerus E. Bennets fracture F. Fracture of the shaft of the radius and ulnar G. Galeazzi fracture H. Fracture of the olecranon I. Fracture of the radial head
Galeazzi fracture
Galeazzi fractures occur after a fall on the hand with a rotational force superimposed on it. On examination, there is bruising, swelling and tenderness over the lower end of the forearm. X- Rays reveal a displaced fracture of the radius and a prominent ulnar head due to dislocation of the inferior radio-ulnar joint.
Upper limb fractures
Colles’ fracture
Fall onto extended outstretched hands
Described as a dinner fork type deformity
Classical Colles’ fractures have the following 3 features:
Features of the injury
- Transverse fracture of the radius
- 1 inch proximal to the radio-carpal joint
- Dorsal displacement and angulation
Smith’s fracture (reverse Colles’ fracture)
Volar angulation of distal radius fragment (Garden spade deformity)
Caused by falling backwards onto the palm of an outstretched hand or falling with wrists flexed
Bennett’s fracture
Intra-articular fracture of the first carpometacarpal joint
Impact on flexed metacarpal, caused by fist fights
X-ray: triangular fragment at ulnar base of metacarpal
Monteggia’s fracture
Dislocation of the proximal radioulnar joint in association with an ulna fracture
Fall on outstretched hand with forced pronation
Needs prompt diagnosis to avoid disability
Galeazzi fracture
Radial shaft fracture with associated dislocation of the distal radioulnar joint
Occur after a fall on the hand with a rotational force superimposed on it.
On examination, there is bruising, swelling and tenderness over the lower end of the forearm.
X Rays reveal the displaced fracture of the radius and a prominent ulnar head due to dislocation of the inferior radio-ulnar joint.
Barton’s fracture
Distal radius fracture (Colles’/Smith’s) with associated radiocarpal dislocation
Fall onto extended and pronated wrist
Scaphoid fractures
Scaphoid fractures are the commonest carpal fractures.
Surface of scaphoid is covered by articular cartilage with small area available for blood vessels (fracture risks blood supply)
Forms floor of anatomical snuffbox
Risk of fracture associated with fall onto outstretched hand (tubercle, waist, or proximal 1/3)
The main physical signs are swelling and tenderness in the anatomical snuff box, and pain on wrist movements and on longitudinal compression of the thumb.
Ulnar deviation AP needed for visualization of scaphoid
Immobilization of scaphoid fractures difficult
Radial head fracture
Fracture of the radial head is common in young adults.
It is usually caused by a fall on the outstretched hand.
On examination, there is marked local tenderness over the head of the radius, impaired movements at the elbow, and a sharp pain at the lateral side of the elbow at the extremes of rotation (pronation and supination).
A 42 year old skier falls and impacts his hand on his ski pole. On examination he is tender in the anatomical snuffbox and on bimanual palpation. X-rays with scaphoid views show no evidence of fracture.
A. Admission and surgical debridement
B. Application of futura splint and fracture clinic review
C. Application of tubigrip bandage and fracture clinic review
D. Admission for open reduction and fixation
E. Discharge with reassurance
F. Commence oral prednisolone
G. Commence oral diclofenac
Application of futura splint and fracture clinic review
A fracture may still be present and should be immobilised until repeat imaging can be performed. If clinical suspicion persists then subsequent imaging should be with MRI scanning or CT if MRI is contra-indicated.
Scaphoid fractures:
80% of all carpal fractures
80% occur in men
80% occur at the waist of the scaphoid
Scaphoid fractures are the commonest carpal fractures.
Surface of scaphoid is covered by articular cartilage with small area available for blood vessels (fracture risks blood supply)
Forms floor of anatomical snuffbox
Risk of fracture associated with fall onto outstretched hand (tubercle, waist, or proximal third)
Ulnar deviation AP needed for visualization of scaphoid
Immobilization of scaphoid fractures difficult
Complications
Non union of scaphoid
Avascular necrosis of the scaphoid
Scapholunate disruption and wrist collapse
Degenerative changes of the adjacent joint
A 43 year old man falls over landing on his left hand. Although there was anatomical snuffbox tenderness no x-rays either at the time or subsequently have shown evidence of scaphoid fracture. He has been immobilised in a futura splint for two weeks and is now asymptomatic.
A. Admission and surgical debridement
B. Application of futura splint and fracture clinic review
C. Application of tubigrip bandage and fracture clinic review
D. Admission for open reduction and fixation
E. Discharge with reassurance
F. Commence oral prednisolone
G. Commence oral diclofenac
Discharge with reassurance
This patient is at extremely low risk of having sustained a scaphoid injury and may be discharged.
Scaphoid fractures:
80% of all carpal fractures
80% occur in men
80% occur at the waist of the scaphoid
Scaphoid fractures are the commonest carpal fractures.
Surface of scaphoid is covered by articular cartilage with small area available for blood vessels (fracture risks blood supply)
Forms floor of anatomical snuffbox
Risk of fracture associated with fall onto outstretched hand (tubercle, waist, or proximal third)
Ulnar deviation AP needed for visualization of scaphoid
Immobilization of scaphoid fractures difficult
Complications
Non union of scaphoid
Avascular necrosis of the scaphoid
Scapholunate disruption and wrist collapse
Degenerative changes of the adjacent joint
A builder falls from scaffolding and lands on his left hand he suffers a severe laceration to his palm. An x-ray shows evidence of scaphoid fracture that is minimally displaced.
A. Admission and surgical debridement
B. Application of futura splint and fracture clinic review
C. Application of tubigrip bandage and fracture clinic review
D. Admission for open reduction and fixation
E. Discharge with reassurance
F. Commence oral prednisolone
G. Commence oral diclofenac
Admission and surgical debridement
This is technically an open fracture and should be debrided prior to attempted fixation (which should occur soon after).
Scaphoid fractures:
80% of all carpal fractures
80% occur in men
80% occur at the waist of the scaphoid
Scaphoid fractures are the commonest carpal fractures.
Surface of scaphoid is covered by articular cartilage with small area available for blood vessels (fracture risks blood supply)
Forms floor of anatomical snuffbox
Risk of fracture associated with fall onto outstretched hand (tubercle, waist, or proximal third)
Ulnar deviation AP needed for visualization of scaphoid
Immobilization of scaphoid fractures difficult
Complications
Non union of scaphoid
Avascular necrosis of the scaphoid
Scapholunate disruption and wrist collapse
Degenerative changes of the adjacent joint
A 4 year boy presents with an abnormal gait. He has a history of recent viral illness. His WCC is 11 and ESR is 30.
A. Musculoskeletal pain B. Congenital dysplasia of the hip C. Slipped upper femoral epiphysis D. Transient synovitis E. Septic arthritis F. Perthes disease G. Tibial fracture
Transient synovitis
Viral illnesses can be associated with transient synovitis. The WCC should ideally be > 12 and the ESR > 40 to suggest septic arthritis.
A 6 year old boy presents with groin pain. He is known to be disruptive in class. He reports that he is bullied for being short. On examination he has an antalgic gait and pain on internal rotation of the right hip.
A. Musculoskeletal pain B. Congenital dysplasia of the hip C. Slipped upper femoral epiphysis D. Transient synovitis E. Septic arthritis F. Perthes disease G. Tibial fracture
Perthes disease
This child is short, has hyperactivity (disruptive behaviour) and is within the age range for Perthes disease. Hyperactivity and short stature are associated with Perthes disease.
An obese 12 year old boy is referred with pain in the left knee and hip. On examination he has an antaglic gait and limitation of internal rotation. His knee has normal range of passive and active movement.
A. Musculoskeletal pain B. Congenital dysplasia of the hip C. Slipped upper femoral epiphysis D. Transient synovitis E. Septic arthritis F. Perthes disease G. Tibial fracture
Slipped upper femoral epiphysis
Slipped upper femoral epiphysis is commonest in obese adolescent males. The x-ray will show displacement of the femoral epiphysis inferolaterally. Treatment is usually with rest and non weight bearing crutches.
A 63 year old man is admitted with rest pain and foot ulceration. An angiogram shows a 3 cm area of occlusion of the distal superficial femoral artery with 3 vessel run off. His ankle - brachial pressure index is 0.4.
A. Primary amputation B. Angioplasty C. Arterial bypass surgery using vein D. Arterial bypass surgery using PTFE E. Conservative management with medical therapy and exercise F. Watch and wait G. Duplex scanning
Angioplasty
Short segment disease and good run off with tissue loss is a compelling indication for angioplasty. He should receive aspirin and a statin if not already taking them.
Peripheral vascular disease
Indications for surgery to revascularise the lower limb Intermittent claudication Critical ischaemia Ulceration Gangrene
Intermittent claudication that is not disabling may provide a relative indication, whilst the other complaints are often absolute indications depending upon the frailty of the patient.
Assessment
Clinical examination
Ankle brachial pressure index measurement
Duplex arterial ultrasound
Angiography (standard, CT or MRI): usually performed only if intervention being considered.
Angioplasty
In order for angioplasty to be undertaken successfully the artery has to be accessible. The lesion relatively short and reasonable distal vessel runoff. Longer lesions may be amenable to sub-intimal angioplasty.
Surgery
Surgery will be undertaken where attempts at angioplasty have either failed or are unsuitable. Bypass essentially involves bypassing the affected arterial segment by utilising a graft to run from above the disease to below the disease. As with angioplasty good runoff improves the outcome.
Some key concepts with bypass surgery
Superficial femoral artery occlusion to the above knee popliteal
In the ideal scenario, vein (either in situ or reversed LSV) would the used as a conduit. However, prosthetic material has reasonable 5 year patency rates and some would advocate using this in preference to vein so that vein can be used for other procedures in the future. In general terms either technique is usually associated with an excellent outcome (if run off satisfactory).
Procedure
Artery dissected out, IV heparin 3,000 units given and then the vessels are cross clamped
Longitudinal arteriotomy
Graft cut to size and tunneled to arteriotomy sites
Anastomosis to femoral artery usually with 5/0 ‘double ended’ Prolene suture
Distal anastomosis usually using 6/0 ‘double ended’ Prolene
Distal disease
Femoro-distal bypass surgery takes longer to perform, is more technically challenging and has higher failure rates.
In elderly diabetic patients with poor runoff a primary amputation may well be a safer and more effective option. There is no point in embarking on this type of surgery in patients who are wheelchair bound.
In femorodistal bypasses vein gives superior outcomes to PTFE.
Rules
Vein mapping 1st to see whether there is suitable vein (the preferred conduit). Sub intimal hyperplasia occurs early when PTFE is used for the distal anastomosis and will lead to early graft occlusion and failure.
Essential operative procedure as for above knee fem-pop.
If there is insufficient vein for the entire conduit then vein can be attached to the end of the PTFE graft and then used for the distal anastomosis. This type of ‘vein boot’ is technically referred to as a Miller Cuff and is associated with better patency rates than PTFE alone.
Remember the more distal the arterial anastomosis the lower the success rate.
A 72 year old man present in the vascular clinic with calf pain present on walking 100 yards. He is an ex-smoker and lives alone. On examination he has reasonable leg pulses. His right dorsalis pedis pulse gives a monophasic doppler signal with an ankle brachial pressure index measurement of 0.7. All other pressures are acceptable. There is no evidence of ulceration or gangrene.
A. Primary amputation B. Angioplasty C. Arterial bypass surgery using vein D. Arterial bypass surgery using PTFE E. Conservative management with medical therapy and exercise F. Watch and wait G. Duplex scanning
Conservative management with medical therapy and exercise
Structured exercise programmes combined with medical therapy will improve many patients. Should his symptoms worsen or fail to improve then imaging with duplex scanning would be required.
Peripheral vascular disease
Indications for surgery to revascularise the lower limb Intermittent claudication Critical ischaemia Ulceration Gangrene
Intermittent claudication that is not disabling may provide a relative indication, whilst the other complaints are often absolute indications depending upon the frailty of the patient.
Assessment
Clinical examination
Ankle brachial pressure index measurement
Duplex arterial ultrasound
Angiography (standard, CT or MRI): usually performed only if intervention being considered.
Angioplasty
In order for angioplasty to be undertaken successfully the artery has to be accessible. The lesion relatively short and reasonable distal vessel runoff. Longer lesions may be amenable to sub-intimal angioplasty.
Surgery
Surgery will be undertaken where attempts at angioplasty have either failed or are unsuitable. Bypass essentially involves bypassing the affected arterial segment by utilising a graft to run from above the disease to below the disease. As with angioplasty good runoff improves the outcome.
Some key concepts with bypass surgery
Superficial femoral artery occlusion to the above knee popliteal
In the ideal scenario, vein (either in situ or reversed LSV) would the used as a conduit. However, prosthetic material has reasonable 5 year patency rates and some would advocate using this in preference to vein so that vein can be used for other procedures in the future. In general terms either technique is usually associated with an excellent outcome (if run off satisfactory).
Procedure
Artery dissected out, IV heparin 3,000 units given and then the vessels are cross clamped
Longitudinal arteriotomy
Graft cut to size and tunneled to arteriotomy sites
Anastomosis to femoral artery usually with 5/0 ‘double ended’ Prolene suture
Distal anastomosis usually using 6/0 ‘double ended’ Prolene
Distal disease
Femoro-distal bypass surgery takes longer to perform, is more technically challenging and has higher failure rates.
In elderly diabetic patients with poor runoff a primary amputation may well be a safer and more effective option. There is no point in embarking on this type of surgery in patients who are wheelchair bound.
In femorodistal bypasses vein gives superior outcomes to PTFE.
Rules
Vein mapping 1st to see whether there is suitable vein (the preferred conduit). Sub intimal hyperplasia occurs early when PTFE is used for the distal anastomosis and will lead to early graft occlusion and failure.
Essential operative procedure as for above knee fem-pop.
If there is insufficient vein for the entire conduit then vein can be attached to the end of the PTFE graft and then used for the distal anastomosis. This type of ‘vein boot’ is technically referred to as a Miller Cuff and is associated with better patency rates than PTFE alone.
Remember the more distal the arterial anastomosis the lower the success rate.
An 83 year old lady is admitted from a nursing home with infected lower leg ulcers. She underwent an attempted long superficial femoral artery sub initimal angioplasty 2 weeks previously. This demonstrated poor runoff below the knee.
A. Primary amputation B. Angioplasty C. Arterial bypass surgery using vein D. Arterial bypass surgery using PTFE E. Conservative management with medical therapy and exercise F. Watch and wait G. Duplex scanning
Primary amputation
Poor runoff and sepsis would equate to poor outcome with attempted bypass surgery.
Peripheral vascular disease
Indications for surgery to revascularise the lower limb Intermittent claudication Critical ischaemia Ulceration Gangrene
Intermittent claudication that is not disabling may provide a relative indication, whilst the other complaints are often absolute indications depending upon the frailty of the patient.
Assessment
Clinical examination
Ankle brachial pressure index measurement
Duplex arterial ultrasound
Angiography (standard, CT or MRI): usually performed only if intervention being considered.
Angioplasty
In order for angioplasty to be undertaken successfully the artery has to be accessible. The lesion relatively short and reasonable distal vessel runoff. Longer lesions may be amenable to sub-intimal angioplasty.
Surgery
Surgery will be undertaken where attempts at angioplasty have either failed or are unsuitable. Bypass essentially involves bypassing the affected arterial segment by utilising a graft to run from above the disease to below the disease. As with angioplasty good runoff improves the outcome.
Some key concepts with bypass surgery
Superficial femoral artery occlusion to the above knee popliteal
In the ideal scenario, vein (either in situ or reversed LSV) would the used as a conduit. However, prosthetic material has reasonable 5 year patency rates and some would advocate using this in preference to vein so that vein can be used for other procedures in the future. In general terms either technique is usually associated with an excellent outcome (if run off satisfactory).
Procedure
Artery dissected out, IV heparin 3,000 units given and then the vessels are cross clamped
Longitudinal arteriotomy
Graft cut to size and tunneled to arteriotomy sites
Anastomosis to femoral artery usually with 5/0 ‘double ended’ Prolene suture
Distal anastomosis usually using 6/0 ‘double ended’ Prolene
Distal disease
Femoro-distal bypass surgery takes longer to perform, is more technically challenging and has higher failure rates.
In elderly diabetic patients with poor runoff a primary amputation may well be a safer and more effective option. There is no point in embarking on this type of surgery in patients who are wheelchair bound.
In femorodistal bypasses vein gives superior outcomes to PTFE.
Rules
Vein mapping 1st to see whether there is suitable vein (the preferred conduit). Sub intimal hyperplasia occurs early when PTFE is used for the distal anastomosis and will lead to early graft occlusion and failure.
Essential operative procedure as for above knee fem-pop.
If there is insufficient vein for the entire conduit then vein can be attached to the end of the PTFE graft and then used for the distal anastomosis. This type of ‘vein boot’ is technically referred to as a Miller Cuff and is associated with better patency rates than PTFE alone.
Remember the more distal the arterial anastomosis the lower the success rate.
A 28 year old man falls on the back of his hand. On x-ray he has a fractured distal radius demonstrating volar displacement of the fracture.
A. Smith's B. Bennett's C. Monteggia's D. Colles' E. Galeazzi F. Pott's G. Barton's
Smith’s
This is a Smith fracture (reverse Colles’ fracture); unlike a Colles’ this is a high velocity injury and may require surgical correction. Note that Colles’ fractures are usually dorsally displaced.
Eponymous fractures
Colles’ fracture (dinner fork deformity)
Fall onto extended outstretched hand
Classical Colles’ fractures have the following 3 features:
- Transverse fracture of the radius
- 1 inch proximal to the radio-carpal joint
- Dorsal displacement and angulation
Smith’s fracture (reverse Colles’ fracture)
Volar angulation of distal radius fragment (Garden spade deformity)
Caused by falling backwards onto the palm of an outstretched hand or falling with wrists flexed
Bennett’s fracture
Intra-articular fracture of the first carpometacarpal joint
Impact on flexed metacarpal, caused by fist fights
X-ray: triangular fragment at ulnar base of metacarpal
Image sourced from Wikipedia
Monteggia’s fracture
Dislocation of the proximal radioulnar joint in association with an ulna fracture
Fall on outstretched hand with forced pronation
Needs prompt diagnosis to avoid disability
Image sourced from Wikipedia
Galeazzi fracture
Radial shaft fracture with associated dislocation of the distal radioulnar joint
Direct blow
Pott’s fracture
Bimalleolar ankle fracture
Forced foot eversion
Barton’s fracture
Distal radius fracture (Colles’/Smith’s) with associated radiocarpal dislocation
Fall onto extended and pronated wrist
Involvement of the joint is a defining feature
A 38 year old window cleaner falls from his ladder. He lands on his left arm and notices an obvious injury. An x-ray and clinical examination demonstrate that he has a fracture of the proximal ulna and associated radial dislocation.
A. Smith's B. Bennett's C. Monteggia's D. Colles' E. Galeazzi F. Pott's G. Barton's
Monteggia’s
This constellation of injuries is referred to as a Monteggia’s fracture.
Eponymous fractures
Colles’ fracture (dinner fork deformity)
Fall onto extended outstretched hand
Classical Colles’ fractures have the following 3 features:
- Transverse fracture of the radius
- 1 inch proximal to the radio-carpal joint
- Dorsal displacement and angulation
Smith’s fracture (reverse Colles’ fracture)
Volar angulation of distal radius fragment (Garden spade deformity)
Caused by falling backwards onto the palm of an outstretched hand or falling with wrists flexed
Bennett’s fracture
Intra-articular fracture of the first carpometacarpal joint
Impact on flexed metacarpal, caused by fist fights
X-ray: triangular fragment at ulnar base of metacarpal
Image sourced from Wikipedia
Monteggia’s fracture
Dislocation of the proximal radioulnar joint in association with an ulna fracture
Fall on outstretched hand with forced pronation
Needs prompt diagnosis to avoid disability
Image sourced from Wikipedia
Galeazzi fracture
Radial shaft fracture with associated dislocation of the distal radioulnar joint
Direct blow
Pott’s fracture
Bimalleolar ankle fracture
Forced foot eversion
Barton’s fracture
Distal radius fracture (Colles’/Smith’s) with associated radiocarpal dislocation
Fall onto extended and pronated wrist
Involvement of the joint is a defining feature
A 32 year old man falls from scaffolding and sustains an injury to his forearm. Clinical examination and x-ray shows that he has sustained a radial fracture with dislocation of the distal radio-ulna joint.
A. Smith's B. Bennett's C. Monteggia's D. Colles' E. Galeazzi F. Pott's G. Barton's
Galeazzi
Isolated fracture of the radius alone can occur but is rare. Always check for associated injury.
Eponymous fractures
Colles’ fracture (dinner fork deformity)
Fall onto extended outstretched hand
Classical Colles’ fractures have the following 3 features:
- Transverse fracture of the radius
- 1 inch proximal to the radio-carpal joint
- Dorsal displacement and angulation
Smith’s fracture (reverse Colles’ fracture)
Volar angulation of distal radius fragment (Garden spade deformity)
Caused by falling backwards onto the palm of an outstretched hand or falling with wrists flexed
Bennett’s fracture
Intra-articular fracture of the first carpometacarpal joint
Impact on flexed metacarpal, caused by fist fights
X-ray: triangular fragment at ulnar base of metacarpal
Image sourced from Wikipedia
Monteggia’s fracture
Dislocation of the proximal radioulnar joint in association with an ulna fracture
Fall on outstretched hand with forced pronation
Needs prompt diagnosis to avoid disability
Image sourced from Wikipedia
Galeazzi fracture
Radial shaft fracture with associated dislocation of the distal radioulnar joint
Direct blow
Pott’s fracture
Bimalleolar ankle fracture
Forced foot eversion
Barton’s fracture
Distal radius fracture (Colles’/Smith’s) with associated radiocarpal dislocation
Fall onto extended and pronated wrist
Involvement of the joint is a defining feature
A 62 year old woman presents with acute bowel obstruction. She has been vomiting up to 15 times a day and is taking erythromycin. She suddenly complains of dizziness. Her ECG shows torsades de pointes. What is the management of choice?
IV Atropine IV Potassium IV Magnesium sulphate IV Bicarbonate IV Adrenaline
Torsades de pointes: Treatment IV magnesium sulphate
This woman is likely to have hypokalaemia and hypomagnasaemia as a result of vomiting. In addition to this, the erythromycin will predispose her to torsades de pointes. The patient needs Magnesium 2g over 10 minutes. Knowledge of the management of this peri arrest diagnosis is hence important in surgical practice.
Torsades de pointes
Torsades de pointes (‘twisting of the points’) is a rare arrhythmia associated with a long QT interval. It may deteriorate into ventricular fibrillation and hence lead to sudden death
Causes of long QT interval congenital: Jervell-Lange-Nielsen syndrome, Romano-Ward syndrome antiarrhythmics: amiodarone, sotalol, class 1a antiarrhythmic drugs tricyclic antidepressants antipsychotics chloroquine terfenadine erythromycin electrolyte: hypocalcaemia, hypokalaemia, hypomagnesaemia myocarditis hypothermia subarachnoid haemorrhage
Management
IV magnesium sulphate
A 27 year old man sustains a single gunshot wound to the left thigh. In the emergency department, he is noted to have a large haematoma of his medial thigh. He complains of parasthesia in his foot. On examination, there are weak pulses palpable distal to the injury and the patient is unable to move his foot. The appropriate initial management of this patient is:
Conventional angiography Immediate exploration and repair Fasciotomy of the anterior compartment Observation for resolution of spasm Local wound exploration
Immediate exploration and repair
The five P’s of arterial injury include pain, parasthesias, pallor, pulselessness and paralysis. In the extremities, the tissues most sensitive to anoxia are the peripheral nerves and striated muscle. The early developments of paresthesias and paralysis are signals that there is significant ischemia present, and immediate exploration and repair are warranted. The presence of palpable pulse does not exclude an arterial injury because this presence may represent a transmitted pulsation through a blood clot. When severe ischemia is present, the repair must be completed within 6 to 8 h to prevent irreversible muscle ischemia and loss of limb function. Delay to obtain a conventional angiogram or to observe for change needlessly prolongs the ischemic time. A CT angiogram may be a reasonable alternative. Fasciotomy may be required but should be done in conjunction with and after re-establishment of arterial flow. Local wound exploration is not recommended because brisk hemorrhage may be encountered without the securing of prior vascular control.
A 54-year-old man presents to the Emergency Department with a 2 day history of a swollen, painful left knee. You aspirate the joint to avoid admission to the orthopaedic wards. Aspirated joint fluid shows calcium pyrophosphate crystals. Which of the following blood tests is most useful in revealing an underlying cause?
Transferrin saturation ACTH ANA Serum ferritin LDH
This is a typical presentation of pseudogout. An elevated transferrin saturation may indicate haemochromatosis, a recognised cause of pseudogout.
A high ferritin level is also seen in haemochromatosis but can be raised in a variety of infective and inflammatory processes, including pseudogout, as part of an acute phase response.
Pseudogout
Pseudogout is a form of microcrystal synovitis caused by the deposition of calcium pyrophosphate dihydrate in the synovium
Risk factors hyperparathyroidism hypothyroidism haemochromatosis acromegaly low magnesium, low phosphate Wilson's disease
Features
knee, wrist and shoulders most commonly affected
joint aspiration: weakly-positively birefringent rhomboid shaped crystals
x-ray: chondrocalcinosis
Management
aspiration of joint fluid, to exclude septic arthritis
NSAIDs or intra-articular, intra-muscular or oral steroids as for gout
A 3 year old child inserts a crayon into their external auditory meatus. Attempts to remove it have not been successful.
A. Manage conservatively B. Immediate emergency theatre C. Treat in emergency department D. Treat in emergency department under sedation E. Operate on next emergency list
Operate on next emergency list
They would not tolerate removal in the emergency department. The tympanic membrane should be carefully inspected and again this will be easier under general anaesthesia.
Management of acute cases- Paediatric
Children will often insert objects into orifices such as the nose and external auditory meatus
Assessment includes assessment of airway and haemodynamic status
Where the airway is not immediately threatened decisions can be made as to whether to manage in the emergency department or transfer to theatre
In general children do not tolerate procedures well and it is usually safer to remove objects in theatre and under general anaesthesia with a secure airway
A chest x-ray is required to ensure that no object is present in the chest, not all objects are radioopaque. However, signs such as focal consolidation may indicate small airway obstruction
In the case of small bore missile injuries the decision relating to surgery depends on the size of the missile and its location. Airgun pellets are a common culprit, if there is a long time interval between the incident and presentation and the object has not caused any significant problems then it may be best left alone
Airgun pellets (and glass) lodged in the soft tissues are usually notoriously difficult to localise and extract, no matter how superficial. Removal in theatre is usually the best option. If the object is radioopaque then an image intensifier should be used
A 2 year old accidentally inhales a peanut. They arrive in the emergency department extremely distressed and cyanotic. Imaging shows it to be lodged in the left main bronchus.
A. Manage conservatively B. Immediate emergency theatre C. Treat in emergency department D. Treat in emergency department under sedation E. Operate on next emergency list
Immediate emergency theatre
As they are cyanosed it requires immediate removal and this should be undertaken in a fully staffed theatre. Ideally a rigid bronchoscopy should be performed.
Management of acute cases- Paediatric
Children will often insert objects into orifices such as the nose and external auditory meatus
Assessment includes assessment of airway and haemodynamic status
Where the airway is not immediately threatened decisions can be made as to whether to manage in the emergency department or transfer to theatre
In general children do not tolerate procedures well and it is usually safer to remove objects in theatre and under general anaesthesia with a secure airway
A chest x-ray is required to ensure that no object is present in the chest, not all objects are radioopaque. However, signs such as focal consolidation may indicate small airway obstruction
In the case of small bore missile injuries the decision relating to surgery depends on the size of the missile and its location. Airgun pellets are a common culprit, if there is a long time interval between the incident and presentation and the object has not caused any significant problems then it may be best left alone
Airgun pellets (and glass) lodged in the soft tissues are usually notoriously difficult to localise and extract, no matter how superficial. Removal in theatre is usually the best option. If the object is radioopaque then an image intensifier should be used
A 10 year old boy is shot in the head with an airgun pellet. He is concerned that he will get into trouble and the injury remains concealed for 10 days. Imaging using CT scanning shows it to be lodged in the frontal lobe.
A. Manage conservatively B. Immediate emergency theatre C. Treat in emergency department D. Treat in emergency department under sedation E. Operate on next emergency list
Manage conservatively
The pellet is small and no serious injury has occurred at this stage. This should therefore be managed conservatively.
Management of acute cases- Paediatric
Children will often insert objects into orifices such as the nose and external auditory meatus
Assessment includes assessment of airway and haemodynamic status
Where the airway is not immediately threatened decisions can be made as to whether to manage in the emergency department or transfer to theatre
In general children do not tolerate procedures well and it is usually safer to remove objects in theatre and under general anaesthesia with a secure airway
A chest x-ray is required to ensure that no object is present in the chest, not all objects are radioopaque. However, signs such as focal consolidation may indicate small airway obstruction
In the case of small bore missile injuries the decision relating to surgery depends on the size of the missile and its location. Airgun pellets are a common culprit, if there is a long time interval between the incident and presentation and the object has not caused any significant problems then it may be best left alone
Airgun pellets (and glass) lodged in the soft tissues are usually notoriously difficult to localise and extract, no matter how superficial. Removal in theatre is usually the best option. If the object is radioopaque then an image intensifier should be used
A 66 year old man is referred via the aneurysm screening programme with an abdominal aortic aneurysm measuring 4.4 cm. Apart from well controlled type 2 DM he is otherwise well
A. Immediate laparotomy B. Immediate CT C. AAA repair during next 48 hours D. USS in 6 months E. CT scan during next 4 weeks F. Endovascular aortic aneurysm repair G. Discharge H. Palliate I. None of the above
USS in 6 months
At this point continue with ultrasound surveillance
Abdominal aorta aneurysm
Abdominal aortic aneurysms are a common problem in vascular surgery.
They may occur as either true or false aneurysm. With the former all 3 layers of the arterial wall are involved, in the latter only a single layer of fibrous tissue forms the aneurysm wall.
True abdominal aortic aneurysms have an approximate incidence of 0.06 per 1000 people. They are commonest in elderly men and for this reason the UK is now introducing the aneurysm screening program with the aim of performing an abdominal aortic ultrasound measurement in all men aged 65 years.
Causes
Several different groups of patients suffer from aneurysmal disease.
The commonest group is those who suffer from standard arterial disease, i.e. Those who are hypertensive and have been or are smokers.
Other patients such as those suffering from connective tissue diseases such as Marfan’s may also develop aneurysms. In patients with abdominal aortic aneurysms the extracellular matrix becomes disrupted with a change in the balance of collagen and elastic fibres.
Management
Most abdominal aortic aneurysms are an incidental finding.
Symptoms most often relate to rupture or impending rupture.
20% rupture anteriorly into the peritoneal cavity. Very poor prognosis.
80% rupture posteriorly into the retroperitoneal space
The risk of rupture is related to aneurysm size, only 2% of aneurysms measuring less than 4cm in diameter will rupture over a 5 year period. This contrasts with 75% of aneurysms measuring over 7cm in diameter.
This is well explained by Laplaces’ law which relates size to transmural pressure.
For this reason most vascular surgeons will subject patients with an aneurysm size of 5cm or greater to CT scanning of the chest, abdomen and pelvis with the aim of delineating anatomy and planning treatment. Depending upon co-morbidities, surgery is generally offered once the aneurysm is between 5.5cm and 6cm.
A CT reconstruction showing an infrarenal abdominal aortic aneurysm. The walls of the sac are calcified which may facilitate identification on plain x-rays
Image sourced from Wikipedia
Indications for surgery
Symptomatic aneurysms (80% annual mortality if untreated)
Increasing size above 5.5cm if asymptomatic
Rupture (100% mortality without surgery)
Surgical procedures
Abdominal aortic aneurysm repair
Procedure:
GA
Invasive monitoring (A-line, CVP, catheter)
Incision: Midline or transverse
Bowel and distal duodenum mobilised to access aorta.
Aneurysm neck and base dissected out and prepared for cross clamp
Systemic heparinisation
Cross clamp (proximal first)
Longitudinal aortotomy
Atherectomy
Deal with back bleeding from lumbar vessels and inferior mesenteric artery
Insert graft either tube or bifurcated depending upon anatomy
Suture using Prolene (3/0 for proximal , distal anastomosis suture varies according to site)
Clamps off: End tidal CO2 will rise owing to effects of reperfusion, at this point major risk of myocardial events.
Haemostasis
Closure of aneurysm sac to minimise risk of aorto-enteric fistula
Closure: Loop 1 PDS or Prolene to abdominal wall
Skin- surgeons preference
Post operatively:
ITU (Almost all)
Greatest risk of complications following emergency repair
Complications: Embolic- gut and foot infarcts
Cardiac - owing to premorbid states, re-perfusion injury and effects of cross clamp
Wound problems
Later risks related to graft- infection and aorto-enteric fistula
Special groups
Supra renal AAA
These patients will require a supra renal clamp and this carries a far higher risk of complications and risk of renal failure.
Ruptured AAA
Pre-operatively the management depends upon haemodynamic instability. In patients with symptoms of rupture (typical pain, haemodynamic compromise and risk factors) then ideally prompt laparotomy. In those with vague symptoms and haemodynamic stability the ideal test is CT scan to determine whether rupture has occurred or not. Most common rupture site is retroperitoneal 80%. These patients will tend to develop retroperitoneal haematoma. This can be disrupted if Bp is allowed to rise too high so aim for Bp 100mmHg.
Operative details are similar to elective repair although surgery should be swift, blind rushing often makes the situation worse. Plunging vascular clamps blindly into a pool of blood at the aneurysm neck carries the risk of injury the vena cava that these patients do not withstand. Occasionally a supracoeliac clamp is needed to effect temporary control, although leaving this applied for more than 20 minutes tends to carry a dismal outcome.
EVAR Increasingly patients are now being offered endovascular aortic aneurysm repair. This is undertaken by surgeons and radiologists working jointly. The morphology of the aneurysm is important and not all are suitable. Here is a typical list of those features favoring a suitable aneurysm: Long neck Straight iliac vessels Healthy groin vessels
Clearly few AAA patients possess the above and compromise has to be made. The use of fenestrated grafts can allow supra renal AAA to be treated.
Procedure:
GA Radiology or theatre Bilateral groin incisions Common femoral artery dissected out Heparinisation Arteriotomy and insertion of guide wire Dilation of arteriotomy Insertion of EVAR Device Once in satisfactory position it is released Arteriotomy closed once check angiogram shows good position and no endoleak
Complications:
Endoleaks depending upon site are either Type I or 2. These may necessitate re-intervention and all EVAR patients require follow up . Details are not needed for MRCS.
A 72 year old man has a CT scan for abdominal discomfort and the surgeon suspects AAA. This shows a 6.6cm aneurysm with a 3.5cm neck and it continues to involve the right common iliac. The left iliac is occluded. He is hypertensive and has Type 2 DM which is well controlled.
A. Immediate laparotomy B. Immediate CT C. AAA repair during next 48 hours D. USS in 6 months E. CT scan during next 4 weeks F. Endovascular aortic aneurysm repair G. Discharge H. Palliate I. None of the above
AAA repair during next 48 hours
Assuming he is fit enough. This would be a typical ‘open ‘ case as the marked iliac disease would make EVAR difficult
Abdominal aorta aneurysm
Abdominal aortic aneurysms are a common problem in vascular surgery.
They may occur as either true or false aneurysm. With the former all 3 layers of the arterial wall are involved, in the latter only a single layer of fibrous tissue forms the aneurysm wall.
True abdominal aortic aneurysms have an approximate incidence of 0.06 per 1000 people. They are commonest in elderly men and for this reason the UK is now introducing the aneurysm screening program with the aim of performing an abdominal aortic ultrasound measurement in all men aged 65 years.
Causes
Several different groups of patients suffer from aneurysmal disease.
The commonest group is those who suffer from standard arterial disease, i.e. Those who are hypertensive and have been or are smokers.
Other patients such as those suffering from connective tissue diseases such as Marfan’s may also develop aneurysms. In patients with abdominal aortic aneurysms the extracellular matrix becomes disrupted with a change in the balance of collagen and elastic fibres.
Management
Most abdominal aortic aneurysms are an incidental finding.
Symptoms most often relate to rupture or impending rupture.
20% rupture anteriorly into the peritoneal cavity. Very poor prognosis.
80% rupture posteriorly into the retroperitoneal space
The risk of rupture is related to aneurysm size, only 2% of aneurysms measuring less than 4cm in diameter will rupture over a 5 year period. This contrasts with 75% of aneurysms measuring over 7cm in diameter.
This is well explained by Laplaces’ law which relates size to transmural pressure.
For this reason most vascular surgeons will subject patients with an aneurysm size of 5cm or greater to CT scanning of the chest, abdomen and pelvis with the aim of delineating anatomy and planning treatment. Depending upon co-morbidities, surgery is generally offered once the aneurysm is between 5.5cm and 6cm.
A CT reconstruction showing an infrarenal abdominal aortic aneurysm. The walls of the sac are calcified which may facilitate identification on plain x-rays
Image sourced from Wikipedia
Indications for surgery
Symptomatic aneurysms (80% annual mortality if untreated)
Increasing size above 5.5cm if asymptomatic
Rupture (100% mortality without surgery)
Surgical procedures
Abdominal aortic aneurysm repair
Procedure:
GA
Invasive monitoring (A-line, CVP, catheter)
Incision: Midline or transverse
Bowel and distal duodenum mobilised to access aorta.
Aneurysm neck and base dissected out and prepared for cross clamp
Systemic heparinisation
Cross clamp (proximal first)
Longitudinal aortotomy
Atherectomy
Deal with back bleeding from lumbar vessels and inferior mesenteric artery
Insert graft either tube or bifurcated depending upon anatomy
Suture using Prolene (3/0 for proximal , distal anastomosis suture varies according to site)
Clamps off: End tidal CO2 will rise owing to effects of reperfusion, at this point major risk of myocardial events.
Haemostasis
Closure of aneurysm sac to minimise risk of aorto-enteric fistula
Closure: Loop 1 PDS or Prolene to abdominal wall
Skin- surgeons preference
Post operatively:
ITU (Almost all)
Greatest risk of complications following emergency repair
Complications: Embolic- gut and foot infarcts
Cardiac - owing to premorbid states, re-perfusion injury and effects of cross clamp
Wound problems
Later risks related to graft- infection and aorto-enteric fistula
Special groups
Supra renal AAA
These patients will require a supra renal clamp and this carries a far higher risk of complications and risk of renal failure.
Ruptured AAA
Pre-operatively the management depends upon haemodynamic instability. In patients with symptoms of rupture (typical pain, haemodynamic compromise and risk factors) then ideally prompt laparotomy. In those with vague symptoms and haemodynamic stability the ideal test is CT scan to determine whether rupture has occurred or not. Most common rupture site is retroperitoneal 80%. These patients will tend to develop retroperitoneal haematoma. This can be disrupted if Bp is allowed to rise too high so aim for Bp 100mmHg.
Operative details are similar to elective repair although surgery should be swift, blind rushing often makes the situation worse. Plunging vascular clamps blindly into a pool of blood at the aneurysm neck carries the risk of injury the vena cava that these patients do not withstand. Occasionally a supracoeliac clamp is needed to effect temporary control, although leaving this applied for more than 20 minutes tends to carry a dismal outcome.
EVAR Increasingly patients are now being offered endovascular aortic aneurysm repair. This is undertaken by surgeons and radiologists working jointly. The morphology of the aneurysm is important and not all are suitable. Here is a typical list of those features favoring a suitable aneurysm: Long neck Straight iliac vessels Healthy groin vessels
Clearly few AAA patients possess the above and compromise has to be made. The use of fenestrated grafts can allow supra renal AAA to be treated.
Procedure:
GA Radiology or theatre Bilateral groin incisions Common femoral artery dissected out Heparinisation Arteriotomy and insertion of guide wire Dilation of arteriotomy Insertion of EVAR Device Once in satisfactory position it is released Arteriotomy closed once check angiogram shows good position and no endoleak
Complications:
Endoleaks depending upon site are either Type I or 2. These may necessitate re-intervention and all EVAR patients require follow up . Details are not needed for MRCS.
An 89 year old man presents with hypotension and collapse and is found by the staff in the toilet of his care home. He is moribund and unable to give a clear history. He had suffered a cardiac arrest in the ambulance but has since been resuscitated and now has a Bp of 95 systolic. He has an obviously palpable AAA.
A. Immediate laparotomy B. Immediate CT C. AAA repair during next 48 hours D. USS in 6 months E. CT scan during next 4 weeks F. Endovascular aortic aneurysm repair G. Discharge H. Palliate I. None of the above
Palliate
He will not survive aortic surgery and whilst some may disagree, I would argue that taking this case to theatre would be futile
Abdominal aorta aneurysm
Abdominal aortic aneurysms are a common problem in vascular surgery.
They may occur as either true or false aneurysm. With the former all 3 layers of the arterial wall are involved, in the latter only a single layer of fibrous tissue forms the aneurysm wall.
True abdominal aortic aneurysms have an approximate incidence of 0.06 per 1000 people. They are commonest in elderly men and for this reason the UK is now introducing the aneurysm screening program with the aim of performing an abdominal aortic ultrasound measurement in all men aged 65 years.
Causes
Several different groups of patients suffer from aneurysmal disease.
The commonest group is those who suffer from standard arterial disease, i.e. Those who are hypertensive and have been or are smokers.
Other patients such as those suffering from connective tissue diseases such as Marfan’s may also develop aneurysms. In patients with abdominal aortic aneurysms the extracellular matrix becomes disrupted with a change in the balance of collagen and elastic fibres.
Management
Most abdominal aortic aneurysms are an incidental finding.
Symptoms most often relate to rupture or impending rupture.
20% rupture anteriorly into the peritoneal cavity. Very poor prognosis.
80% rupture posteriorly into the retroperitoneal space
The risk of rupture is related to aneurysm size, only 2% of aneurysms measuring less than 4cm in diameter will rupture over a 5 year period. This contrasts with 75% of aneurysms measuring over 7cm in diameter.
This is well explained by Laplaces’ law which relates size to transmural pressure.
For this reason most vascular surgeons will subject patients with an aneurysm size of 5cm or greater to CT scanning of the chest, abdomen and pelvis with the aim of delineating anatomy and planning treatment. Depending upon co-morbidities, surgery is generally offered once the aneurysm is between 5.5cm and 6cm.
A CT reconstruction showing an infrarenal abdominal aortic aneurysm. The walls of the sac are calcified which may facilitate identification on plain x-rays
Image sourced from Wikipedia
Indications for surgery
Symptomatic aneurysms (80% annual mortality if untreated)
Increasing size above 5.5cm if asymptomatic
Rupture (100% mortality without surgery)
Surgical procedures
Abdominal aortic aneurysm repair
Procedure:
GA
Invasive monitoring (A-line, CVP, catheter)
Incision: Midline or transverse
Bowel and distal duodenum mobilised to access aorta.
Aneurysm neck and base dissected out and prepared for cross clamp
Systemic heparinisation
Cross clamp (proximal first)
Longitudinal aortotomy
Atherectomy
Deal with back bleeding from lumbar vessels and inferior mesenteric artery
Insert graft either tube or bifurcated depending upon anatomy
Suture using Prolene (3/0 for proximal , distal anastomosis suture varies according to site)
Clamps off: End tidal CO2 will rise owing to effects of reperfusion, at this point major risk of myocardial events.
Haemostasis
Closure of aneurysm sac to minimise risk of aorto-enteric fistula
Closure: Loop 1 PDS or Prolene to abdominal wall
Skin- surgeons preference
Post operatively:
ITU (Almost all)
Greatest risk of complications following emergency repair
Complications: Embolic- gut and foot infarcts
Cardiac - owing to premorbid states, re-perfusion injury and effects of cross clamp
Wound problems
Later risks related to graft- infection and aorto-enteric fistula
Special groups
Supra renal AAA
These patients will require a supra renal clamp and this carries a far higher risk of complications and risk of renal failure.
Ruptured AAA
Pre-operatively the management depends upon haemodynamic instability. In patients with symptoms of rupture (typical pain, haemodynamic compromise and risk factors) then ideally prompt laparotomy. In those with vague symptoms and haemodynamic stability the ideal test is CT scan to determine whether rupture has occurred or not. Most common rupture site is retroperitoneal 80%. These patients will tend to develop retroperitoneal haematoma. This can be disrupted if Bp is allowed to rise too high so aim for Bp 100mmHg.
Operative details are similar to elective repair although surgery should be swift, blind rushing often makes the situation worse. Plunging vascular clamps blindly into a pool of blood at the aneurysm neck carries the risk of injury the vena cava that these patients do not withstand. Occasionally a supracoeliac clamp is needed to effect temporary control, although leaving this applied for more than 20 minutes tends to carry a dismal outcome.
EVAR Increasingly patients are now being offered endovascular aortic aneurysm repair. This is undertaken by surgeons and radiologists working jointly. The morphology of the aneurysm is important and not all are suitable. Here is a typical list of those features favoring a suitable aneurysm: Long neck Straight iliac vessels Healthy groin vessels
Clearly few AAA patients possess the above and compromise has to be made. The use of fenestrated grafts can allow supra renal AAA to be treated.
Procedure:
GA Radiology or theatre Bilateral groin incisions Common femoral artery dissected out Heparinisation Arteriotomy and insertion of guide wire Dilation of arteriotomy Insertion of EVAR Device Once in satisfactory position it is released Arteriotomy closed once check angiogram shows good position and no endoleak
Complications:
Endoleaks depending upon site are either Type I or 2. These may necessitate re-intervention and all EVAR patients require follow up . Details are not needed for MRCS.
A 19 year old soldier has just returned from a prolonged marching exercise and presents with a sudden onset, severe pain, in the forefoot. Clinical examination reveals tenderness along the second metatarsal. Plain x-rays are taken of the area, these demonstrate callus surrounding the shaft of the second metatarsal. What is the most likely diagnosis?
Stress fracture Mortons neuroma Osteochondroma Acute osteomyelitis Freiberg's disease
Stress fractures
Repetitive activity and loading of normal bone may result in small hairline fractures. Whilst these may be painful they are seldom displaced. Surrounding soft tissue injury is unusual. They may present late following the injury, in which case callus formation may be identified on radiographs. Such cases may not require formal immobilisation, injuries associated with severe pain and presenting at an earlier stage may benefit from immobilisation tailored to the site of injury.
A short history of pain together with clinical examination and radiological signs affecting the second metatarsal favour a stress fracture. The fact that callus is present suggests that immobilisation is unlikely to be beneficial. Freibergs disease is an anterior metatarsalgia affecting the head of the second metarsal, it typically occurs in the pubertal growth spurt. The initial injury was thought to be due to stress microfractures at the growth plate. The key feature in the history which distinguishes the injury as being stress fracture is the radiology. In Freibergs disease the x-ray changes include; joint space widening, formation of bony spurs, sclerosis and flattening of the metatarsal head.
A 33 year old female is admitted for varicose vein surgery. She is fit and well. After the procedure she is persistently bleeding. She is known to have menorrhagia. Investigations show a prolonged bleeding time and increased APTT. She has a normal PT and platelet count.
A. Vitamin K deficiency B. von Willebrand's disease C. Acquired haemophilia D. Haemophilia B E. Protein C deficiency F. Disseminated intravascular coagulation G. Factor V Leiden H. Excess heparin I. Warfarin overdose J. Antiphospholipid syndrome
von Willebrand’s disease
Bleeding post operatively, epistaxis and menorrhagia may indicate a diagnosis of vWD. Haemoarthroses are rare. The bleeding time is usually normal in haemophilia (X-linked) and vitamin K deficiency.
A 70 year old heavy smoker presents with 3 weeks of haematuria and bruising. He is normally fit and well. He is on no medications. His results reveal: Hb 9.0 WCC 11 Pl 255 PT 16 (normal) APTT 58 (increased) Thrombin time 20 (normal).
A. Vitamin K deficiency B. von Willebrand's disease C. Acquired haemophilia D. Haemophilia B E. Protein C deficiency F. Disseminated intravascular coagulation G. Factor V Leiden H. Excess heparin I. Warfarin overdose J. Antiphospholipid syndrome
Acquired haemophilia
This patient has Factor 8 acquired disorder. He is likely to have developed a lung malignancy (smoker) and as a result aquired a haemophilia disorder. The elderly, pregnancy, malignancy and autoimmune conditions are associated with acquired haemophilia. Prolonged APTT is key to the diagnosis. Management involves steroids.
A 28 year old female is attends the gynaecology unit for a D+C following an incomplete miscarriage. She has previously had recurrent pulmonary embolic events. After the procedure she is persistently bleeding. Her APTT is 52 (increased).
A. Vitamin K deficiency B. von Willebrand's disease C. Acquired haemophilia D. Haemophilia B E. Protein C deficiency F. Disseminated intravascular coagulation G. Factor V Leiden H. Excess heparin I. Warfarin overdose J. Antiphospholipid syndrome
Antiphospholipid syndrome
A combination of thromboembolism and bleeding in a young woman should raise the possibility of antiphospholipid syndrome. Other features may include foetal loss, venous and arterial thrombosis and thrombocytopenia. A Lupus anticoagulant may be present and the APTT is prolonged.
During short saphenous vein surgery for varicose veins which of the following nerves is particularly at risk?
Sural nerve Popliteal nerve Tibial nerve Femoral nerve Saphenous nerve
Sural nerve
Saphenous vein
Long saphenous vein
This vein may be harvested for bypass surgery, or removed as treatment for varicose veins with saphenofemoral junction incompetence.
Originates at the 1st digit where the dorsal vein merges with the dorsal venous arch of the foot
Passes anterior to the medial malleolus and runs up the medial side of the leg
At the knee, it runs over the posterior border of the medial epicondyle of the femur bone
Then passes laterally to lie on the anterior surface of the thigh before entering an opening in the fascia lata called the saphenous opening
It joins with the femoral vein in the region of the femoral triangle at the saphenofemoral junction
Tributaries Medial marginal Superficial epigastric Superficial iliac circumflex Superficial external pudendal veins
Short saphenous vein
Originates at the 5th digit where the dorsal vein merges with the dorsal venous arch of the foot, which attaches to the great saphenous vein.
It passes around the lateral aspect of the foot (inferior and posterior to the lateral malleolus) and runs along the posterior aspect of the leg (with the sural nerve)
Passes between the heads of the gastrocnemius muscle, and drains into the popliteal vein, approximately at or above the level of the knee joint.
A 78-year-old woman is discharged following a fractured neck of femur. On review, she is making good progress but consideration is given to secondary prevention of further fractures. Unfortunately the orthogeriatricians are all on annual leave and the consultant has asked you to arrange suitable management. Which is the best option?
Alendronate Alendronate, calcium and vitamin D supplementation Strontium Arrange a DEXA scan Hormone replacement therapy
A bisphosphonate, calcium and vitamin D supplementation should be given to all patients aged over 75 years after having a fracture. A DEXA scan is only needed of the patient is aged below 75 years. Hormone replacement therapy has been shown to reduce vertebral and non vertebral fractures, however the risks of cardiovascular disease and breast malignancy make this a less favourable option.
Osteoporosis: secondary prevention
NICE guidelines were updated in 2008 on the secondary prevention of osteoporotic fractures in postmenopausal women.
Key points include
Treatment is indicated following osteoporotic fragility fractures in postmenopausal women who are confirmed to have osteoporosis (a T-score of - 2.5 SD or below).
In women aged 75 years or older, a DEXA scan may not be required ‘if the responsible clinician considers it to be clinically inappropriate or unfeasible’
Vitamin D and calcium supplementation should be offered to all women unless the clinician is confident they have adequate calcium intake and are vitamin D replete
Alendronate is first-line
Around 25% of patients cannot tolerate alendronate, usually due to upper gastrointestinal problems. These patients should be offered risedronate or etidronate (see treatment criteria below)
Strontium ranelate and raloxifene are recommended if patients cannot tolerate bisphosphonates (see treatment criteria below)
Supplementary notes on treatment
Bisphosphonates
Alendronate, risedronate and etidronate are all licensed for the prevention and treatment of post-menopausal and glucocorticoid-induced osteoporosis
All three have been shown to reduce the risk of both vertebral and non-vertebral fractures although alendronate, risedronate may be superior to etidronate in preventing hip fractures
Ibandronate is a once-monthly oral bisphosphonate
Vitamin D and calcium
Poor evidence base to suggest reduced fracture rates in the general population at risk of osteoporotic fractures - may reduce rates in frail, housebound patients
Raloxifene - selective oestrogen receptor modulator (SERM)
Has been shown to prevent bone loss and to reduce the risk of vertebral fractures, but has not yet been shown to reduce the risk of non-vertebral fractures
Has been shown to increase bone density in the spine and proximal femur
May worsen menopausal symptoms
Increased risk of thromboembolic events
May decrease risk of breast cancer
Strontium ranelate
‘Dual action bone agent’ - increases deposition of new bone by osteoblasts and reduces the resorption of bone by osteoclasts
Strong evidence base, may be second-line treatment in near future
Increased risk of thromboembolic events
Which of the following statements relating to avascular necrosis is false?
When associated with fracture may occur despite the radiological evidence of fracture union. Pain and stiffness will typically precede radiological evidence of the condition. Drilling of affected bony fragments may be used to facilitate angiogenesis where arthroplasty is not warranted. The earliest detectable radiological evidence is a radiolucency of the affected area coupled with subchondral collapse. It is less likely when prompt anatomical alignment of fracture fragments is achieved.
The earliest detectable radiological evidence is a radiolucency of the affected area coupled with subchondral collapse.
Radiolucency and subchondral collapse are late changes. The earliest evidence on plain films is the affected area appearing as being more radio-opaque due to hyperaemia and resorption of the neighboring area. It may be diagnosed earlier using bone scans and MRI.
Avascular necrosis
Cellular death of bone components due to interruption of the blood supply, causing bone destruction
Main joints affected are hip, scaphoid, lunate and the talus.
It is not the same as non union. The fracture has usually united.
Radiological evidence is slow to appear.
Vascular ingrowth into the affected bone may occur. However, many joints will develop secondary osteoarthritis.
Causes P ancreatitis L upus A lcohol S teroids T rauma I diopathic, infection C aisson disease, collagen vascular disease R adiation, rheumatoid arthritis A myloid G aucher disease S ickle cell disease
Presentation
Usually pain. Often despite apparent fracture union.
Investigation
MRI scanning will show changes earlier than plain films.
Treatment
In fractures at high risk sites anticipation is key. Early prompt and accurate reduction is essential.
Non weight bearing may help to facilitate vascular regeneration.
Joint replacement may be necessary, or even the preferred option (e.g. Hip in the elderly).
A 22 year old rugby player falls onto an outstretched hand and sustains a fracture of the distal radius. The x-ray shows a dorsally angulated comminuted fracture.
A. Discharge home with arm sling and fracture clinic appointment
B. Discharge home with futura splint and fracture clinic appointment
C. Admit for open reduction and fixation
D. Fasciotomy
E. Active observation for progression of neurovascular compromise
F. Reduction of fracture in casualty and application of plaster backslab, followed by discharge home.
Admit for open reduction and fixation
Unlike an osteoporotic fracture in an elderly lady this is a high velocity injury and will require surgical fixation.
Fracture management
Bony injury resulting in a fracture may arise from trauma (excessive forces applied to bone), stress related (repetitive low velocity injury) or pathological (abnormal bone which fractures during normal use of following minimal trauma)
Diagnosis involves not just evaluating the fracture ; such as site and type of injury but also other associated injuries and distal neurovascular deficits. This may entail not just clinical examination but radiographs of proximal and distal joints.
When assessing x-rays it is important to assess for changes in length of the bone, the angulation of the distal bone, rotational effects, presence of material such as glass.
Key points in management of fractures
Immobilise the fracture including the proximal and distal joints
Carefully monitor and document neurovascular status, particularly following reduction and immobilisation
Manage infection including tetanus prophylaxis
IV broad spectrum antibiotics for open injuries
As a general principle all open fractures should be thoroughly debrided ( and internal fixation devices avoided or used with extreme caution)
Open fractures constitute an emergency and should be debrided and lavaged within 6 hours of injury
A 10 year old boy undergoes a delayed open reduction and fixation of a significantly displaced supracondylar fracture. On the ward he complains of significant forearm pain and paraesthesia of the hand. Radial pulse is normal.
A. Discharge home with arm sling and fracture clinic appointment
B. Discharge home with futura splint and fracture clinic appointment
C. Admit for open reduction and fixation
D. Fasciotomy
E. Active observation for progression of neurovascular compromise
F. Reduction of fracture in casualty and application of plaster backslab, followed by discharge home.
Fasciotomy
The delay is the significant factor here. These injuries often have neurovascular compromise and inactivity now places him at risk of developing complications. In compartment syndrome the loss of arterial pulsation occurs late.
Fracture management
Bony injury resulting in a fracture may arise from trauma (excessive forces applied to bone), stress related (repetitive low velocity injury) or pathological (abnormal bone which fractures during normal use of following minimal trauma)
Diagnosis involves not just evaluating the fracture ; such as site and type of injury but also other associated injuries and distal neurovascular deficits. This may entail not just clinical examination but radiographs of proximal and distal joints.
When assessing x-rays it is important to assess for changes in length of the bone, the angulation of the distal bone, rotational effects, presence of material such as glass.
Key points in management of fractures
Immobilise the fracture including the proximal and distal joints
Carefully monitor and document neurovascular status, particularly following reduction and immobilisation
Manage infection including tetanus prophylaxis
IV broad spectrum antibiotics for open injuries
As a general principle all open fractures should be thoroughly debrided ( and internal fixation devices avoided or used with extreme caution)
Open fractures constitute an emergency and should be debrided and lavaged within 6 hours of injury
A 28 year old man falls onto an outstretched hand. On examination there is tenderness of the anatomical snuffbox. However, forearm and hand x-rays are normal.
A. Discharge home with arm sling and fracture clinic appointment
B. Discharge home with futura splint and fracture clinic appointment
C. Admit for open reduction and fixation
D. Fasciotomy
E. Active observation for progression of neurovascular compromise
F. Reduction of fracture in casualty and application of plaster backslab, followed by discharge home.
Discharge home with futura splint and fracture clinic appointment
This could well be a scaphoid fracture and should be temporarily immobilised pending further review. A futura splint will immobilise better than an arm sling for this problem.
Fracture management
Bony injury resulting in a fracture may arise from trauma (excessive forces applied to bone), stress related (repetitive low velocity injury) or pathological (abnormal bone which fractures during normal use of following minimal trauma)
Diagnosis involves not just evaluating the fracture ; such as site and type of injury but also other associated injuries and distal neurovascular deficits. This may entail not just clinical examination but radiographs of proximal and distal joints.
When assessing x-rays it is important to assess for changes in length of the bone, the angulation of the distal bone, rotational effects, presence of material such as glass.
Key points in management of fractures
Immobilise the fracture including the proximal and distal joints
Carefully monitor and document neurovascular status, particularly following reduction and immobilisation
Manage infection including tetanus prophylaxis
IV broad spectrum antibiotics for open injuries
As a general principle all open fractures should be thoroughly debrided ( and internal fixation devices avoided or used with extreme caution)
Open fractures constitute an emergency and should be debrided and lavaged within 6 hours of injury
A 32 year old man is involved in a road traffic accident and sustains a significant laceration to the lateral aspect of the nose which is associated with tissue loss.
A. Wound excision and primary closure B. Simple primary closure C. Delayed primary closure D. Debridement and healing by secondary intention E. Split thickness skin graft F. Full thickness skin graft G. Free flap H. Pedicled flap I. Debridement and rotational flap
Debridement and rotational flap
Nasal injuries can be challenging to manage and where there is tissue loss, it can be difficult to primarily close them and still obtain a satisfactory aesthetic result. Debridement together with a rotational flap would obtain the best results here.
Primary closure
Clean wound, usually surgically created or following minor trauma
Standard suturing methods will usually suffice
Wound heals by primary intention
Delayed primary closure
Similar methods of actual closure to primary closure
May be used in situations where primary closure is either not achievable or not advisable e.g. infection
Vacuum assisted closure
Uses negative pressure therapy to facilitate wound closure
Sponge is inserted into wound cavity and then negative pressure applied
Advantages include removal of exudate and versatility
Disadvantages include cost and risk of fistulation if used incorrectly on sites such as bowel
Split thickness skin grafts
Superficial dermis removed with Watson knife or dermatome (commonly from thigh)
Remaining epithelium regenerates from dermal appendages
Coverage may be increased by meshing
Full thickness skin grafts
Whole dermal thickness is removed
Sub dermal fat is then removed and graft placed over donor site
Better cosmesis and flexibility at recipient site
Donor site “cost”
Flaps
Viable tissue with a blood supply
May be pedicled or free
Pedicled flaps are more reliable, but limited in range
Free flaps have greater range but carry greater risk of breakdown as they require vascular anastomosis
A 7 year old boy falls over and sustains a 6cm laceration to his head. On inspection his wound contains some dirt in it.
A. Wound excision and primary closure B. Simple primary closure C. Delayed primary closure D. Debridement and healing by secondary intention E. Split thickness skin graft F. Full thickness skin graft G. Free flap H. Pedicled flap I. Debridement and rotational flap
Wound excision and primary closure
By debriding the wound, the area can then be primarily closed. Prophylactic antibiotics should be administered.
Primary closure
Clean wound, usually surgically created or following minor trauma
Standard suturing methods will usually suffice
Wound heals by primary intention
Delayed primary closure
Similar methods of actual closure to primary closure
May be used in situations where primary closure is either not achievable or not advisable e.g. infection
Vacuum assisted closure
Uses negative pressure therapy to facilitate wound closure
Sponge is inserted into wound cavity and then negative pressure applied
Advantages include removal of exudate and versatility
Disadvantages include cost and risk of fistulation if used incorrectly on sites such as bowel
Split thickness skin grafts
Superficial dermis removed with Watson knife or dermatome (commonly from thigh)
Remaining epithelium regenerates from dermal appendages
Coverage may be increased by meshing
Full thickness skin grafts
Whole dermal thickness is removed
Sub dermal fat is then removed and graft placed over donor site
Better cosmesis and flexibility at recipient site
Donor site “cost”
Flaps
Viable tissue with a blood supply
May be pedicled or free
Pedicled flaps are more reliable, but limited in range
Free flaps have greater range but carry greater risk of breakdown as they require vascular anastomosis
A 45 year old man is gardening and damages his foot with a fork. On examination there are cutaneous defects and the surrounding skin looks dusky.
A. Wound excision and primary closure B. Simple primary closure C. Delayed primary closure D. Debridement and healing by secondary intention E. Split thickness skin graft F. Full thickness skin graft G. Free flap H. Pedicled flap I. Debridement and rotational flap
Debridement and healing by secondary intention
The skin changes described here should be debrided. Closure would not be safe with the skin changes documented and the wound should be left open.
Primary closure
Clean wound, usually surgically created or following minor trauma
Standard suturing methods will usually suffice
Wound heals by primary intention
Delayed primary closure
Similar methods of actual closure to primary closure
May be used in situations where primary closure is either not achievable or not advisable e.g. infection
Vacuum assisted closure
Uses negative pressure therapy to facilitate wound closure
Sponge is inserted into wound cavity and then negative pressure applied
Advantages include removal of exudate and versatility
Disadvantages include cost and risk of fistulation if used incorrectly on sites such as bowel
Split thickness skin grafts
Superficial dermis removed with Watson knife or dermatome (commonly from thigh)
Remaining epithelium regenerates from dermal appendages
Coverage may be increased by meshing
Full thickness skin grafts
Whole dermal thickness is removed
Sub dermal fat is then removed and graft placed over donor site
Better cosmesis and flexibility at recipient site
Donor site “cost”
Flaps
Viable tissue with a blood supply
May be pedicled or free
Pedicled flaps are more reliable, but limited in range
Free flaps have greater range but carry greater risk of breakdown as they require vascular anastomosis
A 19 year old student is involved in a head on car collision. He complains of severe chest pain. A Chest x-ray performed as part of a trauma series shows widening of the mediastinum. Which is the most likely injury in this scenario?
Rupture of the distal oesophagus Rupture of the left main bronchus Rupture of the aorta proximal to the left subclavian artery Rupture of the aorta distal to the left subclavian artery Rupture of the inferior vena cava
Rupture of the aorta distal to the left subclavian artery
The aorta may be injured in deceleration accidents. In the setting of deceleration injury, chest pain and mediastinal widening the most likely problem is aortic rupture. This will typically occur distal to the left subclavian artery. Rupture of the proximal aorta may occur. However, survival is unlikely. It is important to note that the question uses the term Most likely injury as this is the component that distinguishes an ascending rupture from a descending rupture.
Thoracic aorta rupture
Mechanism of injury: Decelerating force i.e. RTA, fall from a great height
Most people die at scene
Survivors may have an incomplete laceration at the ligamentum arteriosum of the aorta.
Clinical features
Contained haematoma: persistent hypotension
Detected mainly by history, CXR changes
CXR changes
Widened mediastinum
Trachea/Oesophagus to right
Depression of left main stem bronchus
Widened paratracheal stripe/paraspinal interfaces
Space between aorta and pulmonary artery obliterated
Rib fracture/left haemothorax
Diagnosis
Angiography, usually CT aortogram.
A 22 year old mechanic is involved in a fight. He is hit on the head with a hammer. On examination he had clinical evidence of an open depressed skull fracture and a GCS of 6/15.
A. Observation B. CT head within 1 hour C. CT head within 8 hours D. Urgent neurosurgical review (even before CT head performed) E. 3 view c-spine xray F. 2 view c-spine xray G. CT c-spine
Urgent neurosurgical review (even before CT head performed)
A patient with GCS <8 or = to 8 needs urgent neurosurgical review. Especially when an open fracture is present.
Head injury management- NICE Guidelines
Summary of guidelines
All patients should be assessed within 15 minutes on arrival to A&E
Document all 3 components of the GCS
If GCS <8 or = to 8, consider stabilising the airway
Treat pain with low dose IV opiates (if safe)
Full spine immobilisation until assessment if:
- GCS < 15
- neck pain/tenderness
- paraesthesia extremities
- focal neurological deficit
- suspected c-spine injury
If a c-spine injury is suspected a 3 view c-spine x-ray is indicated. CT c-spine is preferred if:
- Intubated
- GCS <13
- Normal x-ray but continued concerns regarding c-spine injury
- Any focal neurology
- A CT head scan is being performed
- Initial plain films are abnormal
Immediate CT head (within 1 hour) if: GCS < 13 on admission GCS < 15 2 hours after admission Suspected open or depressed skull fracture Suspected skull base fracture (panda eyes, Battle's sign, CSF from nose/ear, bleeding ear) Focal neurology Vomiting > 1 episode Post traumatic seizure Coagulopathy
Contact neurosurgeon if: Persistent GCS < 8 or = 8 Unexplained confusion > 4h Reduced GCS after admission Progressive neurological signs Incomplete recovery post seizure Penetrating injury Cerebrospinal fluid leak
Observations
1/2 hourly GCS until 15
A 67 year old retired lawyer falls down the stairs. His GCS is 15/15 and he has some bruising over the mastoid.
A. Observation B. CT head within 1 hour C. CT head within 8 hours D. Urgent neurosurgical review (even before CT head performed) E. 3 view c-spine xray F. 2 view c-spine xray G. CT c-spine
CT head within 1 hour
This patient has a basal skull fracture, which is indicated by a positive Battle’s sign. He should have a CT head within 1h.
Head injury management- NICE Guidelines
Summary of guidelines
All patients should be assessed within 15 minutes on arrival to A&E
Document all 3 components of the GCS
If GCS <8 or = to 8, consider stabilising the airway
Treat pain with low dose IV opiates (if safe)
Full spine immobilisation until assessment if:
- GCS < 15
- neck pain/tenderness
- paraesthesia extremities
- focal neurological deficit
- suspected c-spine injury
If a c-spine injury is suspected a 3 view c-spine x-ray is indicated. CT c-spine is preferred if:
- Intubated
- GCS <13
- Normal x-ray but continued concerns regarding c-spine injury
- Any focal neurology
- A CT head scan is being performed
- Initial plain films are abnormal
Immediate CT head (within 1 hour) if: GCS < 13 on admission GCS < 15 2 hours after admission Suspected open or depressed skull fracture Suspected skull base fracture (panda eyes, Battle's sign, CSF from nose/ear, bleeding ear) Focal neurology Vomiting > 1 episode Post traumatic seizure Coagulopathy
Contact neurosurgeon if: Persistent GCS < 8 or = 8 Unexplained confusion > 4h Reduced GCS after admission Progressive neurological signs Incomplete recovery post seizure Penetrating injury Cerebrospinal fluid leak
Observations
1/2 hourly GCS until 15
A 52 year old secretary falls down the stairs. She complains of neck pain. She has a GCS of 15/15 and no neurology. She is unable to rotate her c-spine 45 degrees to the left and right.
A. Observation B. CT head within 1 hour C. CT head within 8 hours D. Urgent neurosurgical review (even before CT head performed) E. 3 view c-spine xray F. 2 view c-spine xray G. CT c-spine
3 view c-spine xray
In the January 2014 NICE guidance relating to the diagnosis of head and spinal injury the evidence for initial imaging of the C spine was reviewed. The current UK practice is that “low risk” patients with pain but no neurology undergo a 3 view C spine x-ray. The more detailed 5 view x-ray was not found to be any better than 3 view films. In patients with high risk injuries (which the patient in the scenario does not have) there is a likelihood that 1 in 6 injuries would be missed on plain films alone and therefore CT scanning of the C spine is recommended in this group.
Head injury management- NICE Guidelines
Summary of guidelines
All patients should be assessed within 15 minutes on arrival to A&E
Document all 3 components of the GCS
If GCS <8 or = to 8, consider stabilising the airway
Treat pain with low dose IV opiates (if safe)
Full spine immobilisation until assessment if:
- GCS < 15
- neck pain/tenderness
- paraesthesia extremities
- focal neurological deficit
- suspected c-spine injury
If a c-spine injury is suspected a 3 view c-spine x-ray is indicated. CT c-spine is preferred if:
- Intubated
- GCS <13
- Normal x-ray but continued concerns regarding c-spine injury
- Any focal neurology
- A CT head scan is being performed
- Initial plain films are abnormal
Immediate CT head (within 1 hour) if: GCS < 13 on admission GCS < 15 2 hours after admission Suspected open or depressed skull fracture Suspected skull base fracture (panda eyes, Battle's sign, CSF from nose/ear, bleeding ear) Focal neurology Vomiting > 1 episode Post traumatic seizure Coagulopathy
Contact neurosurgeon if: Persistent GCS < 8 or = 8 Unexplained confusion > 4h Reduced GCS after admission Progressive neurological signs Incomplete recovery post seizure Penetrating injury Cerebrospinal fluid leak
Observations
1/2 hourly GCS until 15
A 21 year old badminton player attends A&E with a painful, swollen right arm. He is right handed. Clinically he has dusky fingers and his upper limb pulses are present. An axillary vein thrombosis is confirmed. What is the best acute treatment to achieve vein patency?
Intravenous heparin Warfarin Catheter directed tPA Low molecular weight heparin Aspirin
Catheter directed tPA
Axillary vein thrombosis
1-2% of all deep venous thrombosis
Primary cause is associated with trauma, thoracic outlet obstruction or repeated effort in a dominant arm (young active individuals)
Secondary causes include central line insertion, malignancy, pacemakers
Clinical features Pain and swelling (non pitting) Numbness Discolouration: mottling, dusky Pulses present Congested veins
Investigations FBC: viscosity, platelet function Clotting Liver function tests D-dimer Duplex scan: investigation of choice CT scan: thoracic outlet obstruction
Treatment
Local catheter directed TPA
Heparin
Warfarin
A 22 year old man is having a blood transfusion after losing blood from a peptic ulcer. He is normally fit and well. Four hours after the transfusion; he complains of sudden onset shortness of breath and chest pain. On examination his temperature is 37.2, saturations are 88% on air, blood pressure 100/55 mmHg and HR 110 bpm. He has crepitations bilaterally up to the midzones. He is given IV frusemide, but deteriorates and is admitted to ITU. A pulmonary catheter is inserted and the PCWP is 10 mmHg.
A. Neutrophilic febrile reaction B. Acute haemolytic transfusion reaction C. Delayed haemolytic transfusion reaction D. Pulmonary oedema E. Sickle cell crisis F. Transfusion associated lung injury G. Graft vs. Host disease
Transfusion associated lung injury
The pulmonary catheter reading indicates that this is not a case of fluid overload (the PCWP should be high, normal values PCWP systolic 7mmHg, diastolic 10mmHg). Transfusion associated lung injury is a rare reaction causing neutrophilic mediated allergic pulmonary oedema. Patient’s have antibodies to donor leukocytes. It is important to consider this as a diagnosis when patients don’t respond to treatment for pulmonary oedema. Patients normally respond to supportive therapy including fluids and oxygen.
Blood transfusion reactions
Acute transfusion reactions present as adverse signs or symptoms during or within 24 hours of a blood transfusion. The most frequent reactions are fever, chills, pruritus, or urticaria, which typically resolve promptly without specific treatment or complications. Other signs occurring in temporal relationship with a blood transfusion, such as severe dyspnoea, pyrexia, or loss of consciousness may be the first indication of a more severe potentially fatal reaction.
The causes of adverse reactions are multi-factorial. Immune mediated reactions, some of the most feared, occur as a result of component mismatch, the commonest cause of which is clerical error. More common, non immune mediated, complications may occur as a result of product contamination, this may be bacterial or viral.
Transfusion related lung injury is well recognised and there are two proposed mechanisms which underpin this. One involves the sequestration of primed neutrophils within the recipient pulmonary capillary bed. The other proposed mechanism suggests that HLA mismatches between donor neutrophils and recipient lung tissue is to blame.
A 32 year male with leukaemia attends the day unit for a blood transfusion. Five days after the transfusion he attends A&E with a temperature of 38.5, erythroderma and desquamation.
A. Neutrophilic febrile reaction B. Acute haemolytic transfusion reaction C. Delayed haemolytic transfusion reaction D. Pulmonary oedema E. Sickle cell crisis F. Transfusion associated lung injury G. Graft vs. Host disease
Graft vs. Host disease
This is associated with transfusion of unirradiated blood in immunosupressed patients. Transfusion associated GVHD can occur 4-30 days after a transfusion and follows a sub acute pathway. Patients may also have diarrhoea and abnormal liver function tests. Management involves steroid therapy.
Blood transfusion reactions
Acute transfusion reactions present as adverse signs or symptoms during or within 24 hours of a blood transfusion. The most frequent reactions are fever, chills, pruritus, or urticaria, which typically resolve promptly without specific treatment or complications. Other signs occurring in temporal relationship with a blood transfusion, such as severe dyspnoea, pyrexia, or loss of consciousness may be the first indication of a more severe potentially fatal reaction.
The causes of adverse reactions are multi-factorial. Immune mediated reactions, some of the most feared, occur as a result of component mismatch, the commonest cause of which is clerical error. More common, non immune mediated, complications may occur as a result of product contamination, this may be bacterial or viral.
Transfusion related lung injury is well recognised and there are two proposed mechanisms which underpin this. One involves the sequestration of primed neutrophils within the recipient pulmonary capillary bed. The other proposed mechanism suggests that HLA mismatches between donor neutrophils and recipient lung tissue is to blame.
A 22 year old man is having a blood transfusion after losing blood from haemorrhoids. He is normally fit and well. 3h during the transfusion he complains of sudden onset abdominal pain and nausea. His temperature is 39 degrees, Blood pressure 98/42 mmHg, HR 105 bpm and saturations 94% air. His urine appears dark.
A. Neutrophilic febrile reaction B. Acute haemolytic transfusion reaction C. Delayed haemolytic transfusion reaction D. Pulmonary oedema E. Sickle cell crisis F. Transfusion associated lung injury G. Graft vs. Host disease
Acute haemolytic transfusion reaction
Rapid intravascular haemolysis leading to shock, DIC and death can occur with this reaction.
Blood transfusion reactions
Acute transfusion reactions present as adverse signs or symptoms during or within 24 hours of a blood transfusion. The most frequent reactions are fever, chills, pruritus, or urticaria, which typically resolve promptly without specific treatment or complications. Other signs occurring in temporal relationship with a blood transfusion, such as severe dyspnoea, pyrexia, or loss of consciousness may be the first indication of a more severe potentially fatal reaction.
The causes of adverse reactions are multi-factorial. Immune mediated reactions, some of the most feared, occur as a result of component mismatch, the commonest cause of which is clerical error. More common, non immune mediated, complications may occur as a result of product contamination, this may be bacterial or viral.
Transfusion related lung injury is well recognised and there are two proposed mechanisms which underpin this. One involves the sequestration of primed neutrophils within the recipient pulmonary capillary bed. The other proposed mechanism suggests that HLA mismatches between donor neutrophils and recipient lung tissue is to blame.
A 49-year-old male sustained a severe blunt injury just below the bridge of the nose with industrial machinery. Imaging demonstrates a fracture involving the superior orbital fissure. On examination an ipsilateral pupillary defect is present and loss of the corneal reflexes. In addition to these examination findings, which of the following will not be present?
Altered cutaneous sensation from the forehead to the vertex Ptosis Complete opthalmoplegia Nystagmus Enopthalmos
Nystagmus
Orbital apex syndrome
This is an extension of superior orbital fissure syndrome and includes compression of the optic nerve passing through the optic foramen. It is indicated by features of superior orbital fissure syndrome and ipsilateral afferent pupillary defect.
This type of injury will result in the orbital apex syndrome (See above). As such opthalmoplegia will be present and nystagmus cannot occur.
Le Fort 1 The fracture extends from the nasal septum to the lateral pyriform rims, travels horizontally above the teeth apices, crosses below the zygomaticomaxillary junction, and traverses the pterygomaxillary junction to interrupt the pterygoid plates.
Le Fort 2 These fractures have a pyramidal shape and extend from the nasal bridge at or below the nasofrontal suture through the frontal process of the maxilla, inferolaterally through the lacrimal bones and inferior orbital floor and rim through or near the inferior orbital foramen, and inferiorly through the anterior wall of the maxillary sinus; it then travels under the zygoma, across the pterygomaxillary fissure, and through the pterygoid plates.
Le Fort 3 These fractures start at the nasofrontal and frontomaxillary sutures and extend posteriorly along the medial wall of the orbit through the nasolacrimal groove and ethmoid bones. The thicker sphenoid bone posteriorly usually prevents continuation of the fracture into the optic canal. Instead, the fracture continues along the floor of the orbit along the inferior orbital fissure and continues superolaterally through the lateral orbital wall, through the zygomaticofrontal junction and the zygomatic arch. Intranasally, a branch of the fracture extends through the base of the perpendicular plate of the ethmoid, through the vomer, and through the interface of the pterygoid plates to the base of the sphenoid. This type of fracture predisposes the patient to CSF rhinorrhea more commonly than the other types.
Ocular injuries
Superior orbital fissure syndrome
Severe force to the lateral wall of the orbit resulting in compression of neurovascular structures. Results in :
Complete opthalmoplegia and ptosis (Cranial nerves 3, 4, 6 and nerve to levator palpebrae superioris)
Relative afferent pupillary defect
Dilatation of the pupil and loss of accommodation and corneal reflexes
Altered sensation from forehead to vertex (frontal branch of trigeminal nerve)
Orbital blow out fracture
Typically occurs when an object of slightly larger diameter than the orbital rim strikes the incompressible eyeball. The bone fragment is displaced downwards into the antral cavity, remaining attached to the orbital periosteum. Periorbital fat may be herniated through the defect, interfering with the inferior rectus and inferior oblique muscles which are contained within the same fascial sheath. This prevents upward movement and outward rotation of the eye and the patient experiences diplopia on upward gaze. The initial bruising and swelling may make assessment difficult and patients should usually be reviewed 5 days later. Residual defects may require orbital floor reconstruction.
Nasal Fractures
Common injury
Ensure new and not old deformity
Control epistaxis
CSF rhinorrhoea implies that the cribriform plate has been breached and antibiotics will be required.
Usually best to allow bruising and swelling to settle and then review patient clinically. Major persistent deformity requires fracture manipulation, best performed within 10 days of injury.
Retrobulbar haemorrhage
Rare but important ocular emergency. Presents with:
Pain (usually sharp and within the globe)
Proptosis
Pupil reactions are lost
Paralysis (eye movements lost)
Visual acuity is lost (colour vision is lost first)
May be the result of Le Fort type facial fractures.
Management:
Mannitol 1g/Kg as 20% infusion, Osmotic diuretic, Contra-indicated in congestive heart failure and pulmonary oedema
Acetazolamide 500mg IV, (Monitor FBC/U+E) Reduces aqueous pressure by inhibition of carbonic anhydrase (used in glaucoma)
Dexamethasone 8mg orally or intravenously
In a traumatic setting an urgent cantholysis may be needed prior to definitive surgery.
Consider
Papaverine 40mg smooth muscle relaxant
Dextran 40 500mls IV improves perfusion
A 10 year old child is admitted with severe 30% burns following a house fire. After wound cleaning and dressings he is admitted to critical care. 1 day following skin grafts he becomes tachycardic and hypotensive. He vomits twice and this shows evidence of haematemesis
A. Deep vein thrombosis B. Curlings Ulcer C. Contracture D. Type I respiratory failure E. Type II respiratory failure F. Toxic shock syndrome G. Compartment syndrome H. Rhabdomyolysis I. Disseminated intravascular coagulation
Curlings Ulcer
Stress ulcers may occur in the duodenum of burns patients and are more common in children.
Depth of burn assessment Bleeding on needle prick Sensation Appearance Blanching to pressure
Percentage burn estimation
Lund Browder chart: most accurate even in children
Wallace rule of nines
Palmar surface: surface area palm = 0.8% burn
> 15% body surface area burns in adults needs urgent burn fluid resuscitation
Transfer to burn centre if: Need burn shock resuscitation Face/hands/genitals affected Deep partial thickness or full thickness burns Significant electrical/chemical burns
Management
The initial aim is to stop the burning process and resuscitate the patient. Intravenous fluids will be required for children with burns greater than 10% of total body surface area. Adults with burns greater than 15% of total body surface area will also require IV fluids. The fluids are calculated using the Parkland formula which is; volume of fluid= total body surface area of the burn % x weight (Kg) x4. Half of the fluid is administered in the first 8 hours. A urinary catheter should be inserted. Analgesia should be given. Complex burns, burns involving the hand perineum and face and burns >10% in adults and >5% in children should be transferred to a burns unit.
Circumferential burns affecting a limb or severe torso burns impeding respiration may require escharotomy to divide the burnt tissue.
Conservative management is appropriate for superficial burns and mixed superficial burns that will heal in 2 weeks. More complex burns may require excision and skin grafting. Excision and primary closure is not generally practised as there is a high risk of infection.
There is no evidence to support the use of anti microbial prophylaxis or topical antibiotics in burn patients.
Escharotomies
Indicated in circumferential full thickness burns to the torso or limbs.
Careful division of the encasing band of burn tissue will potentially improve ventilation (if the burn involves the torso), or relieve compartment syndrome and oedema (where a limb is involved)
A 26 year old electrician suffers a full thickness high voltage burn to his leg. On routine urine analysis he has + blood. His U+E’s show mild hyperkalaemia and a CK of 3000
A. Deep vein thrombosis B. Curlings Ulcer C. Contracture D. Type I respiratory failure E. Type II respiratory failure F. Toxic shock syndrome G. Compartment syndrome H. Rhabdomyolysis I. Disseminated intravascular coagulation
Rhabdomyolysis
Electrical high voltage burns are associated with rhabdomyolysis. Acute tubular necrosis may occur. Aggressive IV fluids should be given
Depth of burn assessment Bleeding on needle prick Sensation Appearance Blanching to pressure
Percentage burn estimation
Lund Browder chart: most accurate even in children
Wallace rule of nines
Palmar surface: surface area palm = 0.8% burn
> 15% body surface area burns in adults needs urgent burn fluid resuscitation
Transfer to burn centre if: Need burn shock resuscitation Face/hands/genitals affected Deep partial thickness or full thickness burns Significant electrical/chemical burns
Management
The initial aim is to stop the burning process and resuscitate the patient. Intravenous fluids will be required for children with burns greater than 10% of total body surface area. Adults with burns greater than 15% of total body surface area will also require IV fluids. The fluids are calculated using the Parkland formula which is; volume of fluid= total body surface area of the burn % x weight (Kg) x4. Half of the fluid is administered in the first 8 hours. A urinary catheter should be inserted. Analgesia should be given. Complex burns, burns involving the hand perineum and face and burns >10% in adults and >5% in children should be transferred to a burns unit.
Circumferential burns affecting a limb or severe torso burns impeding respiration may require escharotomy to divide the burnt tissue.
Conservative management is appropriate for superficial burns and mixed superficial burns that will heal in 2 weeks. More complex burns may require excision and skin grafting. Excision and primary closure is not generally practised as there is a high risk of infection.
There is no evidence to support the use of anti microbial prophylaxis or topical antibiotics in burn patients.
Escharotomies
Indicated in circumferential full thickness burns to the torso or limbs.
Careful division of the encasing band of burn tissue will potentially improve ventilation (if the burn involves the torso), or relieve compartment syndrome and oedema (where a limb is involved)
A 45 year old man is admitted after his clothing caught fire. He suffers a full thickness circumferential burn to his lower thigh. He complains of increasing pain in lower leg and on examination there is parasthesia and severe pain in the lower leg. Foot pulses are normal
A. Deep vein thrombosis B. Curlings Ulcer C. Contracture D. Type I respiratory failure E. Type II respiratory failure F. Toxic shock syndrome G. Compartment syndrome H. Rhabdomyolysis I. Disseminated intravascular coagulation
Compartment syndrome
Circumferential burns may constrict the limb and cause a compartment syndrome to develop. Eshcarotomy is required, and compartmental decompression.
Depth of burn assessment Bleeding on needle prick Sensation Appearance Blanching to pressure
Percentage burn estimation
Lund Browder chart: most accurate even in children
Wallace rule of nines
Palmar surface: surface area palm = 0.8% burn
> 15% body surface area burns in adults needs urgent burn fluid resuscitation
Transfer to burn centre if: Need burn shock resuscitation Face/hands/genitals affected Deep partial thickness or full thickness burns Significant electrical/chemical burns
Management
The initial aim is to stop the burning process and resuscitate the patient. Intravenous fluids will be required for children with burns greater than 10% of total body surface area. Adults with burns greater than 15% of total body surface area will also require IV fluids. The fluids are calculated using the Parkland formula which is; volume of fluid= total body surface area of the burn % x weight (Kg) x4. Half of the fluid is administered in the first 8 hours. A urinary catheter should be inserted. Analgesia should be given. Complex burns, burns involving the hand perineum and face and burns >10% in adults and >5% in children should be transferred to a burns unit.
Circumferential burns affecting a limb or severe torso burns impeding respiration may require escharotomy to divide the burnt tissue.
Conservative management is appropriate for superficial burns and mixed superficial burns that will heal in 2 weeks. More complex burns may require excision and skin grafting. Excision and primary closure is not generally practised as there is a high risk of infection.
There is no evidence to support the use of anti microbial prophylaxis or topical antibiotics in burn patients.
Escharotomies
Indicated in circumferential full thickness burns to the torso or limbs.
Careful division of the encasing band of burn tissue will potentially improve ventilation (if the burn involves the torso), or relieve compartment syndrome and oedema (where a limb is involved)
A 52 year old lady develops lower leg swelling following redo varicose vein surgery. There is evidence of swelling of the left leg up to the knee. The overlying skin appears healthy.
A. Homans operation B. Charles operation C. Frusemide at high doses D. Frusemide at low doses E. Multilayer compression bandaging F. Lymphovenous anastomosis
Multilayer compression bandaging
Unfortunately lymphoedema may complicate redo varicose vein surgery (in 0.5% of cases). As the presentation is mild, she should be managed using compression hosiery. Diuretics do not help in cases of true lymphoedema and a dramatic response suggests an alternative underlying cause.
Lymphoedema
Due to impaired lymphatic drainage in the presence of normal capillary function.
Lymphoedema causes the accumulation of protein rich fluid, subdermal fibrosis and dermal thickening.
Characteristically fluid is confined to the epifascial space (skin and subcutaneous tissues); muscle compartments are free of oedema. It involves the foot, unlike other forms of oedema. There may be a ‘buffalo hump’ on the dorsum of the foot and the skin cannot be pinched due to subcutaneous fibrosis.
Causes of lymphoedema
Primary
Congenital < 1 year: sporadic, Milroy’s disease
Onset 1-35 years: sporadic, Meige’s disease
> 35 years: Tarda
Secondary
Bacterial/fungal/parasitic infection (filariasis)
Lymphatic malignancy
Radiotherapy to lymph nodes
Surgical resection of lymph nodes
DVT
Thrombophlebitis
Indications for surgery
Marked disability or deformity from limb swelling
Lymphoedema caused by proximal lymphatic obstruction with patent distal lymphatics suitable for a lymphatic drainage procedure
Lymphocutaneous fistulae and megalymphatics
Procedures
Homans operation Reduction procedure with preservation of overlying skin (which must be in good condition). Skin flaps are raised and the underlying tissue excised. Limb circumference typically reduced by a third.
Charles operation All skin and subcutaneous tissue around the calf are excised down to the deep fascia. Split skin grafts are placed over the site. May be performed if overlying skin is not in good condition. Larger reduction in size than with Homans procedure.
Lymphovenous anastamosis Identifiable lymphatics are anastomosed to sub dermal venules. Usually indicated in 2% of patients with proximal lymphatic obstruction and normal distal lymphatics.
A 57 year old lady has suffered from lymphoedema for many years. The left leg is swollen to the mid thigh. Severe limb deformity has developed as a result of process and in spite of compression hosiery. Lymphoscintography shows no patent lymphatics in the proximal leg. The overlying skin is healthy.
A. Homans operation B. Charles operation C. Frusemide at high doses D. Frusemide at low doses E. Multilayer compression bandaging F. Lymphovenous anastomosis
Homans operation
Surgery is indicated in less than 10% of cases. However, severe deformity is one of the indications for surgery. Lymphovenous anastomosis is indicated where the proximal lymphatics are not patent. When the overlying skin is healthy (and limb deformity a problem), a Homans procedure is a reasonable first line operative option.
Lymphoedema
Due to impaired lymphatic drainage in the presence of normal capillary function.
Lymphoedema causes the accumulation of protein rich fluid, subdermal fibrosis and dermal thickening.
Characteristically fluid is confined to the epifascial space (skin and subcutaneous tissues); muscle compartments are free of oedema. It involves the foot, unlike other forms of oedema. There may be a ‘buffalo hump’ on the dorsum of the foot and the skin cannot be pinched due to subcutaneous fibrosis.
Causes of lymphoedema
Primary
Congenital < 1 year: sporadic, Milroy’s disease
Onset 1-35 years: sporadic, Meige’s disease
> 35 years: Tarda
Secondary
Bacterial/fungal/parasitic infection (filariasis)
Lymphatic malignancy
Radiotherapy to lymph nodes
Surgical resection of lymph nodes
DVT
Thrombophlebitis
Indications for surgery
Marked disability or deformity from limb swelling
Lymphoedema caused by proximal lymphatic obstruction with patent distal lymphatics suitable for a lymphatic drainage procedure
Lymphocutaneous fistulae and megalymphatics
Procedures
Homans operation Reduction procedure with preservation of overlying skin (which must be in good condition). Skin flaps are raised and the underlying tissue excised. Limb circumference typically reduced by a third.
Charles operation All skin and subcutaneous tissue around the calf are excised down to the deep fascia. Split skin grafts are placed over the site. May be performed if overlying skin is not in good condition. Larger reduction in size than with Homans procedure.
Lymphovenous anastamosis Identifiable lymphatics are anastomosed to sub dermal venules. Usually indicated in 2% of patients with proximal lymphatic obstruction and normal distal lymphatics.
A 38 year old lady is troubled by lymphoedema that occurred following a block dissection of the groin for malignant melanoma many years previously. Despite therapy with compression bandages she has persistent lower leg swelling impairing her activities of daily living. She has no evidence of recurrent malignancy. Lymphoscintography demonstrates occlusion of the groin lymphatics. However, the distal lymphatic system appears healthy.
A. Homans operation B. Charles operation C. Frusemide at high doses D. Frusemide at low doses E. Multilayer compression bandaging F. Lymphovenous anastomosis
Lymphovenous anastomosis
In young patients with proximal disease and healthy distal lymphatics a lymphovenous anastomosis may be considered. Such cases are rare.
Lymphoedema
Due to impaired lymphatic drainage in the presence of normal capillary function.
Lymphoedema causes the accumulation of protein rich fluid, subdermal fibrosis and dermal thickening.
Characteristically fluid is confined to the epifascial space (skin and subcutaneous tissues); muscle compartments are free of oedema. It involves the foot, unlike other forms of oedema. There may be a ‘buffalo hump’ on the dorsum of the foot and the skin cannot be pinched due to subcutaneous fibrosis.
Causes of lymphoedema
Primary
Congenital < 1 year: sporadic, Milroy’s disease
Onset 1-35 years: sporadic, Meige’s disease
> 35 years: Tarda
Secondary
Bacterial/fungal/parasitic infection (filariasis)
Lymphatic malignancy
Radiotherapy to lymph nodes
Surgical resection of lymph nodes
DVT
Thrombophlebitis
Indications for surgery
Marked disability or deformity from limb swelling
Lymphoedema caused by proximal lymphatic obstruction with patent distal lymphatics suitable for a lymphatic drainage procedure
Lymphocutaneous fistulae and megalymphatics
Procedures
Homans operation Reduction procedure with preservation of overlying skin (which must be in good condition). Skin flaps are raised and the underlying tissue excised. Limb circumference typically reduced by a third.
Charles operation All skin and subcutaneous tissue around the calf are excised down to the deep fascia. Split skin grafts are placed over the site. May be performed if overlying skin is not in good condition. Larger reduction in size than with Homans procedure.
Lymphovenous anastamosis Identifiable lymphatics are anastomosed to sub dermal venules. Usually indicated in 2% of patients with proximal lymphatic obstruction and normal distal lymphatics.
A 42 year old woman is admitted to surgery with acute cholecystitis. She is known to have hypertension, rheumatoid arthritis and polymyalgia rheumatica. Her medical therapy includes: Paracetamol 1g qds Codeine phosphate 30mg qds Bendrofluazide 2.5 mg od Ramipril 10mg od Methotrexate 7.5mg once a week Prednisolone 5mg od You are called by the CT1 to assess this lady as she has become delirious and hypotensive 2 hours after surgery. Her blood results reveal:
Na+ 132 mmol/l
K+ 5.3 mmol/l
Urea 7 mmol/l
Creatinine 108 µmol/l
Hb 12.4 g/dl
Platelets 178 * 109/l
WBC 15.4 * 109/l
What management is needed immediately?
Ceftriaxone IV Hydrocortisone 50mg IV CT scan abdomen Urgent exploratory laparotomy Hydrocortisone 100mg IV
Hydrocortisone 100mg IV
This patient has acute adrenal insufficiency and urgently needs steroid replacement.
Addisonian crisis
Causes
Sepsis or surgery causing an acute exacerbation of chronic insufficiency (Addison’s, Hypopituitarism)
Adrenal haemorrhage eg Waterhouse-Friderichsen syndrome (fulminant meningococcemia)
Steroid withdrawal
Management
Hydrocortisone 100 mg im or iv
1 litre normal saline infused over 30-60 mins or with dextrose if hypoglycaemic
Continue hydrocortisone 6 hourly until the patient is stable. No fludrocortisone is required because high cortisol exerts weak mineralocorticoid action
Oral replacement may begin after 24 hours and be reduced to maintenance over 3-4 days
A 28 year old male is involved in a road traffic accident he is thrown from his motorbike onto the pavement and sustains a haemopneumothorax and flail segment of the right chest.
A. Thoracotomy
B. Manage conservatively
C. Intercostal tube drain insertion
D. CT scanning
E. Bronchoscopy
F. Negative pressure intercostal tube drainage
G. Video assisted thoracoscopy and pleurectomy
Intercostal tube drain insertion
He requires a chest drain and analgesia. In general all haemopneumothoraces should be managed by intercostal chest drain insertion as they have a risk of becoming a tension pneumothorax until the lung laceration has sealed.
Thoracic trauma
Types of thoracic trauma
Tension pneumothorax
Often laceration to lung parenchyma with flap
Pressure develops in thorax
Most common cause is mechanical ventilation in patient with pleural injury
Symptoms overlap with cardiac tamponade, hyper-resonant percussion note is more likely in tension pnemothorax
Flail chest
Chest wall disconnects from thoracic cage
Multiple rib fractures (at least two fractures per rib in at least two ribs)
Associated with pulmonary contusion
Abnormal chest motion
Avoid over hydration and fluid overload
Pneumothorax
Most common cause is lung laceration with air leakage
Most traumatic pneumothoraces should have a chest drain
Patients with traumatic pneumothorax should never be mechanically ventilated until a chest drain is inserted
Haemothorax
Most commonly due to laceration of lung, intercostal vessel or internal mammary artery
Haemothoraces large enough to appear on CXR are treated with large bore chest drain
Surgical exploration is warranted if >1500ml blood drained immediately
Cardiac tamponade
Beck’s triad: elevated venous pressure, reduced arterial pressure, reduced heart sounds
Pulsus paradoxus
May occur with as little as 100ml blood
Pulmonary contusion
Most common potentially lethal chest injury
Arterial blood gases and pulse oximetry important
Early intubation within an hour if significant hypoxia
Blunt cardiac injury
Usually occurs secondary to chest wall injury
ECG may show features of myocardial infarction
Sequelae: hypotension, arrhythmias, cardiac wall motion abnormalities
Aorta disruption
Deceleration injuries
Contained haematoma
Widened mediastinum
Diaphragm disruption
Most due to motor vehicle accidents and blunt trauma causing large radial tears (laceration injuries result in small tears)
More common on left side
Insert gastric tube, may pass into intrathoracic stomach
Mediastinal traversing wounds
Entrance wound in one hemithorax and exit wound/foreign body in opposite hemithorax
Mediastinal haematoma or pleural cap suggests great vessel injury
Mortality is 20%
A 19 year old man is stabbed in the chest at a nightclub. He develops a cardiac arrest in casualty following an attempted transfer to the CT scanning room
A. Thoracotomy
B. Manage conservatively
C. Intercostal tube drain insertion
D. CT scanning
E. Bronchoscopy
F. Negative pressure intercostal tube drainage
G. Video assisted thoracoscopy and pleurectomy
Thoracotomy
This is one indication for an ‘emergency room’ thoracotomy, there are not many others! Typical injuries include ventricular penetration, great vessel disruption and hilar lung injuries.
Thoracic trauma
Types of thoracic trauma
Tension pneumothorax
Often laceration to lung parenchyma with flap
Pressure develops in thorax
Most common cause is mechanical ventilation in patient with pleural injury
Symptoms overlap with cardiac tamponade, hyper-resonant percussion note is more likely in tension pnemothorax
Flail chest
Chest wall disconnects from thoracic cage
Multiple rib fractures (at least two fractures per rib in at least two ribs)
Associated with pulmonary contusion
Abnormal chest motion
Avoid over hydration and fluid overload
Pneumothorax
Most common cause is lung laceration with air leakage
Most traumatic pneumothoraces should have a chest drain
Patients with traumatic pneumothorax should never be mechanically ventilated until a chest drain is inserted
Haemothorax
Most commonly due to laceration of lung, intercostal vessel or internal mammary artery
Haemothoraces large enough to appear on CXR are treated with large bore chest drain
Surgical exploration is warranted if >1500ml blood drained immediately
Cardiac tamponade
Beck’s triad: elevated venous pressure, reduced arterial pressure, reduced heart sounds
Pulsus paradoxus
May occur with as little as 100ml blood
Pulmonary contusion
Most common potentially lethal chest injury
Arterial blood gases and pulse oximetry important
Early intubation within an hour if significant hypoxia
Blunt cardiac injury
Usually occurs secondary to chest wall injury
ECG may show features of myocardial infarction
Sequelae: hypotension, arrhythmias, cardiac wall motion abnormalities
Aorta disruption
Deceleration injuries
Contained haematoma
Widened mediastinum
Diaphragm disruption
Most due to motor vehicle accidents and blunt trauma causing large radial tears (laceration injuries result in small tears)
More common on left side
Insert gastric tube, may pass into intrathoracic stomach
Mediastinal traversing wounds
Entrance wound in one hemithorax and exit wound/foreign body in opposite hemithorax
Mediastinal haematoma or pleural cap suggests great vessel injury
Mortality is 20%
A 32 year old male falls over and sustains a small pneumothorax following a simple rib fracture. He has no physiological compromise.
A. Thoracotomy
B. Manage conservatively
C. Intercostal tube drain insertion
D. CT scanning
E. Bronchoscopy
F. Negative pressure intercostal tube drainage
G. Video assisted thoracoscopy and pleurectomy
Intercostal tube drain insertion
Unlike spontaneous pneumothoraces most would advocate chest tube drainage in the context of pneumothorax resulting from trauma. This is because of the risk of the lung laceration developing a tension. Once there is no further evidence of air leak the chest drain may be removed and a check x-ray performed to check there is no re-accumulation prior to discharge.
Thoracic trauma
Types of thoracic trauma
Tension pneumothorax
Often laceration to lung parenchyma with flap
Pressure develops in thorax
Most common cause is mechanical ventilation in patient with pleural injury
Symptoms overlap with cardiac tamponade, hyper-resonant percussion note is more likely in tension pnemothorax
Flail chest
Chest wall disconnects from thoracic cage
Multiple rib fractures (at least two fractures per rib in at least two ribs)
Associated with pulmonary contusion
Abnormal chest motion
Avoid over hydration and fluid overload
Pneumothorax
Most common cause is lung laceration with air leakage
Most traumatic pneumothoraces should have a chest drain
Patients with traumatic pneumothorax should never be mechanically ventilated until a chest drain is inserted
Haemothorax
Most commonly due to laceration of lung, intercostal vessel or internal mammary artery
Haemothoraces large enough to appear on CXR are treated with large bore chest drain
Surgical exploration is warranted if >1500ml blood drained immediately
Cardiac tamponade
Beck’s triad: elevated venous pressure, reduced arterial pressure, reduced heart sounds
Pulsus paradoxus
May occur with as little as 100ml blood
Pulmonary contusion
Most common potentially lethal chest injury
Arterial blood gases and pulse oximetry important
Early intubation within an hour if significant hypoxia
Blunt cardiac injury
Usually occurs secondary to chest wall injury
ECG may show features of myocardial infarction
Sequelae: hypotension, arrhythmias, cardiac wall motion abnormalities
Aorta disruption
Deceleration injuries
Contained haematoma
Widened mediastinum
Diaphragm disruption
Most due to motor vehicle accidents and blunt trauma causing large radial tears (laceration injuries result in small tears)
More common on left side
Insert gastric tube, may pass into intrathoracic stomach
Mediastinal traversing wounds
Entrance wound in one hemithorax and exit wound/foreign body in opposite hemithorax
Mediastinal haematoma or pleural cap suggests great vessel injury
Mortality is 20%
A 38 year old man is playing football when he slips over during a tackle. His knee is painful immediately following the fall. Several hours later he notices that the knee has become swollen. Following a course of non steroidal anti inflammatory drugs and rest the situation improves. However, complains of recurrent pain. On assessment in clinic you notice that it is impossible to fully extend the knee, although the patient is able to do so when asked.
A. Anterior cruciate ligament rupture B. Posterior cruciate ligament rupture C. Medial collateral ligament tear D. Lateral collateral ligament tear E. Torn meniscus F. Chondromalacia patellae G. Dislocated patella H. Fractured patella I. Tibial plateau fracture
Torn meniscus
Twisting sporting injuries followed by delayed onset of knee swelling and locking are strongly suggestive of a menisceal tear. Arthroscopic menisectomy is the usual treatment.
Knee injury
Types of injury
Ruptured anterior cruciate ligament
Sport injury
Mechanism: high twisting force applied to a bent knee
Typically presents with: loud crack, pain and RAPID joint swelling (haemoarthrosis)
Poor healing
Management: intense physiotherapy or surgery
Ruptured posterior cruciate ligament
Mechanism: hyperextension injuries
Tibia lies back on the femur
Paradoxical anterior draw test
Rupture of medial collateral ligament
Mechanism: leg forced into valgus via force outside the leg
Knee unstable when put into valgus position
Menisceal tear
Rotational sporting injuries
Delayed knee swelling
Joint locking (Patient may develop skills to “unlock” the knee
Recurrent episodes of pain and effusions are common, often following minor trauma
Chondromalacia patellae
Teenage girls, following an injury to knee e.g. Dislocation patella
Typical history of pain on going downstairs or at rest
Tenderness, quadriceps wasting
Dislocation of the patella
Most commonly occurs as a traumatic primary event, either through direct trauma or through severe contraction of quadriceps with knee stretched in valgus and external rotation
Genu valgum, tibial torsion and high riding patella are risk factors
Skyline x-ray views of patella are required, although displaced patella may be clinically obvious
An osteochondral fracture is present in 5%
The condition has a 20% recurrence rate
Fractured patella
2 types:
i. Direct blow to patella causing undisplaced fragments
ii. Avulsion fracture
Tibial plateau fracture
Occur in the elderly (or following significant trauma in young)
Mechanism: knee forced into valgus or varus, but the knee fractures before the ligaments rupture
Varus injury affects medial plateau and if valgus injury, lateral plateau depressed fracture occurs
Classified using the Schatzker system (see below)
A 34 year old woman is a passenger in a car during an accident. Her knee hits the dashboard. On examination the tibia looks posterior compared to the non injured knee.
A. Anterior cruciate ligament rupture B. Posterior cruciate ligament rupture C. Medial collateral ligament tear D. Lateral collateral ligament tear E. Torn meniscus F. Chondromalacia patellae G. Dislocated patella H. Fractured patella I. Tibial plateau fracture
Posterior cruciate ligament rupture
In ruptured posterior cruciate ligament the tibia lies back on the femur and can be drawn forward during a paradoxical draw test.
Knee injury
Types of injury
Ruptured anterior cruciate ligament
Sport injury
Mechanism: high twisting force applied to a bent knee
Typically presents with: loud crack, pain and RAPID joint swelling (haemoarthrosis)
Poor healing
Management: intense physiotherapy or surgery
Ruptured posterior cruciate ligament
Mechanism: hyperextension injuries
Tibia lies back on the femur
Paradoxical anterior draw test
Rupture of medial collateral ligament
Mechanism: leg forced into valgus via force outside the leg
Knee unstable when put into valgus position
Menisceal tear
Rotational sporting injuries
Delayed knee swelling
Joint locking (Patient may develop skills to “unlock” the knee
Recurrent episodes of pain and effusions are common, often following minor trauma
Chondromalacia patellae
Teenage girls, following an injury to knee e.g. Dislocation patella
Typical history of pain on going downstairs or at rest
Tenderness, quadriceps wasting
Dislocation of the patella
Most commonly occurs as a traumatic primary event, either through direct trauma or through severe contraction of quadriceps with knee stretched in valgus and external rotation
Genu valgum, tibial torsion and high riding patella are risk factors
Skyline x-ray views of patella are required, although displaced patella may be clinically obvious
An osteochondral fracture is present in 5%
The condition has a 20% recurrence rate
Fractured patella
2 types:
i. Direct blow to patella causing undisplaced fragments
ii. Avulsion fracture
Tibial plateau fracture
Occur in the elderly (or following significant trauma in young)
Mechanism: knee forced into valgus or varus, but the knee fractures before the ligaments rupture
Varus injury affects medial plateau and if valgus injury, lateral plateau depressed fracture occurs
Classified using the Schatzker system (see below)
A 28 year old professional footballer is admitted to the emergency department. During a tackle he is twisted with his knee flexed. He hears a loud crack and his knee rapidly becomes swollen.
A. Anterior cruciate ligament rupture B. Posterior cruciate ligament rupture C. Medial collateral ligament tear D. Lateral collateral ligament tear E. Torn meniscus F. Chondromalacia patellae G. Dislocated patella H. Fractured patella I. Tibial plateau fracture
Anterior cruciate ligament rupture
This is common in footballers as the football boot studs stick to the ground and high twisting force is applied to a flexed knee. Rapid joint swelling also supports the diagnosis.
Knee injury
Types of injury
Ruptured anterior cruciate ligament
Sport injury
Mechanism: high twisting force applied to a bent knee
Typically presents with: loud crack, pain and RAPID joint swelling (haemoarthrosis)
Poor healing
Management: intense physiotherapy or surgery
Ruptured posterior cruciate ligament
Mechanism: hyperextension injuries
Tibia lies back on the femur
Paradoxical anterior draw test
Rupture of medial collateral ligament
Mechanism: leg forced into valgus via force outside the leg
Knee unstable when put into valgus position
Menisceal tear
Rotational sporting injuries
Delayed knee swelling
Joint locking (Patient may develop skills to “unlock” the knee
Recurrent episodes of pain and effusions are common, often following minor trauma
Chondromalacia patellae
Teenage girls, following an injury to knee e.g. Dislocation patella
Typical history of pain on going downstairs or at rest
Tenderness, quadriceps wasting
Dislocation of the patella
Most commonly occurs as a traumatic primary event, either through direct trauma or through severe contraction of quadriceps with knee stretched in valgus and external rotation
Genu valgum, tibial torsion and high riding patella are risk factors
Skyline x-ray views of patella are required, although displaced patella may be clinically obvious
An osteochondral fracture is present in 5%
The condition has a 20% recurrence rate
Fractured patella
2 types:
i. Direct blow to patella causing undisplaced fragments
ii. Avulsion fracture
Tibial plateau fracture
Occur in the elderly (or following significant trauma in young)
Mechanism: knee forced into valgus or varus, but the knee fractures before the ligaments rupture
Varus injury affects medial plateau and if valgus injury, lateral plateau depressed fracture occurs
Classified using the Schatzker system (see below)
A 65 year old diabetic female presents with a painless ulcer at the medial malleolus, it has been present for the past 16 years. On examination she has evidence of truncal varicosities and a brownish discolouration of the skin overlying the affected area.
A. Mixed ulcer B. Chronic obliterative arterial disease C. Superficial venous insufficiency D. Deep venous insufficiency E. Neuropathic ulcer F. Basal cell carcinoma G. Squamous cell carcinoma
Superficial venous insufficiency
Venous ulcers are usually associated with features of venous insufficiency. These include haemosiderin deposition and varicose veins. Neuropathic ulcers will tend to present at sites of pressure, which is not typically at the medial malleolus.
Venous leg ulcers
Most due to venous hypertension, secondary to chronic venous insufficiency (other causes include calf pump dysfunction or neuromuscular disorders)
Ulcers form due to capillary fibrin cuff or leucocyte sequestration
Features of venous insufficiency include oedema, brown pigmentation, lipodermatosclerosis, eczema
Location above the ankle, painless
Deep venous insufficiency is related to previous DVT and superficial venous insufficiency is associated with varicose veins
Doppler ultrasound looks for presence of reflux and duplex ultrasound looks at the anatomy/ flow of the vein
Management: 4 layer compression banding after exclusion of arterial disease or surgery
If fail to heal after 12 weeks or >10cm2 skin grafting may be needed
Marjolin’s ulcer
Image sourced from Wikipedia
Squamous cell carcinoma
Occurring at sites of chronic inflammation e.g; burns, osteomyelitis after 10-20 years
Mainly occur on the lower limb
Arterial ulcers Occur on the toes and heel Painful There may be areas of gangrene Cold with no palpable pulses Low ABPI measurements
Neuropathic ulcers
Commonly over plantar surface of metatarsal head and plantar surface of hallux
The plantar neuropathic ulcer is the condition that most commonly leads to amputation in diabetic patients
Due to pressure
Management includes cushioned shoes to reduce callus formation
Pyoderma gangrenosum
Image sourced from Wikipedia
Associated with inflammatory bowel disease/RA
Can occur at stoma sites
Erythematous nodules or pustules which ulcerate
A 71 year old man presents with a painful lower calf ulcer, mild pitting oedema and an ABPI of 0.3.
A. Mixed ulcer B. Chronic obliterative arterial disease C. Superficial venous insufficiency D. Deep venous insufficiency E. Neuropathic ulcer F. Basal cell carcinoma G. Squamous cell carcinoma
Chronic obliterative arterial disease
Painful ulcers associated with a low ABPI are usually arterial in nature. The question does not indicate that features of chronic venous insufficiency are present. Patients may have mild pitting oedema as many vascular patients will also have ischaemic heart disease and elevated right heart pressures. The absence of more compelling signs of venous insufficiency makes a mixed ulcer less likely.
Venous leg ulcers
Most due to venous hypertension, secondary to chronic venous insufficiency (other causes include calf pump dysfunction or neuromuscular disorders)
Ulcers form due to capillary fibrin cuff or leucocyte sequestration
Features of venous insufficiency include oedema, brown pigmentation, lipodermatosclerosis, eczema
Location above the ankle, painless
Deep venous insufficiency is related to previous DVT and superficial venous insufficiency is associated with varicose veins
Doppler ultrasound looks for presence of reflux and duplex ultrasound looks at the anatomy/ flow of the vein
Management: 4 layer compression banding after exclusion of arterial disease or surgery
If fail to heal after 12 weeks or >10cm2 skin grafting may be needed
Marjolin’s ulcer
Image sourced from Wikipedia
Squamous cell carcinoma
Occurring at sites of chronic inflammation e.g; burns, osteomyelitis after 10-20 years
Mainly occur on the lower limb
Arterial ulcers Occur on the toes and heel Painful There may be areas of gangrene Cold with no palpable pulses Low ABPI measurements
Neuropathic ulcers
Commonly over plantar surface of metatarsal head and plantar surface of hallux
The plantar neuropathic ulcer is the condition that most commonly leads to amputation in diabetic patients
Due to pressure
Management includes cushioned shoes to reduce callus formation
Pyoderma gangrenosum
Image sourced from Wikipedia
Associated with inflammatory bowel disease/RA
Can occur at stoma sites
Erythematous nodules or pustules which ulcerate
A 79 year old retired teacher has had an ulcer for 15 years. It is at the medial malleolus and has associated lipodermatosclerosis of the lower limb. The ulcer base is heaped up and irregular.
A. Mixed ulcer B. Chronic obliterative arterial disease C. Superficial venous insufficiency D. Deep venous insufficiency E. Neuropathic ulcer F. Basal cell carcinoma G. Squamous cell carcinoma
Squamous cell carcinoma
If after many years an ulcer becomes heaped up and irregular, with rolled edges then suspect a squamous cell carcinoma.
Venous leg ulcers
Most due to venous hypertension, secondary to chronic venous insufficiency (other causes include calf pump dysfunction or neuromuscular disorders)
Ulcers form due to capillary fibrin cuff or leucocyte sequestration
Features of venous insufficiency include oedema, brown pigmentation, lipodermatosclerosis, eczema
Location above the ankle, painless
Deep venous insufficiency is related to previous DVT and superficial venous insufficiency is associated with varicose veins
Doppler ultrasound looks for presence of reflux and duplex ultrasound looks at the anatomy/ flow of the vein
Management: 4 layer compression banding after exclusion of arterial disease or surgery
If fail to heal after 12 weeks or >10cm2 skin grafting may be needed
Marjolin’s ulcer
Image sourced from Wikipedia
Squamous cell carcinoma
Occurring at sites of chronic inflammation e.g; burns, osteomyelitis after 10-20 years
Mainly occur on the lower limb
Arterial ulcers Occur on the toes and heel Painful There may be areas of gangrene Cold with no palpable pulses Low ABPI measurements
Neuropathic ulcers
Commonly over plantar surface of metatarsal head and plantar surface of hallux
The plantar neuropathic ulcer is the condition that most commonly leads to amputation in diabetic patients
Due to pressure
Management includes cushioned shoes to reduce callus formation
Pyoderma gangrenosum
Image sourced from Wikipedia
Associated with inflammatory bowel disease/RA
Can occur at stoma sites
Erythematous nodules or pustules which ulcerate
Which of the following structures is not transmitted by the jugular foramen?
Hypoglossal nerve Accessory nerve Internal jugular vein Inferior petrosal sinus Vagus nerve
Hypoglossal nerve
The jugular foramen may be divided into three compartments:
Anterior compartment transmits the inferior petrosal sinus
Middle compartment transmits cranial nerves IX, X and XI
Posterior compartment transmits the sigmoid sinus
Foramen ovale Sphenoid bone Otic ganglion V3 (Mandibular nerve:3rd branch of trigeminal) Accessory meningeal artery Lesser petrosal nerve Emissary veins
Foramen spinosum Sphenoid bone Middle meningeal artery
Meningeal branch of the Mandibular nerve
Foramen rotundum Sphenoid bone Maxillary nerve (V2)
Foramen lacerum/ carotid canal Sphenoid bone Base of the medial pterygoid plate.
Internal carotid artery*
Nerve and artery of the pterygoid canal
Jugular foramen Temporal bone Anterior: inferior petrosal sinus
Intermediate: glossopharyngeal, vagus, and accessory nerves.
Posterior: sigmoid sinus (becoming the internal jugular vein) and some meningeal branches from the occipital and ascending pharyngeal arteries.
Foramen magnum Occipital bone Anterior and posterior spinal arteries
Vertebral arteries
Medulla oblongata
Stylomastoid foramen Temporal bone Stylomastoid artery
Facial nerve
Superior orbital fissure Sphenoid bone Oculomotor nerve (III)
Recurrent meningeal artery
Trochlear nerve (IV)
Lacrimal, frontal and nasociliary branches of ophthalmic nerve (V1)
Abducent nerve (VI)
Superior ophthalmic vein
A 45 year old motor cyclist sustains a tibial fracture and is noted to have anaesthesia of the web space between his first and second toes. Which of the nerves listed below is most likely to be compromised?
Superficial peroneal nerve Deep peroneal nerve Sural nerve Long saphenous nerve Tibial nerve
The deep peroneal nerve lies in the anterior muscular compartment of the lower leg and can be compromised by compartment syndrome affecting this area. It provides cutaneous sensation to the first web space. The superficial peroneal nerve provides more lateral cutaneous innervation.
Deep peroneal nerve
Origin From the common peroneal nerve, at the lateral aspect of the fibula, deep to peroneus longus
Nerve root values L4, L5, S1, S2
Course and relation
Pierces the anterior intermuscular septum to enter the anterior compartment of the lower leg
Passes anteriorly down to the ankle joint, midway between the two malleoli
Terminates In the dorsum of the foot
Muscles innervated
Tibialis anterior
Extensor hallucis longus
Extensor digitorum longus
Peroneus tertius
Extensor digitorum brevis
Cutaneous innervation Web space of the first and second toes
Actions
Dorsiflexion of ankle joint
Extension of all toes (extensor hallucis longus and extensor digitorum longus)
Inversion of the foot
After its bifurcation past the ankle joint, the lateral branch of the deep peroneal nerve innervates the extensor digitorum brevis and the extensor hallucis brevis
The medial branch supplies the web space between the first and second digits.
Damage to the accessory nerve during a lymph node excision biopsy.
A. Teres major B. Brachialis C. Serratus anterior D. Trapezius E. Flexor digitorum profundus F. Biceps G. Supinator H. Adductor pollicis I. Abductor pollicis brevis J. Abductor digiti minimi
Trapezius
Nerve lesions during surgery
A variety of different procedures carry the risk of iatrogenic nerve injury. These are important not only from the patients perspective but also from a medicolegal standpoint.
The following operations and their associated nerve lesions are listed here:
Posterior triangle lymph node biopsy and accessory nerve lesion.
Lloyd Davies stirrups and common peroneal nerve.
Thyroidectomy and laryngeal nerve.
Anterior resection of rectum and hypogastric autonomic nerves.
Axillary node clearance; long thoracic nerve, thoracodorsal nerve and intercostobrachial nerve.
Inguinal hernia surgery and ilioinguinal nerve.
Varicose vein surgery- sural and saphenous nerves.
Posterior approach to the hip and sciatic nerve.
Carotid endarterectomy and hypoglossal nerve.
There are many more, with sound anatomical understanding of the commonly performed procedures the incidence of nerve lesions can be minimised. They commonly occur when surgeons operate in an unfamiliar tissue plane or by blind placement of haemostats (not recommended).
Damage to the median nerve during a carpal tunnel release.
A. Teres major B. Brachialis C. Serratus anterior D. Trapezius E. Flexor digitorum profundus F. Biceps G. Supinator H. Adductor pollicis I. Abductor pollicis brevis J. Abductor digiti minimi
Abductor pollicis brevis
Adductor pollicis and abductor digiti minimi are innervated by the ulnar nerve
Nerve lesions during surgery
A variety of different procedures carry the risk of iatrogenic nerve injury. These are important not only from the patients perspective but also from a medicolegal standpoint.
The following operations and their associated nerve lesions are listed here:
Posterior triangle lymph node biopsy and accessory nerve lesion.
Lloyd Davies stirrups and common peroneal nerve.
Thyroidectomy and laryngeal nerve.
Anterior resection of rectum and hypogastric autonomic nerves.
Axillary node clearance; long thoracic nerve, thoracodorsal nerve and intercostobrachial nerve.
Inguinal hernia surgery and ilioinguinal nerve.
Varicose vein surgery- sural and saphenous nerves.
Posterior approach to the hip and sciatic nerve.
Carotid endarterectomy and hypoglossal nerve.
There are many more, with sound anatomical understanding of the commonly performed procedures the incidence of nerve lesions can be minimised. They commonly occur when surgeons operate in an unfamiliar tissue plane or by blind placement of haemostats (not recommended).
Injury to the radial nerve in a humeral shaft fracture.
A. Teres major B. Brachialis C. Serratus anterior D. Trapezius E. Flexor digitorum profundus F. Biceps G. Supinator H. Adductor pollicis I. Abductor pollicis brevis J. Abductor digiti minimi
Supinator
Nerve lesions during surgery
A variety of different procedures carry the risk of iatrogenic nerve injury. These are important not only from the patients perspective but also from a medicolegal standpoint.
The following operations and their associated nerve lesions are listed here:
Posterior triangle lymph node biopsy and accessory nerve lesion.
Lloyd Davies stirrups and common peroneal nerve.
Thyroidectomy and laryngeal nerve.
Anterior resection of rectum and hypogastric autonomic nerves.
Axillary node clearance; long thoracic nerve, thoracodorsal nerve and intercostobrachial nerve.
Inguinal hernia surgery and ilioinguinal nerve.
Varicose vein surgery- sural and saphenous nerves.
Posterior approach to the hip and sciatic nerve.
Carotid endarterectomy and hypoglossal nerve.
There are many more, with sound anatomical understanding of the commonly performed procedures the incidence of nerve lesions can be minimised. They commonly occur when surgeons operate in an unfamiliar tissue plane or by blind placement of haemostats (not recommended).
A patient undergoes a femoral hernia repair and at operation the surgeon decides to enter the abdominal cavity to resect small bowel. She makes a transverse incision two thirds of the way between umbilicus and the symphysis pubis. Which of the structures listed below is least likely to be divided?
Rectus abdominis External oblique aponeurosis Peritoneum Fascia transversalis Posterior lamina of the rectus sheath
Posterior lamina of the rectus sheath
An incision at this level lies below the arcuate line and the posterior wall of the rectus sheath is deficient at this level.
Rectus abdominis muscle
The rectus sheath is formed by the aponeuroses of the lateral abdominal wall muscles. The rectus sheath has a composition that varies according to anatomical level.
- Above the costal margin the anterior sheath is composed of external oblique aponeurosis, the costal cartilages are posterior to it.
- From the costal margin to the arcuate line, the anterior rectus sheath is composed of external oblique aponeurosis and the anterior part of the internal oblique aponeurosis. The posterior part of the internal oblique aponeurosis and transversus abdominis form the posterior rectus sheath.
- Below the arcuate line the aponeuroses of all the abdominal muscles lie in anterior aspect of the rectus sheath. Posteriorly lies the transversalis fascia and peritoneum.
The arcuate line is the point at which the inferior epigastric vessels enter the rectus sheath.
A 44 year old lady has undergone a mastectomy and axillary node clearance. Post operatively, she notices a patch of anaesthesia of her axillary skin when she applies an underarm deodorant.
A. Medial pectoral nerve B. Thoracodorsal nerve C. Lateral pectoral nerve D. Intercostobrachial nerve E. Medial cord of the brachial plexus F. Long thoracic nerve G. Axillary nerve H. Accessory nerve
Intercostobrachial nerve
The intercostobrachial nerves traverse the axilla and innervate the overlying skin. These can be injured or divided during axillary surgery and the result is anaesthesia of the overlying skin.
Axilla
Boundaries of the axilla
Medially Chest wall and Serratus anterior
Laterally Humeral head
Floor Subscapularis
Anterior aspect Lateral border of Pectoralis major
Fascia Clavipectoral fascia
Content:
Long thoracic nerve (of Bell) Derived from C5-C7 and passes behind the brachial plexus to enter the axilla. It lies on the medial chest wall and supplies serratus anterior. Its location puts it at risk during axillary surgery and damage will lead to winging of the scapula.
Thoracodorsal nerve and thoracodorsal trunk Innervate and vascularise latissimus dorsi.
Axillary vein Lies at the apex of the axilla, it is the continuation of the basilic vein. Becomes the subclavian vein at the outer border of the first rib.
Intercostobrachial nerves Traverse the axillary lymph nodes and are often divided during axillary surgery. They provide cutaneous sensation to the axillary skin.
Lymph nodes The axilla is the main site of lymphatic drainage for the breast.
A 44 year old lady has undergone a mastectomy and axillary node clearance to treat breast cancer. Post operatively, it is noted that she has winging of the scapula.
A. Medial pectoral nerve B. Thoracodorsal nerve C. Lateral pectoral nerve D. Intercostobrachial nerve E. Medial cord of the brachial plexus F. Long thoracic nerve G. Axillary nerve H. Accessory nerve
Long thoracic nerve
Injury to the long thoracic nerve (which innervates the serratus anterior) can occur as it lies at the medial aspect of the axilla, winging of the scapula will then result.
Axilla
Boundaries of the axilla
Medially Chest wall and Serratus anterior
Laterally Humeral head
Floor Subscapularis
Anterior aspect Lateral border of Pectoralis major
Fascia Clavipectoral fascia
Content:
Long thoracic nerve (of Bell) Derived from C5-C7 and passes behind the brachial plexus to enter the axilla. It lies on the medial chest wall and supplies serratus anterior. Its location puts it at risk during axillary surgery and damage will lead to winging of the scapula.
Thoracodorsal nerve and thoracodorsal trunk Innervate and vascularise latissimus dorsi.
Axillary vein Lies at the apex of the axilla, it is the continuation of the basilic vein. Becomes the subclavian vein at the outer border of the first rib.
Intercostobrachial nerves Traverse the axillary lymph nodes and are often divided during axillary surgery. They provide cutaneous sensation to the axillary skin.
Lymph nodes The axilla is the main site of lymphatic drainage for the breast.
A 44 year old lady who works as an interior decorator has undergone a mastectomy and axillary node clearance to treat breast cancer. Post operatively, she comments that her arm easily becomes fatigued when she is painting walls.
A. Medial pectoral nerve B. Thoracodorsal nerve C. Lateral pectoral nerve D. Intercostobrachial nerve E. Medial cord of the brachial plexus F. Long thoracic nerve G. Axillary nerve H. Accessory nerve
Thoracodorsal nerve
The most likely explanation for this is that the thoracodorsal nerve has been injured. This will result in atrophy of latissimus dorsi and this will become evident with repetitive arm movements where the arm is elevated and moving up and down (such as in painting). Injury to the pectoral nerves may produce a similar picture but this pattern of injury is very rare and the pectoral nerves are seldom injured in breast surgery.
Axilla
Boundaries of the axilla
Medially Chest wall and Serratus anterior
Laterally Humeral head
Floor Subscapularis
Anterior aspect Lateral border of Pectoralis major
Fascia Clavipectoral fascia
Content:
Long thoracic nerve (of Bell) Derived from C5-C7 and passes behind the brachial plexus to enter the axilla. It lies on the medial chest wall and supplies serratus anterior. Its location puts it at risk during axillary surgery and damage will lead to winging of the scapula.
Thoracodorsal nerve and thoracodorsal trunk Innervate and vascularise latissimus dorsi.
Axillary vein Lies at the apex of the axilla, it is the continuation of the basilic vein. Becomes the subclavian vein at the outer border of the first rib.
Intercostobrachial nerves Traverse the axillary lymph nodes and are often divided during axillary surgery. They provide cutaneous sensation to the axillary skin.
Lymph nodes The axilla is the main site of lymphatic drainage for the breast.
A 35 year old farm labourer injures the posterior aspect of his hand with a mechanical scythe. He severs some of his extensor tendons in this injury. How many tunnels lie in the extensor retinaculum that transmit the tendons of the extensor muscles?
One Three Four Five Six
There are six tunnels, each lined by its own synovial sheath.
Extensor retinaculum
The extensor rentinaculum is a thickening of the deep fascia that stretches across the back of the wrist and holds the long extensor tendons in position.
Its attachments are:
The pisiform and triquetral medially
The end of the radius laterally
Structures related to the extensor retinaculum
Structures superficial to the retinaculum
Basilic vein
Dorsal cutaneous branch of the ulnar nerve
Cephalic vein
Superficial branch of the radial nerve
Structures passing deep to the extensor retinaculum
Extensor carpi ulnaris tendon
Extensor digiti minimi tendon
Extensor digitorum and extensor indicis tendon
Extensor pollicis longus tendon
Extensor carpi radialis longus tendon
Extensor carpi radialis brevis tendon
Abductor pollicis longus and extensor pollicis brevis tendons
Beneath the extensor retinaculum fibrous septa form six compartments that contain the extensor muscle tendons. Each compartment has its own synovial sheath.
The radial artery
The radial artery passes between the lateral collateral ligament of the wrist joint and the tendons of the abductor pollicis longus and extensor pollicis brevis.
Which of the muscles listed below is not innervated by the median nerve?
Flexor pollicis brevis Lateral two lumbricals Pronator teres Opponens pollicis Adductor pollicis
Adductor pollicis is innervated by the ulnar nerve.
Medial two lumbricals innervated by the ulnar nerve.
Median nerve
The median nerve is formed by the union of a lateral and medial root respectively from the lateral (C5,6,7) and medial (C8 and T1) cords of the brachial plexus; the medial root passes anterior to the third part of the axillary artery. The nerve descends lateral to the brachial artery, crosses to its medial side (usually passing anterior to the artery). It passes deep to the bicipital aponeurosis and the median cubital vein at the elbow.
It passes between the two heads of the pronator teres muscle, and runs on the deep surface of flexor digitorum superficialis (within its fascial sheath).
Near the wrist it becomes superficial between the tendons of flexor digitorum superficialis and flexor carpi radialis, deep to palmaris longus tendon. It passes deep to the flexor retinaculum to enter the palm, but lies anterior to the long flexor tendons within the carpal tunnel.
Branches Region Branch Upper arm No branches, although the nerve commonly communicates with the musculocutaneous nerve Forearm Pronator teres Flexor carpi radialis Palmaris longus Flexor digitorum superficialis Flexor pollicis longus Flexor digitorum profundus (only the radial half) Distal forearm Palmar cutaneous branch Hand (Motor) Motor supply (LOAF) Lateral 2 lumbricals Opponens pollicis Abductor pollicis brevis Flexor pollicis brevis Hand (Sensory) Over thumb and lateral 2 ½ fingers On the palmar aspect this projects proximally, on the dorsal aspect only the distal regions are innervated with the radial nerve providing the more proximal cutaneous innervation.
Patterns of damage
Damage at wrist
e.g. carpal tunnel syndrome
paralysis and wasting of thenar eminence muscles and opponens pollicis (ape hand deformity)
sensory loss to palmar aspect of lateral (radial) 2 ½ fingers
Damage at elbow, as above plus:
unable to pronate forearm
weak wrist flexion
ulnar deviation of wrist
Anterior interosseous nerve (branch of median nerve)
leaves just below the elbow
results in loss of pronation of forearm and weakness of long flexors of thumb and index finger
A 44 year old lady is undergoing an abdominal hysterectomy and the ureter is identified during the ligation of the uterine artery. At which site does it insert into the bladder?
Posterior Apex Anterior Base Superior aspect of the lateral side
The ureters enter the bladder at the upper lateral aspect of the base of the bladder. They are about 5cm apart from each other in the empty bladder. Internally this aspect is contained within the bladder trigone.
Ureter
25-35 cm long
Muscular tube lined by transitional epithelium
Surrounded by thick muscular coat. Becomes 3 muscular layers as it crosses the bony pelvis
Retroperitoneal structure overlying transverse processes L2-L5
Lies anterior to bifurcation of iliac vessels
Blood supply is segmental; renal artery, aortic branches, gonadal branches, common iliac and internal iliac
Lies beneath the uterine artery
An injured axillary artery is ligated between the thyrocervical trunk of the subclavian and subscapular artery. Subsequent collateral circulation is likely to result in reversal of blood flow in which of the vessels listed below?
Circumflex scapular artery Transverse cervical artery Posterior intercostal arteries Suprascapular artery Profunda brachii artery
The circumflex scapular artery is a branch of the subscapular artery and normally supplies the muscle on the dorsal aspect of the scapula. In this instance, flow is reversed in the circumflex scapular and subscapular arteries forming a collateral circulation around the scapula.
It’s an easy question really, we just made the wording difficult (on purpose). It is asking about the branches of the axillary artery and knowledge of the fact that there is an extensive collateral network around the shoulder joint. As a result, the occlusion of the proximal aspect of the circumflex humeral inflow (from the axillary artery) ceases and there is then retrograde flow through it from collaterals.
Axillary artery
The axillary artery extends from the outer border of the first rib to the lower border of teres major, where it becomes the brachial artery. The vessel is subdivided into three zones; the first part lies above pectoralis minor, the second part is behind the muscle and the third part lies inferior to it.
First part
Together with the axillary vein, the artery is enclosed within the cords of the brachial plexus. Both vessels are contained within the axillary sheath, a prolongation of the prevertebral fascia. Posteriomedial to the sheath lies the first intercostal space, the superior aspect of the serratus anterior and the long thoracic nerve. Within the sheath, the medial cord of the brachial plexus lies behind the artery. Anteriorly lies the clavipectoral fascia. Superolaterally, lie the lateral and posterior cords of the brachial plexus. Inferomedially lies the axillary vein.
Second part
Posterior to the second part lies the posterior cord of the brachial plexus and the subscapularis muscle. Anteriorly, lie pectoralis minor and major. The lateral cord of the brachial plexus lies laterally. Medially, lies the medial cord of the brachial plexus, here it separates the artery from the vein.
Third part
Posterior to the artery lie suscapularis, latissimus dorsi and teres major. Interspersed between the vessel and subscapularis are the axillary and radial nerves. Anterior to the vessel is the medial root of the median nerve. Laterally, the lies the median and musculocutaneous nerves and coracobrachialis. The axillary vein is related medially.
Branches of the axillary artery Highest thoracic artery Thoraco-acromial artery Lateral thoracic artery Subscapular artery Posterior circumflex humeral artery Anterior circumflex humeral artery
A 20 year old lady presents with pain on the medial aspect of her thigh. Investigations show a large ovarian cyst. Compression of which of the nerves listed below is the most likely underlying cause?
Sciatic Genitofemoral Obturator Ilioinguinal Femoral cutaneous
Obturator nerve
The cutaneous branch of the obturator nerve is frequently absent. However, the obturator nerve is a recognised contributor to innervation of the medial thigh and large pelvic tumours may compress this nerve with resultant pain radiating distally.
The obturator nerve arises from L2, L3 and L4 by branches from the ventral divisions of each of these nerve roots. L3 forms the main contribution and the second lumbar branch is occasionally absent. These branches unite in the substance of psoas major, descending vertically in its posterior part to emerge from its medial border at the lateral margin of the sacrum. It then crosses the sacroiliac joint to enter the lesser pelvis, it descends on obturator internus to enter the obturator groove. In the lesser pelvis the nerve lies lateral to the internal iliac vessels and ureter, and is joined by the obturator vessels lateral to the ovary or ductus deferens.
Supplies
Medial compartment of thigh
Muscles supplied: external obturator, adductor longus, adductor brevis, adductor magnus (not the lower part-sciatic nerve), gracilis
The cutaneous branch is often absent. When present, it passes between gracilis and adductor longus near the middle part of the thigh, and supplies the skin and fascia of the distal two thirds of the medial aspect.
Obturator canal
Connects the pelvis and thigh: contains the obturator artery, vein, nerve which divides into anterior and posterior branches.
A 21 year old man undergoes surgical removal of an impacted 3rd molar. Post operatively, he is noted to have anaesthesia on the anterolateral aspect of the tongue. What is the most likely explanation?
Injury to the hypoglossal nerve Injury to the inferior alveolar nerve Injury to the lingual nerve Injury to the mandibular branch of the facial nerve Injury to the glossopharyngeal nerve
The lingual nerve is closely related to the third molar and up to 10% of patients undergoing surgical extraction of these teeth may subsequently develop a lingual neuropraxia. The result is anaesthesia of the ipsilateral anterior aspect of the tongue. The inferior alveolar nerve innervates the teeth themselves.
Lingual nerve
Sensory nerve to the mucosa of the presulcal part of the tongue, floor of mouth and mandibular lingual gingivae
Arises from posterior trunk of the mandibular nerve (branch of trigeminal)
Course runs past tensor veli palatini and lateral pterygoid (where it is joined by the chorda tympani branch of the facial nerve). Emerging from the cover of the lateral pterygoid it proceeds antero inferiorly lying on the surface of the medial pterygoid and lies close to the medial aspect of the mandibular ramus. At the junction of the vertical and horizontal rami of the mandible it is anterior to the inferior alveolar nerve. It then passes below the mandibular attachment of the superior pharyngeal constrictor. Eventually, it lies on the periosteum of the root of the third molar tooth. It then passes medial to the mandibular origin of mylohyoid and then passes forwards on the inferior surface of this muscle
What is the most important structure involved in supporting the uterus?
Round ligament Broad ligament Uterosacral ligaments Cardinal ligaments Central perineal tendon
The central perineal tendon provides the main structural support to the uterus. Damage to this structure is commonly associated with the development of pelvic organ prolapse, even when other structures are intact.
Uterus
The non pregnant uterus resides entirely within the pelvis. The peritoneum invests the uterus and the structure is contained within the peritoneal cavity. The blood supply to the uterine body is via the uterine artery (branch of the internal iliac). The uterine artery passes from the inferior aspect of the uterus (lateral to the cervix) and runs alongside the uterus. It frequently anastomoses with the ovarian artery superiorly. Inferolaterally the ureter is a close relation and ureteric injuries are a recognised complication when pathology brings these structures into close proximity.
The supports of the uterus include the central perineal tendon (the most important). The lateral cervical, round and uterosacral ligaments are condensations of the endopelvic fascia and provide additional structural support.
A 34 year old lady suffers from hyperparathyroidism. The right inferior parathyroid is identified as having an adenoma and is scheduled for resection. From which of the following embryological structures is it derived?
Second pharyngeal pouch Third pharyngeal pouch Fourth pharyngeal pouch First pharyngeal pouch None of the above
The inferior parathyroid is a derivative of the third pharyngeal pouch. The superior parathyroid originates from the fourth pharyngeal pouch.
Parathyroid glands- anatomy
Four parathyroid glands
Located posterior to the thyroid gland
They lie within the pretracheal fascia
Embryology
The parathyroids develop from the extremities of the third and fourth pharyngeal pouches. The parathyroids derived from the fourth pharyngeal pouch are located more superiorly and are associated with the thyroid gland. Those derived from the third pharyngeal pouch lie more inferiorly and may become associated with the thymus.
Blood supply
The blood supply to the parathyroid glands is derived from the inferior and superior thyroid arteries[1]. There is a rich anastomosis between the two vessels. Venous drainage is into the thyroid veins.
Relations Laterally Common carotid Medially Recurrent laryngeal nerve, trachea Anterior Thyroid Posterior Pretracheal fascia
A 23 year old man falls and slips at a nightclub. A shard of glass penetrates the skin at the level of the medial epicondyle, which of the following sequelae is least likely to occur?
Atrophy of the first dorsal interosseous muscle Difficulty in abduction of the the 2nd, 3rd, 4th and 5th fingers Claw like appearance of the hand Loss of sensation on the anterior aspect of the 5th finger Partial denervation of flexor digitorum profundus
Claw like appearance of the hand
Injury to the ulnar nerve in the mid to distal forearm will typically produce a claw hand. This consists of flexion of the 4th and 5th interphalangeal joints and extension of the metacarpophalangeal joints. The effects are potentiated when flexor digitorum profundus is not affected, and the clawing is more pronounced.More proximally sited ulnar nerve lesions produce a milder clinical picture owing to the simultaneous paralysis of flexor digitorum profundus (ulnar half).
This is the ‘ulnar paradox’, due to the more proximal level of transection the hand will typically not have a claw like appearance that may be seen following a more distal injury. The first dorsal interosseous muscle will be affected as it is supplied by the ulnar nerve.
Effects of injury
Damage at the wrist
Wasting and paralysis of intrinsic hand muscles (claw hand)
Wasting and paralysis of hypothenar muscles
Loss of sensation medial 1 and half fingers
Damage at the elbow
Radial deviation of the wrist
Clawing less in 4th and 5th digits
Ulnar nerve
Origin
C8, T1
Supplies (no muscles in the upper arm) Flexor carpi ulnaris Flexor digitorum profundus Flexor digiti minimi Abductor digiti minimi Opponens digiti minimi Adductor pollicis Interossei muscle Third and fourth lumbricals Palmaris brevis
Path
Posteromedial aspect of upper arm to flexor compartment of forearm, then along the ulnar. Passes beneath the flexor carpi ulnaris muscle, then superficially through the flexor retinaculum into the palm of the hand.
A 78 year old man is due to undergo an endarterectomy of the internal carotid artery. Which of the following nervous structures are most at risk during the dissection?
Recurrent laryngeal nerve Sympathetic chain Hypoglossal nerve Phrenic nerve Lingual nerve
Nerves at risk during a carotid endarterectomy:
Hypoglossal nerve
Greater auricular nerve
Superior laryngeal nerve
During a carotid endarterectomy the sternocleidomastoid muscle is dissected, with ligation of the common facial vein and then the internal jugular is dissected exposing the common and the internal carotid arteries. The nerves at risk during the operation include:
Hypoglossal nerve
Greater auricular nerve
Superior laryngeal nerve
The sympathetic chain lies posteriorly and is less prone to injury in this procedure.
The internal carotid artery is formed from the common carotid opposite the upper border of the thyroid cartilage. It extends superiorly to enter the skull via the carotid canal. From the carotid canal it then passes through the cavernous sinus, above which it divides into the anterior and middle cerebral arteries.
Relations in the carotid canal Internal carotid plexus Cochlea and middle ear cavity Trigeminal ganglion (superiorly) Leaves canal lies above the foramen lacerum
Path and relations in the cranial cavity
The artery bends sharply forwards in the cavernous sinus, the aducens nerve lies close to its inferolateral aspect. The oculomotor, trochlear, opthalmic and, usually, the maxillary nerves lie in the lateral wall of the sinus. Near the superior orbital fissure it turns posteriorly and passes postero-medially to pierce the roof of the cavernous sinus inferior to the optic nerve. It then passes between the optic and oculomotor nerves to terminate below the anterior perforated substance by dividing into the anterior and middle cerebral arteries.
Branches Anterior and middle cerebral artery Ophthalmic artery Posterior communicating artery Anterior choroid artery Meningeal arteries Hypophyseal arteries
Which of the structures listed below articulates with the head of the radius superiorly?
Capitulum Trochlea Lateral epicondyle Ulna Medial epicondyle
The head of the radius articulates with the capitulum of the humerus.
Radius
The radius is one of the two long forearm bones that extends from the lateral side of the elbow to the thumb side of the wrist. It has two expanded ends, of which the distal end is the larger. Key points relating to its topography and relations are outlined below;
Upper end
Articular cartilage- covers medial > lateral side
Articulates with radial notch of the ulna by the annular ligament
Muscle attachment- biceps brachii at the tuberosity
Shaft Muscle attachment Upper third of the body Supinator Flexor digitorum superficialis Flexor pollicis longus Middle third of the body Pronator teres Lower quarter of the body Pronator quadratus Tendon of supinator longus
Lower end Quadrilateral Anterior surface- capsule of wrist joint Medial surface- head of ulna Lateral surface- ends in the styloid process Posterior surface: 3 grooves containing: 1. Tendons of extensor carpi radialis longus and brevis 2. Tendon of extensor pollicis longus 3. Tendon of extensor indicis
Which of the following fascial structures encases the apex of the lungs?
Waldeyers fascia Sibsons fascia Pretracheal fascia Clavipectoral fascia None of the above
Sibson’s fascia overlies the apices of both lungs
The suprapleural fascia (Sibson’s fascia) runs from C7 to the first rib and overlies the apex of both lungs.It lies between the parietal pleura and the thoracic cage.
Lung anatomy
The right lung is composed of 3 lobes divided by the oblique and transverse fissures. The left lung has two lobes divided by the oblique fissure.The apex of both lungs is approximately 4cm superior to the sterno-costal joint of the first rib. Immediately below this is a sulcus created by the subclavian artery.
Peripheral contact points of the lung
Base: diaphragm
Costal surface: corresponds to the cavity of the chest
Mediastinal surface: Contacts the mediastinal pleura. Has the cardiac impression. Above and behind this concavity is a triangular depression named the hilum, where the structures which form the root of the lung enter and leave the viscus. These structures are invested by pleura, which, below the hilum and behind the pericardial impression, forms the pulmonary ligament
Right lung
Above the hilum is the azygos vein; Superior to this is the groove for the superior vena cava and right innominate vein; behind this, and nearer the apex, is a furrow for the innominate artery. Behind the hilum and the attachment of the pulmonary ligament is a vertical groove for the oesophagus; In front and to the right of the lower part of the oesophageal groove is a deep concavity for the extrapericardiac portion of the inferior vena cava.
The root of the right lung lies behind the superior vena cava and the right atrium, and below the azygos vein.
The right main bronchus is shorter, wider and more vertical than the left main bronchus and therefore the route taken by most foreign bodies.
Image sourced from Wikipedia
Left lung
Above the hilum is the furrow produced by the aortic arch, and then superiorly the groove accommodating the left subclavian artery; Behind the hilum and pulmonary ligament is a vertical groove produced by the descending aorta, and in front of this, near the base of the lung, is the lower part of the oesophagus.
The root of the left lung passes under the aortic arch and in front of the descending aorta.
Image sourced from Wikipedia
Inferior borders of both lungs 6th rib in mid clavicular line 8th rib in mid axillary line 10th rib posteriorly The pleura runs two ribs lower than the corresponding lung level.
As regards the internal jugular vein, which of the following statements is untrue?
It lies within the carotid sheath It is the continuation of the sigmoid sinus The terminal part of the thoracic duct crosses anterior to it to insert into the right subclavian vein The hypoglossal nerve is closely related to it as it passes near the atlas The vagus nerve is closely related to it within the carotid sheath
The terminal part of the thoracic duct crosses anterior to it to insert into the right subclavian vein
Internal jugular vein
Each jugular vein begins in the jugular foramen, where they are the continuation of the sigmoid sinus. They terminate at the medial end of the clavicle where they unite with the subclavian vein.
The vein lies within the carotid sheath throughout its course. Below the skull the internal carotid artery and last four cranial nerves are anteromedial to the vein. Thereafter it is in contact medially with the internal (then common) carotid artery. The vagus lies posteromedially.
At its superior aspect, the vein is overlapped by sternocleidomastoid and covered by it at the inferior aspect of the vein.
Below the transverse process of the atlas it is crossed on its lateral side by the accessory nerve. At its mid point it is crossed by the inferior root of the ansa cervicalis.
Posterior to the vein are the transverse processes of the cervical vertebrae, the phenic nerve as it descends on the scalenus anterior, and the first part of the subclavian artery.
On the left side its also related to the thoracic duct.
At the level of the wrist joint, which of the statements below best describes the relationship of the ulnar artery to the ulnar nerve?
It lies on its radial side It lies deep to it It lies superficial to it It lies on its ulnar side None of the above
It lies on its radial side
In the middle of the forearm, the artery is overlapped by the flexor carpi ulnaris and on the flexor retinaculum it is covered by a superficial layer from that structure. In its distal two-thirds, flexor digitorum superficialis lies on its radial side, and the ulnar nerve is situated on its ulnar side.
Ulnar artery
Path
Starts: middle of antecubital fossa
Passes obliquely downward, reaching the ulnar side of the forearm at a point about midway between the elbow and the wrist. It follows the ulnar border to the wrist, crossing over the flexor retinaculum. It then divides into the superficial and deep volar arches.
Relations
Deep to- Pronator teres, Flexor carpi radialis, Palmaris longus
Lies on- Brachialis and Flexor digitorum profundus
Superficial to the flexor retinaculum at the wrist
The median nerve is in relation with the medial side of the artery for about 2.5 cm. And then crosses the vessel, being separated from it by the ulnar head of the Pronator teres
The ulnar nerve lies medially to the lower two-thirds of the artery
Branch
Anterior interosseous artery
Which of the following anatomical structures lies within the spiral groove of the humerus?
Median nerve Radial nerve Tendon of triceps Musculocutaneous nerve Axillary nerve
The radial nerve lies in this groove and may be compromised by fractures involving the shaft.
Humerus
The humerus extends from the scapula to the elbow joint. It has a body and two ends. It is almost completely covered with muscle but can usually be palpated throughout its length. The smooth rounded surface of the head articulates with the shallow glenoid cavity. The head is connected to the body of the humerus by the anatomical neck. The surgical neck is the region below the head and tubercles and where they join the shaft and is the commonest site of fracture. The capsule of the shoulder joint is attached to the anatomical neck superiorly but extends down to 1.5cm on the surgical neck.
The greater tubercle is the prominence on the lateral side of the upper end of the bone. It merges with the body below and can be felt through the deltoid inferior to the acromion. The tendons of the supraspinatus and infraspinatus are inserted into impressions on its superior aspect. The lesser tubercle is a distinct prominence on the front of the upper end of the bone. It can be palpated through the deltoid just lateral to the tip of the coracoid process.
The intertubercular groove passes on the body between the greater and lesser tubercles, continuing down from the anterior borders of the tubercles to form the edges of the groove. The tendon of biceps within its synovial sheath passes through this groove, held within it by a transverse ligament.
The posterior surface of the body is marked by a spiral groove for the radial nerve which runs obliquely across the upper half of the body to reach the lateral border below the deltoid tuberosity. Within this groove lie the radial nerve and brachial vessels and both may be affected by fractures involving the shaft of the humerus.
The lower end of the humerus is wide and flattened anteroposteriorly, and inclined anteriorly. The middle third of the distal edge forms the trochlea. Superior to this are indentations for the coronoid fossa anteriorly and olecranon fossa posteriorly. Lateral to the trochlea is a rounded capitulum which articulates with the radius.
The medial epicondyle is very prominent with a smooth posterior surface which contains a sulcus for the ulnar nerve and collateral vessels. It’s distal margin gives attachment for the ulnar collateral ligament and, in front of this, the anterior surface has an impression for the common flexor tendon.
A 24 year old man falls and sustains a fracture through his scaphoid bone. From which of the following areas does the scaphoid derive the majority of its blood supply?
From its proximal medial border From its proximal lateral border From its proximal posterior surface From the proximal end From the distal end
The blood supply to the scaphoid enters from a small non articular surface near its distal end. Transverse fractures through the scaphoid therefore carry a risk of non union.
Scaphoid bone
The scaphoid has a concave articular surface for the head of the capitate and at the edge of this is a crescentic surface for the corresponding area on the lunate.
Proximally, it has a wide convex articular surface with the radius. It has a distally sited tubercle that can be palpated. The remaining articular surface is to the lateral side of the tubercle. It faces laterally and is associated with the trapezium and trapezoid bones.
The narrow strip between the radial and trapezial surfaces and the tubercle gives rise to the radial collateral carpal ligament. The tubercle receives part of the flexor retinaculum. This area is the only part of the scaphoid that is available for the entry of blood vessels. It is commonly fractured and avascular necrosis may result.
Which of the following laryngeal tumours will not typically metastasise to the cervical lymph nodes?
Glottic Supraglottic Subglottic Transglottic Aryepiglottic fold
Glottic
The vocal cords have no lymphatic drainage and therefore this region serves as a lymphatic watershed. The supraglottic part drains to the upper deep cervical nodes through vessels piercing the thyrohyoid membrane. The sub glottic part drains to the pre laryngeal, pre tracheal and inferior deep cervical nodes. The aryepiglottic and vestibular folds have a rich lymphatic drainage and will metastasise early.
Larynx
The larynx lies in the anterior part of the neck at the levels of C3 to C6 vertebral bodies. The laryngeal skeleton consists of a number of cartilagenous segments. Three of these are paired; arytenoid, corniculate and cuneiform. Three are single; thyroid, cricoid and epiglottic. The cricoid cartilage forms a complete ring (the only one to do so).
The laryngeal cavity extends from the laryngeal inlet to the level of the inferior border of the cricoid cartilage.
Divisions of the laryngeal cavity
Laryngeal vestibule Superior to the vestibular folds
Laryngeal ventricle Lies between vestibular folds and superior to the vocal cords
Infraglottic cavity Extends from vocal cords to inferior border of the cricoid cartilage
The vocal folds (true vocal cords) control sound production. The apex of each fold projects medially into the laryngeal cavity. Each vocal fold includes:
Vocal ligament
Vocalis muscle (most medial part of thyroarytenoid muscle)
The glottis is composed of the vocal folds, processes and rima glottidis. The rima glottidis is the narrowest potential site within the larynx, as the vocal cords may be completely opposed, forming a complete barrier.
Blood supply
Arterial supply is via the laryngeal arteries, branches of the superior and inferior thyroid arteries. The superior laryngeal artery is closely related to the internal laryngeal nerve. The inferior laryngeal artery is related to the inferior laryngeal nerve. Venous drainage is via superior and inferior laryngeal veins, the former draining into the superior thyroid vein and the latter draining into the middle thyroid vein, or thyroid venous plexus.
Lymphatic drainage
The vocal cords have no lymphatic drainage and this site acts as a lymphatic watershed.
Supraglottic part Upper deep cervical nodes
Subglottic part Prelaryngeal and pretracheal nodes and inferior deep cervical nodes
The aryepiglottic fold and vestibular folds have a dense plexus of lymphatics associated with them and malignancies at these sites have a greater propensity for nodal metastasis.
A patient presents with superior vena caval obstruction. How many collateral circulations exist as alternative pathways of venous return?
None One Two Three Four
There are 4 collateral venous systems:
Azygos venous system
Internal mammary venous pathway
Long thoracic venous system with connections to the femoral and vertebral veins (2 pathways)
Despite this, venous hypertension still occurs.
Superior vena cava
Drainage Head and neck Upper limbs Thorax Part of abdominal walls
Formation
Subclavian and internal jugular veins unite to form the right and left brachiocephalic veins
These unite to form the SVC
Azygos vein joins the SVC before it enters the right atrium